Download as pdf or txt
Download as pdf or txt
You are on page 1of 112

MATHEMATICAL TOOLS

UNITS & DIMENSIONS


Copyright  EDUSQUARE
This study module contains copyright subject matter proprietary to
EDUSQUARE, the content development wing of Edusquare, Ludhiana, India. No
part of this module may be reproduced, stored in a retrieval system or
transmitted in any form or by any means, electronic, mechanical, photocopy,
recording, or otherwise, by anyone, without prior written permission from
Edusquare. Violators are liable to be legally prosecuted.

2
CONTENTS

S. No. Topic Page No.

1 CHAPTER 1: MATHEMATICAL TOOLS 4

2 CLASS ASSIGNMENT 25

3 HOME ASSIGNMENT 26

4 OBJECTIVE QUESTION BANK 32

5 CHAPTER 2: UNITS & DIMENSIONS 35

6 CLASS ASSIGNMENT 74

7 HOME ASSIGNMENT 78

8 OBJECTIVE QUESTION BANK 94

9 WINDOW TO JEE MAIN 104

10 WINDOW TO NEET 110

3
PHYSICS MATHEMATICAL TOOLS

MATHEMATICAL TOOLS
BINOMIAL THEOREM
If n is any number, positive, negative or fraction and x is any real number, such that –1 < x < 1, i.e. x lies
between –1 and +1, then according to Binomial theorem
nx n (n  1) 2 n (n  1)(n  2) 3 n (n  1)(n  2)(n  3) 4
(1  x)n  1   x  x  x  ............
1! 2! 3! 4!
where n! = n (n – 1) (n – 2) (n – 3) … 3  2  1
e.g. 3! = 3  2  1 = 6

Note:
1. If n is a positive integer, then the expansion will have (n + 1) terms.
2. If n is a negative integer or a fraction, then the number of terms in the expansion will be
infinite.
3. If |x| << 1, then only the first two terms of the expansion are significant. It is so because the
values of second and higher order terms being very very small, can be neglected. In this
case, the expansion reduces to the following simplified forms.
1  x   1  nx and 1  x   1 nx
n
n

4. In Physics, most of the time above relation will be used. Before applying above relation
make sure |x| << 1.
Examples: (x + y)2 = x2 + 2xy + y2 (second power: three terms)
(x + y)3 = x3 + 3x2y + 3xy2 + y3 (third power: four terms)
(x + y)4 = x4 + 4x3y + 6x2y2 + 4xy3 + y4 (fourth power: five terms)
Example 1: Evaluate (1001)1/3 upto six places of decimal.
1/3
  1 
1001  1000  1  1000 1   10 1  .001
1/3 1/3 1/3
Solution: 
  1000  
 11  
 1   1 
 10 1  .001  
3 3 
.001  ...
2

 3 2! 
 
 1 
 10 1  0.0003333   0.000001  ....
 9 
 101  0.0003333  0.0000001  10.003332

MEASUREMENT OF ANGLE & RELATIONSHIP BETWEEN DEGREE & RADIAN


In a circle of radius r,
l length of arc
 (in radian)  
r radius of circle
 For complete circle
l 2 r
 (in rad)    2  = angle subtended at the center of circle
r r
Thus, 2  3600 i.e.   1800

4
MATHEMATICAL TOOLS PHYSICS

Conversion Formulae

1 degree = ( 0.02) radian
180

Degree to Radian: Multiply by
180
1 radian = 57.3o
180
Radian to Degree: Multiply by


Example 2: (a) Convert 45º to radian (b) Convert rad to degree
6
Solution:
 
(a) 45 = rad
180 4
 180
(b)  = 30º
6 
Note: In Calculations, if we are dealing with only angle (without trigonometric ratio), then angle
must be taken into radian.

MEASUREMENT OF POSITIVE & NEGATIVE ANGLES


y y

x
Positive Negative
Measure
measure
x

An angle in the xy-plane is said to be in standard position if its vertex lies at the origin and its initial ray lies
along the positive x-axis (Fig.). Angles measured counterclockwise from the positive x-axis are assigned
positive measures; angles measured clockwise are assigned negative measures.
(Rules for finding Trigonometric ratio of angles greater than 90°)


Trigonometric ratios of 
2
     
sin      cos  cos       sin  tan       cot 
2  2  2 
     
cosec      sec  cot       tan  sec      cosec
2  2  2 

5
PHYSICS MATHEMATICAL TOOLS

Trigonometric ratios of    
sin       sin  tan      tan  cosec      cosec
cos       cos cot      cot  sec       sec
3
Trigonometric ratios of 
2
 3   3   3 
sin       cos  tan      cot  cosec       sec 
 2   2   2 
 3   3   3 
cos       sin  cot      tan  sec      cosec
 2   2   2 
3
Trigonometric ratios of 
2
 3   3   3 
sin       cos  cos      sin  tan       cot 
 2   2   2 
 3   3   3 
cot       tan  sec      cosec cosec       sec 
 2   2   2 
Example 3: Evaluate
(a) sin 135° (b) cos 135° (c) cos 210° (d) tan 210°
Solution:
1
(a) sin 135° = sin (90° + 45°) = cos 45° =
2
1
(b) cos 135° = cos (90° + 45°) = – sin 45° = 
2
3
(c) cos 210° = cos (180° + 30°) = – cos30° = 
2
1
(d) tan 210° = tan (180° + 30°) = tan 30° =
3
Think, and fill up the blank blocks in following table.
    2 5 7 4 3 5 11
0  2
6 4 3 2 3 6 6 3 2 3 6
1 1 3
sin 0 1
2 2 2
3 1 1
cos 1 0
2 2 2
1
tan 0 1 3 N.D.
3

3 4 4 3
Note: sin 37o  cos 37o  sin 53o  cos53o 
5 5 5 5

IMPORTANT TRIGONOMETRIC IDENTITIES

sin 2   cos2   1
sec2  tan 2   1
cosec2  cot 2  1

6
MATHEMATICAL TOOLS PHYSICS

Addition/ Subtraction Formulae

sin  A  B   sin A cos B  cos A sin B


cos  A  B   cos A cos B sin A sin B
sin  A  B   sin  A  B   2sin A cos B
sin  A  B   sin  A  B   2cos A sin B
cos  A  B   cos  A  B   2cos A cos B
cos  A  B   cos  A  B   2sin A sin B
tan A  tan B
tan  A  B  
1 tan A tan B
Multiple Angle Formulae

sin 2 A  2sin Acos A


cos 2A  cos2 A  sin 2 A  2cos2 A  1  1  2sin 2 A
2 tan A
tan 2 A 
1  tan 2 A
C-D Formulae

CD CD
sin C  sin D  2sin cos
2 2
CD CD
sin C  sin D  2cos sin
2 2
CD CD
cos C  cos D  2cos cos
2 2
CD D C
cos C  cos D  2sin sin
2 2
1  cos 2 1  cos 2
Note: sin 2   ; cos 2  
2 2

Example 4: Prove that sin 5A + sin 3A = 2sin 4A cos A


Solution: L.H.S. sin 5A + sin 3A = 2sin 4A cos A = R.H.S.
CD CD
[Using, sin C  sin D  2sin cos ]
2 2
Graph of sin x in [0, 2]

7
PHYSICS MATHEMATICAL TOOLS

Graph of cos x in [0, 2]

STRAIGHT LINE
Slope of a Line

The slope of a line joining two points P (x1, y1) and Q (x2, y2) is given by
y2  y1
m  tan  
x2  x1
where  is the angle which the line makes with the positive direction of x-axis.
y y

 
x x

Figure - 1 Figure - 2
In figure-1 slope is positive as  < 90 & in figure-2 slope is negative as 90o <  < 180o.
o

Note: If in any question, it is mentioned that slope is 1 in 100, then it means sin  = 1/100 not
tan  = 1/100

Equation of a Straight Line


The straight line can be represented by the equation y = mx + c where, m = slope of line and c = intercept
on y-axis
The equation of x-axis is y = 0
The equation of y-axis is x = 0
y y
y m= m=1
m=0 (B) (C) 45º
(A)
x
x x
y (1)
y (E) m1=m2
m=–3 (2)
(D)
120 x
º x

Note: If a line passes through point (x1, y1) and (x2, y2) then equation of line
y2  y1
y  y1   x  x1 
x2  x1

8
MATHEMATICAL TOOLS PHYSICS

CIRCLE
(i) The circle with centre as origin & radius ‘r’ has the equation; x2 + y2 = r2.

.
(ii) The circle with centre (h, k) & radius ‘r’ has the equation;  x  h    y  k   r 2
2 2

PARABOLA
Symmetric Parabola

(i) y 2  kx Opening on right side


(ii) y 2  kx Opening on left side
(iii) x  ky
2
Opening in upward direction
(iv) x  ky
2
Opening in downward direction
2
Graph for y = 4ax
Y

A F
X
(0, 0) (a, 0)

F = focus = (a, 0); A = Vertex = (0, 0)

Asymmetric Parabola Rectangular Hyperbola

k
y  ax  bx 2 where a and b are positive constants. y
x
Y

ELLIPSE

x2 y 2
Equation of an ellipse is given by  1
a 2 b2
Distance between F and F = 2c; where c  a  b
2 2

a = OA = semi major axis & b = OB = semi minor axis


c
Eccentricity  e   & Area of ellipse = ab
a

9
PHYSICS MATHEMATICAL TOOLS

LOGARITHM
Logarithm of a Number
Logarithm or log of a number to a given base is equal to that power to which if base is raised, we get the
number.
log a N  x  a x  N ,  a  0, a  1 & N  0
If a = 10, then we write log b rather than log10b.
If a = e, we write ln b rather than loge b. Here ‘e’ is called as Napiers base & has numerical value equal
to 2.7182.

Note: “Log” being the abbreviation of the word “logarithm’’

Thus log2128 = 7; since 27 = 128


log3 81 = 4; since 34 = 81
log13 1 = 0; since 130 = 1.
Conversely, a x  m  x  log a m
Thus 23 = 8  x = log2 8 = 3
33 = 27  log3 27 = 3
1
91/2 = 3  log9 3 =
2
1
271/3 = 3  log27 3 =
3
The Principal Properties of Logarithm
Let M & N are arbitrary positive numbers, a > 0, a  1 , b > 0, b  1 and a, b are any real numbers, then:
(i) loga  M .N   loga M  loga N ;
In general loga  x1 x2 ... xn   loga x1  loga x2  ...  loga xn
M 
(ii) log a    log a M  log a N
N 
(iii) log a M N  N log a M
1
(iv) log a M  log a M

log a M
(v) log b M  (base changing theorem)
log a b

Note:
1. Remember am = b is called the exponential form and loga b = m is called the
logarithmic form.
2. log102 = 0.3010 ; log103 = 0.4771
ln 2 = 0.693 ; ln 10 = 2.303
3. loga1 = 0 ; loga a = 1
log 1 a  1 1
4. ; logb a 
a log a b

10
MATHEMATICAL TOOLS PHYSICS

Two Systems of Logarithm

Common Logarithm
The logarithms calculated to the base 10 are called common logarithms
Clearly 101  10  log10  1; 102  100  log100  2
101  0.1  log 0.1  1;
102  .001  log.001  2
Natural Logarithm
Logarithms to the base e (= 2.7182,) are called Natural logarithms. In all theoretical problems, we use natural
logarithms.
Example 5: Find log 3 243

Solution: Let log 3 243  x


 by def. 3x  243  (3)5
 x=5

9  35   15 
Example 6: Evaluate log    log    log  
 14   24   16 

9  35   15 
Solution: log    log    log  
 14   24   16 
 9 35   15 
 log     log  
 14 24   16 
 15   15 
 log    log    0
 16   16 
Characteristic and Mantissa

The integral part of the logarithms of a number, after expressing the decimal part as positive is called
characteristic and the positive decimal part is called the mantissa.
Rules to find the characteristic of Logarithm of any number
Rule 1: The characteristic of the logarithm of a number greater than unity is less by one than the number
of digits in its integral part and is positive.
e.g. The characteristics of log 314, log 87.263, log 2.78, log 3500 are respectively 2, 1, 0, 3.
Rule 2: The characteristics of the logarithms of a number less than one is negative and absolute value
one more than the number of zeros immediately after the decimal point.
e.g. The characteristics of log0.4, log0.3748, log0.000135, log . 08 are respectively –1, –1, – 4, –2.
To find the mantissa of the logarithm of number (say log 46.98)
(i) Removing the decimal point from the given number 46.98, we get 4698. The first two figures from
the left form 46, the third figure is 9 and the fourth is 8.
(ii) In the tables of logarithms (given in log book), we run the eye down the extreme left hand column
headed by a blank space until it arrives at 46.
(iii) In the horizontal row beginning with 46, and under the column headed by 9, we find the number
6712 at the intersection. We note in down.

11
PHYSICS MATHEMATICAL TOOLS

(iv) In continuation of this horizontal row and under the small column on the right (known as mean
differences column) headed by 8, we find the number 7 at the intersection.
(v) Adding 7 to 6712 we get 6719.
If any two numbers have the same sequence of digits, differing only in the position of the decimal point,
one must be equal to the other multiplied or divided by some integral power of 10. Hence their
logarithms must differ by an integer.
----------------------- ---------------------
x log10x x log10x
------------------------ --------------------
1 0 6 0.77815
2 0.30103 7 0.8451
3 0.47712 8 0.90309
4 0.60206 9 0.95424
5 0.69897 10 1
---------------------- ---------------------
Example 7: Find (a) log 42500 (b) log 0.0425 (c) log 0.000425
Solution:
(i) log 42500  log(4.25 104 )  log 4.25  log104  log 4.25  4
(ii) log.0425  log(4.25 102 )  log 4.25  log102  log 4.25  2
(iii) log.000425  log(4.25 104 )  log 4.25  log104  log 4.25  4.

Example 8: Evaluate log (48.26.)


Solution: The number is 48.26, containing two digits in integral part.
 the characteristic = 1
Now for mantissa, we remove the decimal point and take number = 4826
The number corresponding to the mantissa of 482 = 6830
Mean difference for 6 = 5
 the number corresponding to the mantissa of 4826 = 6835
 Mantissa =.6835
Hence, log (48.26) =1.683

ANTILOGARITHMS
The number whose logarithms is x is known as the antilogarithm of x and is generally written as antilog x.
For example, if log 3 = 0.4771, then 3 = antilog (.4771)
Thus, if log (y) = x, then y = antilog (x)
Antilogarithms (given in the log book) and the method can be illustrated with the help of the following
solved examples.
------------------------ -----------------------
x Antilog x x Antilog x
------------------------ -----------------------
0.10 1.259 0.60 3.981
0.20 1.585 0.70 5.102
0.30 1.995 0.80 6.310
0.40 2.512 0.90 7.943
0.50 3.162 0.99 9.772
------------------------ ----------------------

12
MATHEMATICAL TOOLS PHYSICS

Example 9: Find the number whose logarithms is 2.5423.


Solution: Clearly characteristic of the logarithms = 2 and
 this is one less than the number of digits in the integral part 2 from the given number 2.5423, we
get 0.5423. From the tables of antilogarithms, the number corresponding to 0.542 = 3483
Mean difference for 3 = 2
 the number corresponding to 0.5423=3483 + 2 = 3485
Hence the required number = 348.5
Example 10: Find the cube root of 48, correct to two places of decimal.
Solution: Let n  (48)1/3 .
Taking log on both sides, we get,
1
log n  log(48)1/3  log 48
3
1
 (1.6812)  0.5604
3
Hence n = antilog (0.5604) = 3.634 = 3.63 correct to two decimal places.

FUNCTION
If we say “y is a function of x”, then in writing it will be represented as
y = f (x) (“y equals f of x”)
In this notation, the symbol f represents the function. The letter x, called the independent variable, represents
an input value from the domain of f and y the dependent variable, represents the corresponding output value f
(x) in the range of f.
x f(x)
f
Input Ouput
(Domain) (Range)
Think of a function f as a kind of machine that produces an output value f (x) in its range whenever we feed it
an input value x from its domain (figure).
We usually define functions in one of the two ways :
(i) By giving a formula such as y = x2 that uses a dependent variable y to denote the value of the function.
(ii) By giving a formula such as f (x) = x2 that defines a function symbols f to name the function.
4
Example 11: The volume V of a ball (solid sphere) of radius r is given by the function V (r )   r 3 . What is
3
the volume of the ball if radius is 3m?
4
V  3    3  36 m3 .
3
Solution:
3
Example 12: Suppose that the function F is defined for all real numbers r by the formula.
F(r) = 2(r – 1) + 3.
Evaluate F at the input values 0, 2, x + 2, and F(2).
Solution: In each case we substitute the given input value for r into the formula for F:
F(0) = 2(0 – 1) + 3 = – 2 + 3 = 1
F(2) = 2(2 – 1) + 3 = 2 + 3 = 5
F(x + 2) = 2(x +2 – 1) + 3 = 2x + 5
F(F(2)) = F(5) = 2(5 – 1) + 3 = 11

13
PHYSICS MATHEMATICAL TOOLS

DIFFERENTIATION OR DERIVATIVE
Finite Difference

Difference in two values of y is written as y as given in below table.


y2 100 100 100
y1 50 99 99.5
y = y2 – y1 50 1 0.5

Infinitely Small Difference


The infinitely small difference by words means very-very small difference. And this difference is
represented by ‘d’ notation instead of ‘’.
For example, infinitely small difference in the values of y is written as ‘dy’
if y2 = 100 and y1 = 99.99999999........
then dy = 0.000000...................00001
Similarly, if ‘v’ represents velocity then ‘dv’ represents infinitely small difference in velocity. Physically
this will represent the difference of two velocities of two instants having very small duration in between.

Definition of Differentiation (Derivative)


Differentiation of y with respect to x is defined as rate of change of y w.r.t. x when change in x is
dy
infinitely small. It is denoted by symbol f   x  
dx
y f ( x  x)  f ( x)
The average rate of change of y with respect to x  
x x
y QR
Geometrically, = = Slope of the line PQ
x PR

Q Q
y+y y+y

y Q

Q y
P 
y R P Q
x y  R
x
x x+x
x x+x
y
In triangle QPR tan  =
x Figure - 2
If x will be infinitely small and secant PQ to the given curve will become a tangent at point P.
Therefore, we can say that of average rate of change of y with respect to x is equal to slope of the line
joining P & Q. If Q lies very close to P such that x  0 ,then the slope of the tangent at point P (x, y)
dy
or tan  
dx
RULES:
d
RULE-1: Derivative of every constant function is zero. If C is constant, then (C )  0
dx

14
MATHEMATICAL TOOLS PHYSICS

e.g.
d
dx
(8)  0 ,
d  1
    0,
dx  2 
d
dx
 3  0
d n
RULE-2: If n is a real number, then x  nx n1 .
dx
d  1  d 1
e.g. (a)  
dx  x  dx
 x    1 x 2   2
1
x
d  4
 3   4  x   4  3 x   4
d 3 4 12
(b)
dx  x  dx x
d 1/2 1 1
(c) ( x )  x 1/2 
dx 2 2 x
d 1/5 1 –4/5
(d) (x )  x
dx 5
d du
RULE-3: If u is a differentiable function of x, and c is a constant, then (cu)  c
dx dx
d
e.g. (3x 2 )  3  2 x   6 x
dx
RULE-4: The derivative of the sum of two differentiable functions is the sum of their derivatives.
d du du du
(u1  u2  .....  un )  1  2  .......  n
dx dx dx dx
Example 13: Find derivative of y = x4 + 12x w.r.t. x.
dy d 4 d
Solution:  ( x )  (12 x)
dx dx dx
= 4x3 + 12

4 2
Find derivative of y  x  x  5x  1 w.r.t. x.
3
Example 14:
3
dy d 3 d  4 2  d d
Solution:  x   x   (5 x)  (1)
dx dx dx  3  dx dx
4 8
 3x2  .2 x  5  0  3x2  x  5
3 3
RULE-5: If u and v are differentiable at x, then so is their product uv (product rule), and
d dv du
 uv   u  v
dx dx dx

Example 15:  
Find the derivatives of y  x 2  1 x3  3 w.r.t x. 
Solution: From the product Rule with u  x 2  1 and v  x3  3 , we find
 x 2  1 x3  3   x 2  1 3x 2    x3  3  2 x 
d
we find,
dx  
 3 x 4  3 x 2  2 x 4  6 x  5 x 4  3x 2  6 x

15
PHYSICS MATHEMATICAL TOOLS

u
RULE-6: If u and v are differentiable at x, and v(x)  0, then the quotient is differentiable at x, and
v
du dv
uv
d u dx dx
 
dx  v  v 2

t 2 1
Example 16: Find the derivative of y  w.r.t. ‘t’.
t2 1
Solution: Applying the Quotient Rule with u = t2 – 1 and v = t2 + 1
d  u  v(du / dt )  v(du / dt )
 
dt  v  v2
dy  t  1 .2t   t  1 .2t
2 2

 
 t 2  1
2
dt

2t 3  2t  2t 3  2t 4t
  2
(t  1)
2 2
(t  1) 2

d d
RULE-7: (a) (sin x)  cos x (b) (cos x)   sin x
dx dx
d d
(c)  tan x   sec2 x (d)  sec x   sec x tan x
dx dx
d d
(e)  cot x   cosec2 x (f)  cos ec x    cos ec x cot x
dx dx
dy
Example 17: Find if (a) y  x 2  sin x (b) y  x 2 sin x
dx
Solution:
(a) y  x 2  sin x (Difference Rule)
dy d
 2 x  (sin x)  2 x  cos x
dx dx
(b) y  x 2 sin x
dy d
 x2  sin x   2 x sin x  x2 cos x  2x sin x (Product Rule)
dx dx
dy
Example 18: Find if y  tan x .
dx
d d
cos x  sin x  – sin x  cos x 
 tan x   
d d sin x  dx dx
Solution: 
dx dx  cos x  cos x2

cos x cos x – sin x  – sin x 



cos 2 x
cos 2 x  sin 2 x 1
 2
 2
 sec2 x
cos x cos x

16
MATHEMATICAL TOOLS PHYSICS

dy 2
Example 19: Find if (a) y = (3x + cot x) (b) y 
dx sin x
Solution:
d d
(a)  3x  cot x   3   cot x   3  cosec2 x
dx dx
d  2  d d
(b)    2 cosec x   2  cosec x 
dx  sin x  dx dx
 2  cosec x cot x   2cosec x cot x

RULE-8: (a)
d
dx
 loge x  
1
x
(b)
dx
 e  =ex
d x

Example 20: y  ex .loge  x 


dy  x d (log e x)  de x
Solution:  e   log e x
dx  dx  dx
x
dy e
  e x .log e  x  
dx x

Example 21: Find the derivative of y  x2  1

 
1
Solution:
dy d

dx dx
x2  1 
d 2
dx
 x  1 2
1
  x  1 2  x  1
dy 1 2 1 d 2

dx 2 dx
dy 1 x
  . 2 x  
dx 2 x  1
2
x 1
2

d n du
RULE-9: Power Chain Rule: u  nu n-1
dx dx
sin 5 x  5  sin 4 x   sin x   5sin 4 x cos x
d d
e.g. (a)
dx dx
d 4 d
 2 x  1  3  2 x  1  2 x  1
3
(b)
dx dx
 3  2 x  1  2   6  2 x  1
4 4

d
 5 x3  x 4   7  5 x3  x 4   5 x3  x 4 
7 6 d
(c)
dx dx
 7  5 x3  x 4   5.3 x 2  4 x 3   7  5 x3  x 4  15 x 2  4 x3 
6 6

 x    x  
sin  x o   cos  x o 
d d
RULE-10: sin   cos  
dx dx  180  180  180  180

DOUBLE DIFFERENTIATION
d  dy  d 2 y
   f   x 
dx  dx  dx 2

17
PHYSICS MATHEMATICAL TOOLS

Example 22: If f (x) = x cos x, find and interpret f   x  .


Solution: Using the Product Rule, we have
d d
f  x  x  cos x   cos x  x   x sin x  cos x
dx dx
To find f  (x) we differentiate f  (x):
d
f   x     x sin x  cos x 
dx
d d d
  x  sin x   sin x   x    cos x 
dx dx dx
  x cos x  sin x  sin x    x cos x  2sin x

Note:
dx
1. v  this means velocity ‘v’ is rate of change of displacement ‘x’ with respect to time ‘t’.
dt
d v d 2x
2. a   = this means acceleration ‘a’ is rate of change of velocity ‘v’ with respect
dt dt 2
to time ‘t’.

Example 23: The position of a particle is given by the equation, s  f  t   t 3  6t 2  9t , where t is measured in
seconds and s in meters. (a) Find the acceleration at time t. (b) What is the acceleration after 4 s?
Solution:
(a) The velocity function is the derivative of the position function:
s  f t   t 3  6t 2  9t
ds
v t    3t 2  12t  9
dt
The acceleration is the derivative of the velocity function:
d 2 s dv
a t     6t  12
dt 2 dt
acceleration at t = 4s will be given as
a(4) = 6(4) – 12 = 12 m/s2

SLOPE (DERIVATIVE) OF A CURVE AT MINIMUM AND MAXIMUM VALUES


y
Suppose a quantity y depends on another quantity x in a manner shown in the figure. It
becomes maximum at x1 and minimum at x2. At these points the tangent to the curve is
parallel to the x-axis and hence its slope is (tan  = 0. Thus, at a maximum or a minimum,
dy x
slope = 0 x1 x2
dx

18
MATHEMATICAL TOOLS PHYSICS

Maxima
Just before the maximum the slope is positive, at the maximum it is zero and
dy
just after the maximum it is negative. Thus, decreases at a maximum
dx
dy
and hence the rate of change of is negative at a maximum i.e.
dx
d  dy 
   0 at maximum.
dx  dx 
d  dy  d2y
The quantity   is the rate of change of the slope. It is written as .
dx  dx  dx 2
dy d2y
Conditions for maxima are: (a) 0 (b) 0
dx dx 2
Minima
Similarly, at a minimum the slope changes from negative to positive. The
d  dy 
slope increases at such a point and hence  0.
dx  dx 
dy d2y
Conditions for minima are: (a) 0 (b) 0
dx dx 2
Note: If then it is known from the physical situation whether the quantity is a maximum or a
d2y
minimum, The test on may be omitted.
dx 2

Example 24: Particle’s position as a function of time is given as x  5t 2  9t  3 . Find out the minimum value
of position co-ordinate?
Solution: x  5t 2  9t  3
dx
 10t  9  0
dt
9
 t   0.9
10
Check, whether maxima or minima exists.
d 2x
 10  0
dt 2
 there exists a minima at t = 0.9
x  5  0.9   9  0.9   3
2

 4.05  8.1 3  1.05

19
PHYSICS MATHEMATICAL TOOLS

INTEGRATION
As inverse operation of addition is subtraction, inverse operation of multiplication is division and inverse
function of square is square root, Integration is the inverse process of Differentiation. If we are given a
function f(x) and we obtain another function F(x) such that derivative of F(x) w.r.t. x is f(x), then F(x) is
defined as the integral or primitive function of f(x).
d
Symbolically, F  x   f ( x)
dx
Then,  f ( x) dx  F ( x)
d 2
e.g. If ( x )  2 x, then  2 x dx  x 2
dx
Indefinite Integrals

 f ( x)dx  F ( x)  C. …(i)

The symbol  is an integral sign. The constant C is the constant of integration or arbitrary constant,
Equation (1) is read, “The indefinite integral of f with respect to x is F(x) + C.”

Integral Formulas

x n 1
 x dx   C , n  –1, n rational Special Case:  dx  1dx  x  C
n
1.
n 1
cos kx sin kx
2.  sin kx dx  
k
C 3.  cos kx dx 
k
C

 sec x dx  tan x  C  cosec x dx   cot x  C


2 2
4. 5.

6.  sec x tan x dx  sec x  C 7.  cosec x cot x dx  cosec x  C

x6
 x dx  6  C
5
e.g. (a)

1
(b)  dx   x 1/2 dx  2 x1/2  C  2 x  C
x
 cos 2 x
(c)  sin 2 x dx  C
2
x x sin( x / 2) x
(d)  cos dx   cos dx   C  2sin  C
2 2 1/ 2 2
Rule No. (1)  k f ( x) dx  k  f ( x) dx
e.g. Rewriting the constant of integration

 5sec x tan xdx  5 sec  x  tan  x  dx


 5 sec x  C   5sec x  5C  5sec x  C


Rule No. (2) [ f ( x)  g ( x)dx]   f ( x)dx   g ( x)dx

20
MATHEMATICAL TOOLS PHYSICS

x  2 x  5 dx
2
Example 25: Evaluate:
x3
 ( x  2 x  5) dx   x dx   2 xdx   5dx   x 2  5x  C
2 2
Solution:
3

 sin  x  dx  cos  x dx
2 2
Example 26: Evaluate (a) (b)
Solution:
1  cos 2 x 1  cos 2 x
 sin x  dx   dx (As, sin 2 x 
2
(a) )
2 2
1 1 1
  1  cos 2 x  dx   dx   cos 2 xdx
2 2 2
x 1  sin 2 x  x sin 2 x
   C   C
2 2 2  2 4
1  cos 2 x 1  cos 2 x
 cos x dx   2 dx (As, cos x  2 )
2 2
(b)

x sin 2 x
  C
2 4
As in part (a), but with a sign change
Rule No. (3) Rule of substitution

 ( x  2) dx
5
Example 27: Evaluate:

 ( x  2) dx   u du
5 5
Solution:
Put u = x + 2, du = dx
u6
 ( x  2) dx   u du  C
5 5

6
Replace u by x + 2
 x  2
6

 C
6
Example 28: Evaluate:  1  y 2  2 y  dy
Solution: Let u  1  y 2 , du  2 ydy

 1  y 2  2 y  dy   u1/2 du
u (1/ 2) 1 2
  u 3/2  C Simpler form
(1 / 2)  1 3
Replace u by 1 + y2.
2
 (1  y 2 )3/2  C
3

x  2 x  3  x  1 dx
2 2
Example 29: Evaluate:
Solution: Let u  x2  2 x  3
 du  2xdx  2dx  2  x  1 dx

21
PHYSICS MATHEMATICAL TOOLS

1
i.e.   du   x  1 dx
2
1 2
 ( x  2x  3)  x  1 dx  2
2 2
Now, u du

1 u3 1
   C  u3  C
2 3 6


1 2
x  2 x  3  C
3

6
Definite Integration or Integration with Limit
The function is the integrand.
Upper limit of integration
x is the variable of integration
b

Integral sing
f ( x) dx
Lower limit of integration
a   when find the value of the integral
Integral of f from a to b you have evaluated the integral.
b

 f ( x) dx   g ( x)  g (b)  g (a)


b
a
a
4
Example 30: Evaluate:  1
3dx

3dx  3 dx  3 x1
4 4

4
Solution:
1 1

 3  4   1   3 5  15
 /2
Example 31: Evaluate:  0
sin x dx
 /2
sin x dx    cos x  0
 /2
Solution:  0

 
  cos    cos  0   0  1  1
2
Example 32: Does the curve y  x 4  2 x 2  2 have any horizontal tangents? If so, where?
Solution: The horizontal tangents, if any, occur where the slope dy/dx
is zero. To find these points. We

1. Calculate dy/dx:
dy d 4

dx dx

x  2 x 2  2  4 x3  4 x 
dy
2. Solve the equation:  0 for x : 4 x3  4 x  0
dx
4 x x2 1  0  
x  0, 1, 1
The curve y  x  2 x  2 has horizontal tangents at x = 0, 1 and –1.
4 2

Example 33: A hot air balloon rising straight from a level field is tracked by a range finder 5000 ft. from the

lift-off point. At the moment the range finder’s elevation angle is , the angle is increasing at
4
the rate of 0.14 rad/min. How fast is the balloon rising at the moment?
Solution: We answer the question in six steps.

22
MATHEMATICAL TOOLS PHYSICS

Step-1: Draw a picture and name the variables and constants (Figure). The variables in the picture are 
= the angle the range finder makes with the ground (radians)
y = the height of the balloon (feet).
We let t represent time and assume  and y to be differentiable functions of t.
The one constant in the picture is the distance from the range finder to the lift-off point (500 ft.).
There is no need to give it a special symbol s.
Step-2: Write down the additional numerical information.
d 
 0.14 rad/min when  
dt 4
dy 
Step-3: Write down what we are asked to find. We want when   .
dt 4
Step-4: Write an equation that relates the variables y and .
y
 tan  or y  500 tan 
500
dy
Step-5: Differentiate with respect to t using the Chain Rule. The result tells how (which we want) is
dt
d
related to (which we know).
dt
dy d
 500sec2 
dt dt
 d dy
Step-6: Evaluate with   and  0.14 to find .
4 dt dt

dy
 
 500 2  0.14   1000  0.14   140
 
2
 sec  2 
dt  4 
At the moment in question, the balloon is rising at the rate of 140 ft./min.
Example 34: A police cruiser, approaching a right-angled intersection from the north, is chasing a speeding
car that has turned the corner and is now moving straight east. When the Cruiser is 0.6 ml north
of the intersection and the car is 0.8 ml to the east, the police determine with radar that the
distance between them and the car is increasing at 20 mph. If the cruiser is moving at 60 mph at
the instant of measurement, what is the speed of the car?
Solution: We carry out the steps of the basic strategy.

23
PHYSICS MATHEMATICAL TOOLS

Step-1: Picture and variables. We picture the car and cruiser in the coordinate plane, using the positive x-
axis as the eastbound highway and the positive y-axis as the northbound highway (figure). We let
t represent time and set
x = position of car at time t y = position of cruiser at time t
s = distance between car and cruiser at time t
We assume x, y and s to be differentiable functions of t.
dy ds
x = 0.8 mi, y = 0.6 mi,  60 mph ,  20 mph
dt dt
dy
( is negative because y is decreasing)
dt
dx
Step-2: To find:
dt
Step-3: How the variables are related: s2  x2  y2 Pythagorean theorem

(The equation s  x 2  y 2 would also work.)


ds dx dy
Step-4: Differentiate with respect to t. 2s  2x  2 y Chain Rule
dt dt dt
ds 1  dx dy  1  dx dy 
 x  y  x  y 
dt s  dt dt  x 2  y 2  dt dt 
dy ds dx
Step-5: Evaluate, with x = 0.8, y = 0.6,  60 ,  20 and solve for .
dt dt dt
1  dx 
20   0.8   0.6  60  
 0.8   0.6   dt 
2 2

1
dx dx 20  36
 20  0.8  36    70
dt dt 0.8
At the moment in question, the car’s speed is 70 mph.
Example 35: Using an area to evaluate a definite integral
b
 a
xdx 0ab
Solution: We sketch the region under the curve y  x, a  x  b (figure) and see that it is a trapezoid with
height (b – a) and bases a and b. The value of the integral is the area of this trapezoid.

a  b b2 a 2
xdx   b  a  .
b
 a 2

2

2
x2
Notice that is an antiderivative of x, further evidence of a connection between antiderivatives
2
and summation.

24
MATHEMATICAL TOOLS PHYSICS

CLASS ASSIGNMENT
1. Evaluate (26)1/2.
Z2
2. Solve y  , given h << Z using binomial theorem.
 Z  h
2

3. Find the value:



(a) sin 225o  
(b) tan 315o  
(c) sin 1485o   
(d) sin 50o
4. The displacement covered by a particle depends upon time according to the equation s  3t  2 where s
is in meter and t is in second. Calculate the velocity of the particle using graphical method.
 4936  180
1/3 1/5

5. Find the value of .


 369
1/7

6. Differentiate the following with respect to x.


(a) y  3 x  4 x  5
2

(b) y  x 2 tan x
(c) y  3x2   loge x  sin x 
(d) y  4 x 2 e x
(e) y  3 x  4 x cos x
2

x
(f) y 
sin x
x2
(g) y 
 sin x   e x 
x2
(h) y 
loge x
 
7
(i) y  4 x 2  3 x

(j) y   3x  2  sin 8 x 
3

(k) y  x loge  sin x 

(l) y  x2e x
2

(m) y  x tan  x 2 
(n) y  cos3 x

  4x  cos x  dx
2
7. Integrate
8. A force F = (2y + 3)N is acting along y-axis. Calculate the work done in moving a body from y = 1m to
y = 3m.
 
9. The current flowing through a wire is given by I  4t 2  5 A . Calculate the amount of charge flowing
through it during 1 to 4s.

25
PHYSICS MATHEMATICAL TOOLS

HOME ASSIGNMENT

I. Questions based on Quadratic Equation

1. Solve 6 x2 – 13 x + 6 = 0
2. Solve the equation for x: 4x2–4ax + (a2–b2) = 0
II. Questions based on Binomial Theorem
3. Write down constraints (conditions/restriction) to apply Binomial theorem in following form.
(1  x) n  1  n x
4. Solve (1  x ) 4 using Binomial theorem upto two terms if 𝑥 ≪ 1.
5. Find number of terms in following expressions
15
 3x 
(b) 1  9x 
2/3
(a) 1  
 2 
6. Solve 1  2n  up to three terms if 𝑛 ≪ 2
2 1

7. Mention first step that should be followed in solving the following expression binomially: (a + b)n
Given: n is either a negative number or is a fraction and b >> a.
 624 
1/4
8. Solve using binomial theorem upto (a) two terms & (b) three terms. Then compare the
difference in two cases.
III. Questions based on Algebra
9. Complete the following algebraic expressions:
(i) (a + b)2 = (ii) (a – b)2 =
(iii) (a + b + c)2 = (iv) (a + b) (a – b) =
(v) (a + b) =3 (vi) (a – b)3 =
(vii) (a + b)2 – (a – b)2 = (viii) (a + b)2 + (a – b)2 =
(ix) a3– b3 = (x) a3  b3 
10. Find the nth term and sum of series 7 + 10 + 13 + 16 + 19 + 22 + 25 using standard relations of AP
IV. Questions based on Trigonometry
11. Find the values of
(i) cos (–600) (ii) tan 2100 (iii) sin3000
(iv) cos 1200 (v) sin(1485o )
12. If A = 600 then find the value of sin 2A using formula for sin2x.
 13
13. Convert rad and rad into degree.
9 6
14. Convert 570 and 6400 into radian.
15. Solve sin (750) using addition formula.
16. Solve sin (750) sin (150) using C-D formula.
17. Write down condition(s) for applying following approximations.
(a) sin   (b) tan   (c) cos  1
0 0
18. Write down approximate value of sin 50 and cos 50 , without referring trigonometric tables.
19. Find the angle (in degree) subtended by an arc of length 20 cm at the center of circle of radius 3m.

26
MATHEMATICAL TOOLS PHYSICS

20. Mention trigonometric ratios that are positive in respective quadrants in the following diagram.
Second quadrant First quadrant

Third quadrant Fourth quadrant

V. Questions based on Graphs


21. What must be exponents of x and y in the equation of straight line?
22. Write down equation of the following line (shown in diagram) if angle made by line with positive
direction of x-axis is 45o
Y


X
O

23. Write down general equation of the following curve (shown in diagram)
Y

X
O

24. Draw a graph (curve) represented by y = 3x –4x2 Also, mention the nature of graph.
25. Draw graphs for following lines.
(a) 3x + y + 1 = 0 (b) y – 15x +23/4 = 0
Mention intercept and slope in each case.
26. A function f (x) is defined as f (x) = 5x2–3x + 5. Find
(a) f (2) (b) f ( f (1))
27. Draw a graph for: xy = constant
28. Draw a graph for: y = ax + bx2
29. Depict the nature of graph drawn between V (along y-axis) and I (along x-axis). Given. What does slope
of the represent?
VI. Questions based on Logarithm & Antilogarithm
30. Express expression (28 = 256) in terms of log.
31. Given: log of 25.67 is 1.4094. What is antilog of 1.4094?
 2  2  2
32. Prove that: log  a   log  b   log  c   0
 bc   ca   ab 
33. Solve log( x  1)  log( x  1)  log 3
34. Evaluate log10 0.0012.
35. Find the value of: (25.36)  0.4569
2

847.5
(11.1)2/3  4.9
36. Find the value of :
7.68
37. Write down approximate value of log (35) without referring log table.
38. Find antilog of (3.1234)
39. If v  u (ln m0  ln m) , then use standard relations of logarithm to simply this relation and then express in
common log.

27
PHYSICS MATHEMATICAL TOOLS

VII. Questions based on Differentiation / Derivative


40. Fill the following blanks to review your concepts:
dy
(a) If is positive, then it means y is ______ with increasing of x.
dx
(b) The differential coefficient or derivative of variable y with respect to variable x is defined as the
instantaneous ____________________ of y w.r.t x. It is denoted by ____________.
d
(c) (____)  0
dx
dv
______  u
dy d  u  dx
(d)   
dx dx  v  v2
(e) Slope of a curve at point (1,4) is equal to 1.2. Then dy/dx at that point is _______.
41. Differentiate the following w.r.t x
(i) x3 (ii) x (iii) ax 2  bx  c
(iv) 2 x3  e x (v) 6 loge x  x  7
x2
42. Differentiate w.r.t. x
log x
1 2
43. The height reached in time t by a particle thrown upward with a speed u is given by h  ut  gt . Find
2
dh
the time taken in reaching the maximum height. (Hint for h to be maximum,  0)
dt
1
44. The radius of an air bubble is increasing at the rate of cm / s. Determine the rate of increase in its
2
volume when the radius is 1 cm. (Hint use chain rule)
45. Find the angle of tangent drawn to the curve y  3x 2  7 x  5 at the point (1, 1) with the x- axis. (Hint
dy
Find at (1, 1))
dx
46. Write symbol for double derivative. Give one practical example where we use double derivative in Physics.
47. Ideal gas equation is written as PV = nRT. If n, R and T are constants in this equation, then find dP/dV.
48. If position (x) of a particle as a function of time (t) is given as x = 5t3–12t2 + 1, then find x, v and a at t =
dx dv
1s. Here x is in meter and t is in second. (Hint v  and a  )
dt dt
49. If x  a 1  sin   & y  a 1  cos  , then find dy/dx.

VIII. Questions based on Integration


50. Integrate the following w.r.t. x
(i) x1/2 (ii) sec2 x
6
51. Evaluate  (2 x 2  3x  5)dx
0
52. Integrate the following
2 1  /2 Kq1q2  /4
 
r2
(i)  dx (ii) cos x dx (iii)  dr (iv) tan 2 x dx
0
x 0 r1 r2 0

2
1
53. Evaluate  dx
0 
1  3x 

28
MATHEMATICAL TOOLS PHYSICS

d
40
54. Evaluate   
50 0
’ where 0 = 30oC, a constant temperature of surrounding.


55. Evaluate sin x (Be careful)
0
56. Calculate area under curve y = x2 from x = 1 to x = 3.
v t
57. Evaluate  dv   a dt . Here a represent constant acceleration.
u 0
58. If derivative of a function p is q, then what is the integral of q.?
59. Sign  is used for summation of discrete values, while  is used for ________________ of
______________ function.
IX. Questions based on Derivative and Integration
1
60. If 𝑦 = 𝑥 4 then what is the value of dy/dx?
4
(A) 𝑥 3 (B) 4x3
4 4
(C) – (D)
𝑥5 𝑥5
𝑑𝑦
61. If y = sin x & x = 3t, then what is the value of 𝑑𝑡
?
(A) 3 cosec (x) (B) cos x
(C) 3 cos (3t) (D) – cos x
𝑑𝑦
62. If y = sin(x) + ln(x2) + e2x then what is the value of 𝑑𝑥
?
2 2
(A) cos x + + e2x (B) cos x + + 2e2x
𝑥 𝑥
2 2
(C) – cos x + 𝑥 2 + e2x (D) –cos x – 𝑥 2 + 2e2x
𝑑𝑦
63. If y = ekt then what is the value of 𝑑𝑡
?
ekt
(A) e kt (B)
k
(C) tekt (D) ke kt
2
at
64. If s  ut  ,where s is displacement, u is the initial velocity (constant), v = final velocity, a =
2
acceleration (constant) then t is time taken then differentiation of 'S' w.r.t. 't' will be
at
(A) u  (B) u + at
2
ut 2 at 3
(C) u + 2at  (D)
2 6
65. In the above question what is the differentiation of the result w.r.t. 't' ?
(A) a (B) u + a
(C) u (D) none of these
ln x dy
66. If y = then what is the value of ?
x dx
1  ln x 1  ln x 1  ln x ln x  1
(A) (B) (C) (D)
x x2 x2 x2
67. Differentiation of sin(x2) w.r.t. x is
(A) cos (x2) (B) 2x cos(x2) (C) x2 cos(x2) (D) – cos (2x)

29
PHYSICS MATHEMATICAL TOOLS

ANSWER KEY
SUBJECTIVE PROBLEMS
1. x = 3/2; 2/3 17. All approximations can be applied iff is very small
( a  b ) ( a  b) 
2. , (  < 100 or  < ). Remember must be taken
2 2 18
3. |x|<<1 into radian.
4. 1  4x 18. (a) 0.76 (b) 0.64
5. (a) 16 (b)  19. 3.82 0

6. 1  4n  12n 20. First quadrant: All trigonometric ratios are


2

7. Take b common from the bracket to make positive.


n Second quadrant: sin  and cosec  are positive.
 a
expression b 1  
n
Third quadrant: tan  and cot  are positive.
 b Fourth quadrant: cos  and sec  are positive.
8. (a) 4.998 (b) 4.99799 21. Both must be 1.
2 2
9. (i) a + b + 2ab 22. y=x
(ii) a2 + b2 – 2ab 23. y2 = – kx
(iii) a2 + b2 + c2 + 2ab + 2bc + 2ca 24.
(iv) a2 – b2 Y

(v) a3 + b3 + 3ab(a + b)
(vi) a3 – b3 – 3ab(a – b) X
(vii) 4ab O , Asymmetric parabola
(viii) 2(a2 + b2) 25. (a) c = –1, m = –3 (b) c = –23/4, m = 15
(ix) (a  b) (a 2  b 2  ab) 26. (a) 19 (b) 229
27.
(x) (a  b) (a 2  b 2  ab) Y

10. (a) Tn = 7 + (n – 1)3 = 3n + 4


n 7
(b) Sn   2a0  (n  1)d   [14  (7  1)3]  112 X
2 2 O

28.
1  3 Y
11. (i) 1/2 (ii) (iii)
3 2
1 1 O
X

(iv) (v) 
2 2 , upward Parabola.
3 29. (a) Straight line (b) slope gives value of R
12.
2 30. log2 256 = 8
13. (a) 200 (b) 3900 31. 25.67
14. (a) 1o (b) 11.22807 32. Using formula log m  log n  log mn
1 1 3  a 2b 2 c 2 
15.     0.966 We get log   log1  0
2  2 2    ab  bc  ac  
 
16. 0.25 33. x  2

30
MATHEMATICAL TOOLS PHYSICS

34. 3.0792  2.9208 x  2 log x  1


42.
 log x 
2
35. Let n  (25.36)  0.4569 Taking log on both
2

847.5 u
43. t 
 (25.36)2  0.4569  g
sides, we get log n  log  
 847.5  dV 1
44.  4  (1)2   2 cm3 / sec
 log (25.36) 2  log (0.4569)  log (847.5) dt 2
dy dy
 2log (25.36)  log (0.4569)  log(847.5)  6 x  7, at x  1  1  tan     135 .
o
45.
dx dx
 2(1.1041)  (1.6599)  (2.9282) 2
d y d2x
 2.8082  1.6999  2.9282 46. , acceleration =
dx 2 dt 2
 2.4681  2.9282 dP  P
 1  (3.4681  2.9282) 47. PdV  VdP  0  
dv V
 1  0.5399  1.539 48. –6 m ; –9 m/s ; 6 m/s2
Hence, n = antilog (. 5399) = 0.3466 dy dx dy
36. 3.174 49.  a sin  ,  a cos   tan 
d d dx
37. Hint: You know value of log 3 and log 4. Use
2
these values and standard formulae of loft to get 50. (i) ( x3/2 ) (ii) tan x (iii) tan x  sec x
approximate value of log (35).log (35) = 1.54
3
51. 228
(approximately)

38. 1328 52. (i) 2 2 ; (ii) 1 ; (iii) kq1q2  1  1  ; (iv) 1 
 r1 r2  4
m0
39. v  2.303 u log10 1
m 53. ln 7
dy 3
40. (a) Increasing (b) rate of change, 54. –0.6931
dx 

(c) constant (d) v


du
(e) 1.2 
55. Statement is wrong. It should be sin x dx
0
dx
56. 26/3 unit
2 1
41. (i) 3x (ii) (iii) 2ax  b s 57. v=u+at
2 x
58. p
(iv)  6 x  e
2 x
(v)  6  1 s 59. summation, continuous
x 2 x

OBJECTIVE PROBLEMS
Questions Answers Questions Answers
60 C 64 B
61 C 65 A
62 B 66 C
63 D 67 B

31
PHYSICS MATHEMATICAL TOOLS

OBJECTIVE QUESTIONS BANK

1. Convert 18 degree into radian.


π π
(A) rad (B) rad
10 180
π 18
(C) rad (D) rad
18 π
2. Value of sin (37°) cos (53°) is
9 12
(A) (B)
25 25
16 3
(C) (D)
25 5
1
3. If sin  = , then cos  will be
3
8 4
(A) (B)
9 3
2 2 3
(C) (D)
3 4
4. sin (90º + ) is
(A) sin  (B) cos 
(C) – cos  (D) – sin 
5. cos (30°) is equal to
3 1
(A) (B)
2 2
1
(C) (D) None
2
6. Which of the following has value zero ?
(A) Sin 0° (B) tan 90°
(C) cos 0° (D) cot 0°
7. The differential co-efficient of sin 5 x is :
(A) 5 cos x (B) 5 cos 5 x
cos 5 x
(C) cos 5 x (D)
5
1
8. If square of x varies as cube of y and x = 3 when y = 4, the value of y at x = will be :
3
3 4
(A) (B)
4 3
4
(C) 4 (D)
3 3
9. If y = 2 sin (wt +) where w and  constants then dy/dt will be -
(A) 2w cos(wt + ) (B) – 2w cos(wt + )
(C) 2 sin(wt + ) (D) 2w sin(wt + )

32
MATHEMATICAL TOOLS PHYSICS

nx dy
10. If y = then will be :
x dx
1 n x 1 n x
(A) (B)
x x2
1 n x n x 1
(C) (D)
x2 x2

x
2
11. is equal to :
3
(A) x +C (B) 2x
3
3
(C) 2x (D) Meaningless
3
1


12. If y = x2, then ydx will be :
0
3
1
(A) x +C (B)
3 3
2
(C) (D) 0
3
13. If velocity of particle is given by v = 2t4 then its acceleration (dv/dt) at any time t will be given by:
(A) 8t3 (B) 8t
(C) –8t 3
(D) t2
14. Differentiation of 2x2 + 3x w.r.t. x is :
(A) 4x + 3 (B) 4x
(C) 3 (D) 4x + 1
15. If  = sec (3) then dα will be

(A) 3 sec (3) tan (3) (B) 32 sin (3)
(C) sec (3) tan (3) (D) 3 sec2 (3)
dy
16. If y = ex . cot x then will be
dx
(A) excot x – cosec2x (B) ex cosec2x
(C) ex[cot x – cosec2x] (D) excot x
dy
17. If y = x2 sin x , then will be
dx
(A) x2 cos x + 2x sin x (B) 2x sin x
(C) x2 cos x (D) 2 x cos x
dy
18. If y = tan x . cos2x then will be
dx
(A) 1 + 2sin2x (B) 1 – 2sin2x
(C) 1 (D) 2 sin2x
19. Double differentiation of displacement w.r.t. time is :
(A) acceleration (B) velocity
(C) force (D) none

33
PHYSICS MATHEMATICAL TOOLS

Comprehension
If a = (3t2 + 2t + 1)m/s2 is the expression according to which the acceleration of a particle varies. Then
20. The expression for instantaneous velocity at any time 't' will be (if the particle was initially at rest)
(A) t3 + 2t + 1 (B) t3 + t + 1
(C) t3 + t2 + t (D) t3 + t2 + t + C
21. The change in velocity after 3 seconds of its start is
(A) 30 m/s (B) 39 m/s
(C) 3 m/s (D) 20 m/s
22. Find displacement of the particle after 2 seconds of start
(A) 26 m (B) 26/3 m
(C) 30/7 m (D) 26/7 m
Comprehension
If charge flown through a wire is given by q = 3 sin (3t) then
23. Find out the amount of charge flown through the wire at t =  π  seconds.
6
(A) 3 C (B) 6 C
(C) 1 C (D) Zero
π
24. Find out the current flown through the wire at t = second.
9
(A) 4.5 A (B) 4.5 3 A
(C) 3 /2 A (D) 9 A
π π
25. Find out the area under i – t curve from t = to t = seconds:
9 6
2  3  2  3 
(A) 3   (B) 3  
 2   2 
2 3  2 3 
(C)   (D)  
 2   2 
26. The displacement of a body at any time t after starting is given by s=10t – (0.1)t2. The velocity of the
body is zero after:
(A) 50 s (B) 100 s
(C) 80 s (D) 40 s

ANSWER KEY
Questions Answers Questions Answers Questions Answers Questions Answers
1 A 8 B 15 A 22 B
2 A 9 A 16 C 23 A
3 C 10 C 17 A 24 A
4 B 11 D 18 B 25 A
5 A 12 B 19 A 26 A
6 A 13 A 20 C
7 B 14 A 21 B

34
UNITS & DIMENSIONS PHYSICS

UNITS & DIMENSIONS


PHYSICS
Physics is the branch of science in which we deal with observations, measurement and description of natural
phenomenon related to matter and energy. These natural phenomena can be classified as mechanics,
properties of matter, sound, thermodynamics, light, electricity, atomic physics, nuclear physics, particle
physics, semiconductors, superconductors and so on.
In Physics, we attempt to explain diverse physical phenomena in terms of a few concepts and laws. The
effort is to see the physical world as manifestation of some universal laws in different domains and
conditions. For example, the same law of gravitation (given by Newton) describes the fall of an apple to the
ground, the motion of the moon around the earth and the motion of planets around the sun. Similarly, the
basic laws of electromagnetism (Maxwell’s equations) govern all electric and magnetic phenomena. The
attempts to unify fundamental forces of nature reflect this same quest for unification.
A related effort is to derive the properties of a bigger, more complex, system from the properties and
interactions of its constituent simpler parts. This approach is called reductionism and is at the heart of physics.

PHYSICAL QUANTITIES
Those quantities which can describe the laws of physics & possible to measure are called physical quantities.
A physical quantity is that which can be measured directly or indirectly.
Physical quantities are of three types

Fundamental Derived Supplementary


or Quantities Quantities
Basic quantities
Physical quantity is completely specified, if It has either
(i) Numerical value only (ratio); e.g. refractive index dielectric constant etc. (or)
(ii) Magnitude only (scalar); e.g. mass, charge etc. (or)
(iii) Magnitude and Direction (vector); e.g. Displacement, torque etc.

Note: There are also some physical quantities which are not completely specified even by magnitude,
unit and direction. These physical quantities are called tensors. e.g. moment of Inertia.

Fundamental Quantities

The physical quantities which do not depend upon any other physical quantity are called fundamental or
absolute or basic quantities. Initially, only three fundamental quantities – length, mass and time were
considered. With the development of science, four more basic physical quantities were added. These are
temperature, electric current, luminous intensity and amount of a substance.

Derived quantities

The physical quantities derived from fundamental quantities are called derived quantities.
distance mass
e.g. speed = , Density =
time volume

35
PHYSICS UNITS & DIMENSIONS

Example 1: Which of the following sets can not enter into the list of fundamental quantities in any system of
units.
(A) Length, mass and velocity (B) Length, time and velocity
(C) Mass, time and velocity (D) Length, time and mass
Solution: The group of fundamental quantities are those quantities which do not depend upon other physical
quantities in the group. But is set (B) we can predict the relation between given quantities as length =
velocity × time. Hence set (B) cannot enter into the list of fundamental quantities.

UNITS
The fixed and definite quantity taken as standard of reference with which the physical quantity of the same
kind can be measured is defined as a unit. Unit is defined as standard of measurement. For the measurement
of a physical quantity a definite magnitude of quantity is taken as standard and the name given to this
standard is called unit.
The measuring process
To determine the magnitude of a physical quantity completely we should know:
(1) the unit in which it is measured
(2) the number of times that unit is contained in the physical quantity (also called numerical value)
 Physical quantity = Numerical value (N) unit (U)
For the same measurement, NU = constant

Note: While selecting a unit for measurement of a physical quantity, following points should be
kept in mind:
(i) Selected unit should represent quantity to be measured.
(ii) Size of unit should be in comparison with size of quantity to be measured.

Fundamental or Base units


The units of fundamental physical quantities are called fundamental units. For example, units of length,
mass and time or those of fundamental physical quantities are called fundamental units.
Derived units
Units of derived physical quantities are called derived units. For example, units of velocity, density,
force, momentum, volume etc.

SYSTEMS OF UNITS
A complete set of base and derived units of all the physical quantities is called system of units.
Initially three systems of units, namely, CGS, FPS and MKS based on three fundamental quantities Length,
Mass and Time came into existence. In 1970, in a world conference a consenses evolved and a standard
international system of units was developed. It is more popularly known as SI (abbreviated for its French
name). In addition to seven fundamental units, two supplementary units were also included, namely, radian
abbreviated as ‘rad’ (unit of supplementary quantity – plane angle) and steradian abbreviated as ‘sr’ (unit of
supplementary quantity – solid angle).
CGS System
The system is also called Gaussian system of units, In this system length, mass and time have been taken
as the fundamental quantities, and corresponding fundamental units are centimeter (cm), gram (g) and
second (s) respectively.

36
UNITS & DIMENSIONS PHYSICS

FPS System
It uses foot, pound and second for the length, mass and time measurement respectively.
MKS System
In this system the length, mass and time have been taken as the fundamental quantities, and the
corresponding fundamental units are the meter, kilogram and second.
International System of Units [SI Units]
Fundamental Quantities and their units
S. No. Physical quantity Unit Symbol
1. Length metre m
2. Mass kilogram kg
3. Time second s
4. Temperature kelvin K
5. Electric current ampere A
6. Luminous intensity candela cd
7. Amount of substance Mole mol
Supplementary Quantities and their units
S. No. Physical quantity Unit Symbol
1. Plane angle radian rad
2. Solid angle steradian sr
While writing a unit, the following conventions are adopted
(i) Unit named after a person starts with a capital letter, for example, Newton is written as N, Curie as Ci.
The full name of a unit always begins with a small letter even if it is named after a scientist.
Example: 5 N or 5 newton.
(ii) Fundamental units are written with small letters, for example, meter as m and kilogram as kg etc.
(iii) The symbols are not expressed in plural form. For example, 50 meters will be written as 50 m.
(iv) Punctuation mark such as full stop are not used after the symbol of unit. For example, 1 milliliter is
written as 1 ml or 1 cc (not m.l. or c.c.)

MAGNITUDES
In physics we have to deal from very small (micro) to very large (macro) magnitudes as on one side we talk
about the atom while on the other side of universe e.g. the mass of an electron is 9.11031 kg while that of
the sun is 2 1030 kg.
PREFIXES FOR POWER OF TEN
Power of 10 Prefix Symbol Power of 10 Prefix Symbol
18 exa E 1 deci d
10 10
1015 peta P 102 centi c
1012 tera T 103 milli m
109 giga G 106 micro 
10 6 mega M 109 nano n
3 kilo k 12 pico p
10 10
2 hecto h 15 femto f
10 10
1 deca da 18 atto a
10 10
For example: 1 micro second = 1 s = 10–6 s
1 nano second = 1 ns = 10–9 s
1 kilo-meter = 1 km = 10–3 m

37
PHYSICS UNITS & DIMENSIONS

Example 2: Convert all in meters (m):


(i) 5 m (ii) 3 km
(iii) 20 mm (iv) 73 pm
(v) 7.5 nm
Solution:
(i) 5 m = 5 × 10–6 m
(ii) 3 km = 3 × 103 m
(iii) 20 mm = 20 × 10–3 m
(iv) 73 pm = 73 ×10–12 m
(v) 7.5 nm = 7.5 × 10–9 m

While using prefixes


(a) The use of double prefixes should be avoided as far as possible, i.e. instead of μμF we must write
pF and instead of kMW we must write GW.
(b) When prefix is placed before the symbol of a unit, the combined prefix and symbol must be raised to
a positive or negative power without any bracket e.g. km3 means (10 m) but never 103m3 or μs
3 3 -1

-6 -1
means (10 s) but not 10-6s -1 .
(c) Using the table of prefixes we can express magnitudes of many physical quantities more compactly,
e.g, 5×10-6 F will be written as 5μF or 1.3×106eV as 1.3MeV .

Note: While expressing the magnitude of a physical quantity, keep in mind that normally decimal is used
after first digit using powers of ten, e.g., 3750 m will be written as 3.750  103 m or 3.750 km.

LENGTH
Length is the measure of intervals in space. So to specify a distance we must use some unit of length. Most
common unit of length is meter. Originally one meter was defined as (1/10 7)th part of the distance between
pole and equator along a meridian through Paris. But this is a not a very convenient definition for practical
use.
In 1889 the standard meter was redefined as the distance between two scratches engraved on a platinum -
iridium bar at a constant temperature of 273.16 K and 1 bar pressure preserved in the International Bureau of
Weights and Measures at Sevres near Paris, France. All other metallic meters had to be calibrated against this
meter and has an accuracy of 1 part in 107 parts.

MASS
Mass is a basic property of matter. The most common unit of the mass is kilogram. Originally kilogram was
defined as the mass of 1 liter i.e. 1000 cc of water at 4 degree Celsius, the temperature at which the water has
maximum density.

Note: In Astrophysics sometimes we come across a term Chandrasekhar limit. This term refers to
1.4 times the mass of sun ( 2 10 kg ) i.e. 1 Chandrasekhar limit  2.8×1030 kg .
30

[Chandrasekhar has shown that if the mass of a star becomes 1.4 times the mass Sun, under
gravitational collapse it turns to a white dwarf.]

38
UNITS & DIMENSIONS PHYSICS

TIME
Time is defined as the duration between the successive happenings of the same event. According to Einstein,
time is what a clock reads. Originally, 1 second was defined as the time taken by the bob of simple pendulum
of length 99.3 cm to go from one extreme position to other extreme position at sea level.
Later on, 1 second was defined as 1/ 86400 of a mean solar day.

Note: Regarding, time here it is worth noting that:


(1) Time can never flow back i.e. it cannot decrease, as from present, future will come and
not the past.
(2) At a given instant of time a physical object cannot be present at more than one position in
space.
CHARACTERISTICS OF A STANDARD UNIT
(i) It should be well defined.
(ii) It should not change with time, place & physical conditions such as temperature and pressure.
(iii) It should be easily accessible.
(iv) It should be reproducible.
(v) It should be convenient in use.

DEFINITIONS OF SI UNITS
1. Meter (m): Since 1960, the standard meter is defined in terms of the wavelength of light and is called
atomic standard of length. The meter is the distance containing 1650763.73 wavelengths in vacuum of
the radiation corresponding to orange – red light of krypton– 86. An accuracy of 1 part in 109 parts can be
achieved with it.
At present, meter is defined as the length of the path travelled by light in vacuum in (1/299,792,458)th
part of a second (1983).
2. Kilogram (kg): One kilogram is the mass of prototype [a certain platinum (90%) – iridium alloy (10%)
cylinder] preserved at the International Bureau of Weights and Measures, at Severs, near Paris, France (1889).
3. Second (s): One second is the duration of 9,192,631,770 periods of the radiation corresponding to the
transition between the two hyperfine levels of the ground state of the cesium-133 atom (1967).
With this clock it is possible to achieve an accuracy of 1 part in 1012 parts i.e. a cesium clock will go
wrong by 1 s in (10 / 3.16 10 )  30000 years.
12 7

4. Ampere (A): One ampere is that constant current which, if maintained in two straight parallel conductors
of infinite length, of negligible circular cross-section, and placed 1 meter apart in vacuum would produce
between these conductors of a force equal to 2  10–7 newton per meter of length (1948).
5. Kelvin (K): One Kelvin is the fraction 1/(273.16) of the thermodynamic temperature of the triple point of
water (1967).
6. Candela (cd): One candela is the luminous intensity, in a given direction, of a source that emits
monochromatic radiation of frequency 540  1012 hertz and that has a radiant intensity of 1/683 watt per
steradian in that direction (1979).
7. Mole (mol): One mole is that amount of substance which contain as many elementary entities as there are
atoms in 0.012 kg of carbon-12 isotope (1971). The entities may be atoms, molecules, ions etc.
12 12
As the mass of a Carbon–12 atom is, = kg
Avogardo's No 6.02×1026
1  12 
So, 1 amu (or u) = × 26 
=1.67×10-27 kg
12  6.02×10 

39
PHYSICS UNITS & DIMENSIONS

SUPPLEMENTARY PHYSICAL QUANTITIES


1. Plane angle ( θ ): Plane angle is defined as the angle subtended at the centre of
circle (planer geometry) by an arc.
l length of arc
 (in radian)  
r radius of circle
Radian (rad): 1 radian is the angle subtended at the centre of a circle by an arc
equal in length to the radius of the circle.

Arc r
Thus     1 rad
Radius r
2. Solid angle (): Solid angle is the two-dimensional angle subtended at a
point in three-dimensional space by certain area. It is a measure of how
large the area appears to an observer looking from that point.

Surface Area

Radius2
Steradian (sr): 1 steradian is the solid angle subtended at the centre of a
sphere by a surface of the sphere equal in area to square of radius of that
sphere.
Surface Area r 2
Thus    2  1sr
Radius 2 r
Note: Solid angle subtended at the center of complete sphere is 4 sr .

DEFINITION OF SUPPLEMENTARY UNITS


Advantages of SI
Following are the main advantages of SI over the other system of units:
1. SI is a coherent system of units, i.e. a system based on a certain set of fundamental units, from which
all derived units are obtained by multiplication or division without introducing numerical factors.
2. SI is a rational system of units, as it assigns only one unit to a particular physical quantity. For example,
joule is the unit for all types of energy. This is not so in other system of units. For example, in mks
system, mechanical energy is in joule, heat energy is in calorie and electric energy is in watt hour.
3. SI is an absolute system of units. There are no gravitational units on the system. The use of factor ‘g’
is thus eliminated.
4. SI is a metric system, i.e., the multiples and submultiples of units are expressed as powers of 10.
5. In current electricity, the absolute units on the SI, like ampere (A) for current, volt (V) for potential
difference, ohm () for resistance, Henry (H) for inductance, farad (F) for capacity and so on,
happen to be the practical units for measurement of these quantities.
MEASUREMENT OF LENGTH BY INDIRECT METHODS
MEASUREMENT OF MODERATE DISTANCES
(i) Triangulation method: In this method, the angular elevation of the object, whose
length is to be measured is determined with respect to a point. Using this angle, the
required length can be calculated.
(a) Accessible object: Let h be the height of an accessible object AB situated at a
distance d (known) from a point O (figure). Let  be the angle subtended by the height h at O.

40
UNITS & DIMENSIONS PHYSICS

h
In ΔOAB,  tan 
d
Thus by measuring d and , h can be measured.
(b) Inaccessible object: If the object AB is inaccessible, i.e., its distance from the points of observation
is not known, let the angular elevation of AB be noted with respect to two points O 1 and O2 separated
a distance ‘d’ apart (figure).
In triangles O1AB and O2AB,
d1 d
cot 1  and cot 2  2
h h
d1  d2 d
 cot 1  cot 2  
h h
d
or h 
cot 1  cot  2
Thus by measuring d and 1 & 2, h can be measured.

MEASUREMENT OF LARGE DISTANCES


(1) Reflection method or Echo method
To measure the distance of an object by this method we send a wave signal (sound or light) towards the
object. The reflected signal is received, by a receiver, at the same
place (in figure). Let ‘t’ be the time taken by the signal to cover two
way journey.
where x = distance of the object from place of observation and
v = velocity of signal (sound or light).
2x = v t
t
 x  v
2
APPLICATIONS OF REFLECTION METHOD
(i) LASER method: The word LASER stands for Light
Amplification by Stimulated Emission of Radiation. The Laser
light can travel long distances without fading its intensity. A laser
beam is sent towards the object (moon etc) whose distance is to
be measured and its reflected pulse is received. If t is the time
elapsed between the instants the laser beam is sent and it returns,
ct
then the distance of the moon from the earth is given by, s  ,
2
where c is the speed of light, which is 3  108 m/s.
(ii) RADAR method: The word RADAR stands for Radio
Detection and Ranging. A radar can be used to measure
accurately the distance of an aero plane etc. Radio waves are
sent from a transmitter which after reflection from the aero
plane are detected by the receiver. If t is the time between the
instants the radio waves are sent and received, then the distance

41
PHYSICS UNITS & DIMENSIONS

ct
of the aero plane is given by s  , where c = 3  108 m/s is the speed radio waves.
2
(iii) SONAR method: The word SONAR stands for Sound Navigation and Ranging. This method is used
to detect the submarines or to find the depth of sea. Ultrasonic waves (waves of frequency greater
than 20000 Hz) are sent into the sea; they are reflected by the bottom
of sea and received by the receiver. Transmitter and receiver are set
into the ship. If t is the time taken by the ultrasonic waves from the
instant of transmission to receiving, then depth of sea is given by,
vt
s , where v is the speed of sound waves in water, which is nearly
2
1498 m/s.
(2) Parallax method
Large distances such as the distance of a planet or a star from the earth cannot be measured directly with a
meter scale. An important method in such cases is the parallax method. When you hold a
pencil in front of you against some specific point on the background (a wall) and look at
the pencil first through your left eye A (closing the right eye) and then look at the pencil
through your right eye B (closing the left eye), you would notice that the position of the
pencil seems to change with respect to the point on the wall. This is called parallax.
The distance between the two points of observation is called the basis. In this
example, the basis is the distance between the eyes. To measure the distance D of a far away planet S by
the parallax method, we observe it from two different positions (observatories) A and B on the Earth,
separated by distance AB = b at the same time as shown in Figure. We measure the angle between the
two directions along which the planet is viewed at these two points. The triangle ASB in figure
represented by symbol  is called the parallax angle or parallactic angle.
As the planet is very far away, b/D<< 1 and therefore,  is very small. Then we approximately take AB
as an arc of length b of a circle with centre at S and the distance D as the radius AS = BS so that AB = b
= D, where  is in radians.
b
D

Distance of a near by star (using parallax method)
In figure the near by star whose distance S is to be found is shown. Taking a distance star P (fixed star)
whose position remains fix for all positions of the earth in its orbital motion. When the earth is at position
E1, let 1 is angle subtended by star from reference E1P and 2 when
earth is at the position E2.
Arc EE
The parallactic angle  = 1 + 2   1 2
Radius S
E1 E2
 S

The distance E1E2 is the diameter of the orbital plane of earth around sun. This method is useful for the
determination of distances which are less than 100 light years away from the earth.

Note: For a star more than 100 light years away, the parallax angle is so small that it cannot be
measured accurately.

42
UNITS & DIMENSIONS PHYSICS

(3) Measurement of the diameter of the moon


Let  be the angle subtended by two diametrically opposite ends A and B of the moon at point E on earth
(figure).
If r = Distance of moon from earth,
Arc d
then  
Radius r
where, d is the diameter of the moon
 d  rx
Hence, diameter of the moon can be calculated only by measuring the angle  if r is
known.

MEASUREMENT OF VERY SMALL DISTANCES


(1) To estimate size of molecule of Oleic acid
Oleic acid is a soapy liquid with large molecular size of the order of 10 –9 m. The idea is to first form
mono-molecular layer of oleic acid on water surface. We dissolve 1 cm3 of oleic acid in alcohol to make
a solution of 20 cm3. Then we take 1 cm3 of this solution and dilute it to 20 cm3, using alcohol. So, the

concentration of the solution is equal to 


 1  3
 cm .
 20×20 
Pour n drops of this solution of volume V each on the surface of water and sprinkle some Lycopodium
powder. This solution of oleic acid spreads very fast on the surface of water and forms a very thin layer
of thickness t. The alcohol evaporates and oleic acid layer is left on the surface. If A is area of surface
volume of the film nV
film left, then thickness of film is given by t   cm
Area of the film 20  20 A
If we assume that the film has mono-molecular thickness, then this becomes the size or diameter of a
molecule of oleic acid. The value of this thickness comes out to be of the order of 10-9 m.

MEASUREMENT OF MASS
Measurement of Gravitational Mass
Gravitational Mass of a body is measured by using a spring balance or a dynamometer (figure). The design
of a spring balance is based on the fact that elastic force is directly proportional to the
elongation of the spring when an object is attached to the lower end of the spring
balance. The elongation itself is proportional to the gravitational force on the object and
hence its gravitational mass. If the spring balance is already calibrated in terms of
standard masses, the gravitational mass of an unknown object is easily measured. Since
the value of gravity changes from place to place, the measurement of gravitational mass
is precise if it is carried out at the same place where the spring has been calibrated. In a
common balance, there is no such restriction as the gravity affects the object and the
standard masses equally.
Measurement of Inertial Mass
Inertial mass can be measured by using an inertial balance. It consists of a long metallic strip. One end of the
strip is clamped to a table and the second end vibrates horizontally.
Let T1 be the time period of vibration of the strip with mass m1 an d T2 be the time period with the mass
m2.

43
PHYSICS UNITS & DIMENSIONS

Now m1  T12
and m2  T22
m1 T12
 
m2 T22
Thus, two unknown masses can be compared by knowing the values
of T1 and T2. If mass of a standard body is known, the mass of other
body can be calculated.

MEASUREMENT OF TIME
To measure usual time intervals, a clock or a stop watch or a digital watch is used. Very small and very large time
intervals have been measured with great accuracy. Various techniques for measuring time intervals are given below:
(i) Electrical oscillators: These use a.c. supply of frequency 50 Hz. The rotations of a synchronous motor
run on a.c. is used to obtain a time scale.
(ii) Electronic oscillators: A vacuum tube or a semi - conductor device is used to produce electromagnetic
oscillations of very high frequency. The time period of such oscillations can be used to measure small
time intervals.
(iii) Quartz Crystal clocks: A quartz crystal shows piezo electric effect that is, if such a crystal is subjected
to a fluctuating pressure across its one pair of parallel faces, an oscillatory e.m.f is developed across
another pair which is perpendicular to earlier pair of faces and vice -versa. The oscillations so produced
can be used to measure time intervals. Such clocks are quite in use.
(iv) Atomic clocks: Such clocks make use of periodic vibrations taking place within the atoms like that of cesium.
Quartz crystal and atomic clocks are more accurate than the clocks based on astronomical observations.
If two cesium clocks are operated, the accuracy of cesium atom clock is 1 part in 10 11 which is 200 times
more accurate than the accuracy of the clock based on astronomical observations.
A clock based on hydrogen MASER (Microwave Amplification by Stimulated Emission of Radiations) will
give error of 1 second in 33,000,000 years which is 6500 times more accurate than the cesium atom clock.
(v) Decay of elementary particles: Many unstable elementary particles decay in very short time interval ranging
from 10–10 second to 10–24 second. By studying their decay, very small time intervals can be measured.
(vi) Radioactive dating: Very long time intervals, such as age of fossils (carbon dating), rocks and earth
(Uranium dating) etc. are estimated by a technique known as radioactive dating.
Example 3: The parallax of a heavenly body measured from two points diameterically opposite on equator of
earth is 1.0 minute. If the radius of earth is 6400 km, find the distance of the heavenly body from
the centre of earth in AU, taking 1AU = 1.5  1011 m.
10 1 
Solution: Here,   1    rad
60 60 180
l = diameter of earth = 2  6400 km = 1.28  104 km = 1.28  107 m
distance, r = ?
As l  r
l 1.28 107
r   4.4 1010 m
  / 60 180
4.4 1010
r  0.293 AU
1.5 1011

44
UNITS & DIMENSIONS PHYSICS

Example 4: The moon is observed from two diametrically opposite points A and B on earth. The angle 
subtended at the moon by the two directions of observation is 1 o54. Given the diameter of earth
to be about 1.276  107 m, calculate the distance of moon from earth.
Solution: Here,   1o54  1.9o

   1.9   3.32 102 rad
180
Diameter of earth, l  r
1.276 107
1
 r   3.84 108 m
 3.32 10 2

Example 5: The sun’s angular diameter is measured to be 1920. The distance of the sun from the earth is
1.496  1011m. What is the diameter of the sun?
1920 
Solution: Here,   1920   radian,
60  60 180
r = 1.496  1011 m, l = ?
As l = r
1920 
 l  1.496 1011    1.39 109 m
60  20 180
Example 6: A man wishes to estimate the distance of a nearby tower from him. He stands at a point A in
front of the tower C and spots a very distant object O in line with AC. He then walks
perpendicular to AC upto B, a distance of 100 m and looks at O and C again. Since O is very
distant, the direction of BO is practically the same as AO, but he finds the line of sight of C
shifted from the original line of sight by an angle  = 40o ( is known as parallax). Estimate the
distance of the tower C from his original position A.
Solution: As is clear from figure, parallax angle,
 = 40o, AB = 100m, AC = ?
AB
In ΔABC , tan  
AC
AB 100
AC  
tan  tan 40o
100
AC   119m
0.8391
Example 7: Two atomic clocks allowed to run for average life of an Indian (say, 70 years) differ by 0.2s
only. Calculate the accuracy of standard atomic clock in measuring a time interval of 1 sec.
Solution: Time interval = 70 yrs = (70  365 +17) days
(17 additional days are on account of 17 leap years during 70 yrs)
= 25567 days = 25567  24  60  60 sec
Difference = 0.2s
0.2
 Fractional Error =  9.04 1011  1010
25567  24  60  60
Hence, accuracy shown by atomic clock is 10–10s in 1s or 1s in 1010s
Example 8: If the size of a nucleus (  10–15 m) is scaled up to the tip of a sharp pin (  10–5 m), what roughly is
the size of an atom?

45
PHYSICS UNITS & DIMENSIONS

105
Solution: Magnification = size of tip of pin/size of nucleus = 15
 1010
10
As actual size of atom  10–10 m and it is magnified 1010 times
Apparent size of atom = 10–10  1010 = 1m
Example 9: Ten drops of olive oil of radius 0.20 mm spread into a circular film of radius 14.6 cm on the
surface of water. Estimate the size of an oil molecule.
Solution: Here, n = 10, r = 0.20 mm = 2  10–4 m, R = 14.6 cm = 14.6  10–2 m.
Let d be the diameter of oil molecule = thickness of layer
volume of olive oil = area  thickness of layer
4
n   r 3   R2  d
3
 
3
4
4 r 3 4 10 2 10
d  n    5  109 m
3R 2

3 14.6 10 2

SOME PRACTICAL UNITS
There are some practical units which are simultaneously used with SI units.
1. 1 Fermi = 10–15 m
2. 1 angstrom (Å) = 10–10 m
3. 1 nanometre (nm) = 10–9 m
4. 1 micron (m) = 10–6 m
5. 1 amu = 1.66  10–27 kg
6. 1 quintal = 100 kg
7. 1 tonne = 1000 kg
8. 1 lunar month = 27.3 days
9. 1 solar year = 365.25 average solar days = 366.25 sidereal days
10. 1 shake = 10–8 s
11. 1 Astronomical unit = 1 AU = 1.49 1011 m
[Average distance between sun and earth, i.e. radius of earth’s orbit]
15
12. 1 Light year = 1 ly = 9.46 10 m [Distance that light travels in 1 year in vacuum]
13. 1 parsec = parallactic second = 3.08 1016 m = 3.26 light year
[The distance at which a star subtends an angle of parallax of 1 sec at an arc of 1 AU]
l l
 or r 
r 
1 parsec 
1 AU 1.496 1011
Thus   3.08 1016 m
1   1 1 
   
 180 60 60 
Also 1 parsec = 3.26 ly
14. 1 Sidereal day: Time taken by earth to complete 1 rotation about its own axis with respect to some distant
star.
366.25
1 Solar day  sidereal day
365.25
15. Lunar month: The time taken by moon to complete 1 revolution around the earth.
1 lunar month = 27.3 days

46
UNITS & DIMENSIONS PHYSICS

DIMENSIONAL ANALYSIS

DIMENSIONS AND DIMENSIONAL FORMULAE


All physical quantities can be expressed in terms of the seven fundamental units. We call these seven
physical quantities as seven dimensions of the physical world. Thus taking mass as [M]. length [L] and time
[T] in mechanics all physical quantities can be expressed in terms of [ M], [L] and [T].

Dimensions
Dimensions of a physical quantity are the powers to which the fundamental quantities must be raised to
represent the given physical quantity.
mass [M]
e.g.: density = = 3 = [ML-3 ]
volume [L ]
So, the dimensions of density are 1 in mass, – 3 in length and 0 in time.
 Height, width, radius, displacement etc. are a kind of length. So, we can say that their dimension
is [L]
 [Volume] = [Length] × [Height] = L × L × L = [L3]
4 3
For sphere, Volume = r
3
4 
[Volume] =     r   1  L    L 
3 3 3

3 
So, dimensions of volume will be always 0 in mass, 3in length and 0 in time whether it is volume of
a cuboid or volume of sphere.
Dimensional Formula
It is an expression which tells, which of the fundamental units are present in a given physical quantity
and with what powers.
[Ma Lb Tc] is the dimensional formula of a physical quantity which had dimensions a, b and c in mass,
length and time respectively. So, dimensional formula of force is [MLT –2].

47
PHYSICS UNITS & DIMENSIONS

Dimensional Equation
Whenever a physical quantity is equated to dimensional formula, we get a dimensional equation. So,
dimensional equation for force is, [F] = [MLT –2]
In general, and physical quantity X, having dimensional formula MaLbTc, has a dimensional equation,
[X] = [Ma Lb Tc]
No. Physical Quantity Formula S.I. Unit Dimensional Formulae
1. Length () meter [L]
2. Distance (s) meter [L]
3. Displacement(s) Meter [L]
4. Mass (M) kilogram [M]
5. Time (t) Second [T]
2
6. Area (A) length × breadth (meter) [L2]
3
7. Volume (V) length × breadth × height (meter) [L3]
8. Density (r) mass kg/m3 [ML–3]
volume
density of body
9. Specific gravity No unit dimensionless
density of water at 40C
10. Speed (V) distance covered meter/sec. [LT–1]
time taken
displacement
11. Velocity (V) meter/sec. [LT–1]
time
changein velocity
12. Velocity gradient per second [T–1]
distance
13. Momentum (P) mass × velocity kg-metre/sec. [MLT–1]
14. Acceleration (a) or metre/sec2 [LT–2]
Acc. due to gravity (g)
15. Force (F) or mass × acceleration kg-metre/sec2 [MLT–2]
Weight (w) or kg-metre/sec2 [MLT–2]
Tension (T) or kg-metre/sec2 [MLT–2]
Normal reaction (R or N) kg-metre/sec2 [MLT–2]
force of friction
16. Coefficient of friction() No unit dimensionless
normal reaction
17. Force constant or kg/sec2 [MT–2]
Spring constant or
Spring factor or
Stiffness (k)
18. Work (W) force × displacement kg m2/s2 [ML2T–2]
19. Kinetic energy (K) kg m2/s2 [ML2T–2]
20. PE (U) mgh kg m2/s2 [ML2T–2]
21. Energy (All forms of energy) kg m2/s2 [ML2T–2]
work
22. Power (P) kg m2/s3 [ML2T–3]
time
relative velocity of separation
23. Coefficient of No unit dimensionless
relative velocity of approach
restitution (e)

48
UNITS & DIMENSIONS PHYSICS

24. Impulse (I) force × time kg m/s [MLT–1]


25. Pressure (P) force N/m2 [ML–1T–2]
area
26. Gravitational
Fr 2 N m2/kg2 [M–1L3T–2]
G=
m1m2
constant (G)
GM
27. Gravitational field m2/s2 [LT–2]
r2
strength (g)
GM
28. Gravitational  m2/s2 [L2T–2]
r
potential (V)
Gm1m2
29. Gravitational PE  kg m2/s2 [ML2T–2]
r
2GM
30. Escape velocity (Ve) m/s [LT–1]
r
GM
31. Orbital velocity (V0) m/s [LT–1]
r
force
32. Stress N/m2 [ML–1T–2]
area
arc
33. Angle () radian dimensionless
radius
changein configuration
34. Strain No unit dimensionless
original configuration
stress
35. Modulus or coefficient N/m2 [ML–1T–2]
strain
of elasticity
1
36. Compressibility (K) kg–1 metre sec–2 [M–1LT2]
bulk modulus
lateralstrain
37. Poisson ratio (s) No unit dimensionless
longitudinalstrain
force
38. Surface tension (T) N/m [MT–2]
length
volume
39. Rate of flow m3/s [M0L3T–1]
time
40. Coefficient of viscosity ()
F = 6prv kg m–1 s–1 [ML–1T–1]
41. Moment of force or force × distance Nm [ML2T–2]
Torque (  )
42. Angular displacement (  ) radian dimensionless
angular displacement
43. Angular frequency or radian/sec [M0L0T–1]
time

49
PHYSICS UNITS & DIMENSIONS

angular velocity ()


44. Radius of gyration (K) Distance m [M0LT0]
45. Moment Inertia (I) Mass × (radius of gyration)2 kg m2 [ML2T0]
46. Angular acceleration () rad/sec2 [T–2]
47. Angular momentum (J) Moment of inertia × angular Velocity
kg m2 s–1 [ML2T–1]
1
48. Rotational kinetic Iw 2 kg m2 s–2 [ML2T–2]
2
energy
49. Angular Impulse torque × time kg m [ML2T–1]
50. Time period (T) second [T]
1
51. Frequency () per second [T–1]
time period
52. Wave velocity m/s [LT–1]
53. Wavelength () Distance between two nearest m [L]
points in the same phase
54. Amplitude of wave (a) Maximum displacement m [L]
55. Energy density (u) Energy per unit volume kg metre–1 sec–2 [ML–1T–2]
energy
56. Intensity of wave kg sec–3 [MT–3]
area  time
57. Temperature kelvin (K) [K] or []
58. Heat energy (Q) kg m2 s–2 [ML2T–2]
59. Mechanical equivalent work done No unit dimensionless
heat energy
of heat (J)
Q
60. Specific heat (C) C m2 s–2 K–1 [L2T–2K–1]
m
Q (heat energy)
61. Latent heat of fusion or L  m s–2 [L2T–2]
m (mass)
latent heat of vaporization
62. Molar gas constant (R) PV = nRT kg m2s–2 K–1 [ML2T–2K–1]
molar gas constant
63. Boltzmann constant (K) kg m2s–2K–1 [ML2T–2K–1]
Avogadro's number
dQ
64. Rate of cooling kg m2 s–3 [ML2T–3]
dt
e
65. Emissivity ()  No unit dimensionless
E
66. Wein’s constant (b) mT mK [LK] or [L]
Q
67. Intensity of thermal kg sec–3 [MT–3]
At
radiation (E)
E
68. Stefan’s constant (s) kg sec–3k–4 [MT–3K–4] or
T4
[MT–3–4]

50
UNITS & DIMENSIONS PHYSICS

d
69. Temperature gradient Kelvin m–1 [L–1]
dx
Qd
70. Coefficient of thermal K kg m s–3 K–1 [MLT–3K–1]
A 1   2  t
conductivity (K)
1   2
71. Thermal resistance Kg–1m–2sec3 [M–1L–2T3K]
Q
energy of photon
72. Plank’s constant (h) h kg m2 s–1 [ML2T–1]
frequency
luminous intensity
73. Intensity of illumination m–2 candela [L–2cd]
 distance 
2

74. Electric charge (q) current × time ampere second [TA]


q1q2
75. Permittivity constant () coulomb2 newton–1 [M–1L–3T4A2]
4 r 2 F

76. Dielectric constant or K No unit dimensionless
0
relative permittivity (r or K)
charge
77. Electric current ampere [A]
time
charge
78. Linear charge density  coulomb/metre [L–1TA]
length
charge
79. Surface charge density   coulomb/metre2 [L–2TA]
area
charge
80. Volume charge density   coulomb/metre3 [M–3LA]
volume
work done
81. Electric potential volt [ML2T–3A–1]
charge
82. Electric potential gradient
electric potential
volt/metre [MLT–3A–1]
distance
83. Electric field strength (E)
electric force
N/coulomb [MLT–3A–1]
charge
84. Electric flux (E) electric field × area volt metre [ML3T–3A–1]
85. Electric dipole moment P = 2ql coulomb metre [LTA]
86. Electric susceptibility ( = r – 1) No unit dimensionless
charge
87. Capacitance (C) farad (F) [M–1L–2T4A2]
potential difference
potential difference
88. Resistance (R) ohm [ML2T–3A–2]
current
RA
89. Specific resistance or ohm metre [ML3T–3A–2]
l
resistivity ()

51
PHYSICS UNITS & DIMENSIONS

1
90. Conductance (G) Siemen [M–1L–2T3A2]
R
1
91. Conductivity (s) Siemen/metre [M–1L–3T3A2]

R  R0
92. Temperature coefficient t K–1 [–1] or [K–1]
R0t
of resistance ()
m
93. Electrochemical kg/coulomb [MA–1T–1]
Q
equivalent (Z)
dt
94. Coefficient of self e henry (H) [ML2T–2A–2]
di
induction or self inductance (L)
F
95. Magnetic induction (B) tesla (T) [MT–2A–1]
qv
96. Magnetic flux (B) BA weber (Wb) [ML2T–2A–1]
97. Magnetic dipole moment (M)
current × area ampere metre2 [L2A]
M
98. Magnetic charge or ampere metre [LA]
2l
pole strength (m)
Fr
99. Magnetic permeability weber/ampere metre [MLT–2A–2]
i 2l
of free space (0)
100. Magnetic field intensity (H)
B
ampere/metre [L–1A]
0
101. Intensity of magnetization (I)
Pm
ampere/metre [AL–1]
V
102. Magnetic susceptibility (m)
I
no unit dimensionless
H
Physical Quantities Having Same Dimensions

Dimensions Physical Quantity


[M0L0T–1] Frequency, angular frequency, angular velocity, velocity gradient and decay constant
[M1L2T–2] Work, internal energy, potential energy, kinetic energy, torque, moment of force
[M1L–1T–2] Pressure, stress, Young’s modulus, bulk modulus, modulus of rigidity, energy density
[M1L1T–1] Momentum, impulse
[M0L1T–2] Acceleration due to gravity, gravitational field intensity
[M1L1T–2] Thrust, force, weight, energy gradient
[M1L2T–1] Angular momentum and Planck’s constant
[M1L0T–2] Surface tension, Surface energy (energy per unit area)

52
UNITS & DIMENSIONS PHYSICS

[M0L0T0] Strain, refractive index, relative density, angle, solid angle, distance gradient, relative
permittivity (dielectric constant), relative permeability etc.
[M0L2T–2] Latent heat and gravitational potential

PRINCIPLE OF HOMOGENEITY OF DIMENSIONS


According to this principle, the dimensions of all the terms occuing on both sides of the equation must be
same.
If, X  A   BC   DEF then according to Principal of Homogeneity
2

X  A   BC    DEF   Ma LbTc


2

Rule No. 1: One quantity can be added/ subtracted with a similar quantity only and give rise to the similar
quantity.
 Suppose in any formula, (L + ) term is coming (where L is length). As length can be added only with a
length, so a should also be a kind of length.
So, [] = [L]
 Similarly consider a term (F – b) where F is force. A force can be added/subtracted with a force only and
give rise to a third force. So, b should be a kind of force and its result (F – b) should also be a kind of
force.

Note: [L]-[L] = [L] and is not equal to zero in dimensions.

 
Example 10: In the following equation: 2
 Fv  2 , find dimensional formula for [] and [ ]. Here t =
t x
time, F = force, v = velocity & x = distance
Solution: Since [Fv] = M1L2T–3,
 
So,  2  should also be M1L2T–3
x 
  = M1L2T–3     = M1L4T–3
 x 2 

 
and  Fv  2
will also have dimensional formula [M1L2T–3]
 x 
 
so, 2 = [M1L2T–3]     [M1L2T–1]
 t 

Rule No. 2: Arguments of trigonometric functions are dimensionless.


Consider a term sin ()
 Perpendicular 
Here  is dimensionless and sin     is also dimensionless.
 Hypoteneous 
Rule No. 3: Power of exponential function and argument of log functions are also dimensionless. This
implies if, y = ex, then x is dimensionless. Similarly, If y = loge x, then x is dimensionless.

53
PHYSICS UNITS & DIMENSIONS

F
Example 11: In the equation,  sin   t  , find the dimension of  and . Here, V = velocity, F = force
V2
and t = time.
Solution: because argument of the sin function is dimensionless.
 t = dimensionless
[ ][t] = [M0L0T0]
[ ] = [M0L0T–1]

So,   
F 

 M1L1T 2 
  M1L1T 0 
V 2  1 1 2
 L T 

FV 2  2 
Example 12: In the equation:   log e  2  , find the dimensions of  and . Where F = force &
 2
 V 
V = velocity.
Solution: Because argument of the log function is dimensionless.
2
 = Dimensionless
V2
 F  V 2 
   
  2 


 2      M L T 
0 0 0

V 2 


    M 0 L0T 0 
 L T 
2 2

      L2T 2 
 M1L1T 2   L2 T 2 
    2     M1L1T0
 L2 T 2 

 n2  n1 
Example 13: In the formula; N   D   , D = diffusion coefficient, n1 and n2 is number of molecules
 x2  x1 
in unit volume at distances x1 and x2 along X-axis. N is number of molecules passing through per
unit area per unit time. Calculate dimensional equation of D.
Solution: By homogeneity theory of Dimension
Dimension of  n2  n1 
Dimension of (N)  Dimension of D 
Dimension of  x2  x1 
1 𝐿−3
𝐿2𝑇
= Dimensional formula of 𝐷 × 𝐿
L  2

 Dimensional formula of ‘D’      L2 T 1 


T  

54
UNITS & DIMENSIONS PHYSICS

Example 14: If in the formula x = 3yz2, x and z represent electrical capacitances and magnetic induction then
calculate dimensional equation of y.
Solution: By the principle of homogeneity of dimension
Dimensional formula of x = Dimension formula of (3yz)2
[M–1L–2T4A2] = Dimensional formula of (y)  [M1L0T–2A–1]2
Dimensional formula of (y) = [M–3L–2T8A4]
Example 15: Find dimensions of resistance R and inductance L. Speculate what physical quantities (L/R) and
1 2
Li represent?
2
di dt
Solution: As e  L i.e., L  e
di di
   2 2

So, [L]  W   t    ML T  T 
 
 q   i   AT A
i.e.,  L   ML2T 2 A 2 
V
and as V = IR i.e. R 
I
   2 2

so,  R    W    ML T 
 qA   ATA 
i.e.,  R    ML2T 3A 2 

L  ML2T 2 A 2   1 
so,     2 3 2 
 1   T 
 R   ML T A   T 
1 
and  Li    ML T A   A   MT T 
2 2 2 2 2 2 2

 2 
1 2
Now as (L/R) has dimensions of time and so is called time constant of L–R circuit and Li has
2
dimensions of work or energy, so it represents magnitude energy stored in a coil.

APPLICATIONS OF DIMENSIONAL ANALYSIS


Following are the main applications of theory of dimensions in physics:

1. To find the unit of a given physical quantity in a given system of units:

Writing the definition of formula for the physical quantity we find its dimensions. Now in the
dimensional formula replacing M, L and T by the fundamental units of the required system we get the
unit of physical quantity. However, sometimes to this unit we further assign a specific name
e.g. Force = mass  acc.
So, [ F ]  [M]  [LT 2 ]  [MLT 2 ]
So its units in CGS system will be g cm/s2 which is called dyne while in MKS system will be kg m/s 2
which is called newton (N).

2. To find dimensions of physical constants or coefficients:

As dimensions of a physical quantity are unique, we write any formula or equation incorporating the
given physical constant and then substituting the dimensional formulae of all other quantities, we can

55
PHYSICS UNITS & DIMENSIONS

find the dimensions of the required constant or coefficient. To make this more clear, consider the
following examples:

(i) Gravitational Constant (G)


I Approach II Approach
From Newton’s law of gravitation, we have: From the relation between G and ‘g’, we have:
m1m2 GM
F G g
r2 R2
Fr 2 gR 2
or G or G
m1m2 M
Substituting the dimensions of all physical quantities:
 MLT 2   L2   LT 2   L2 
G   G  
 M  M  M
or G   M1L3T 2 
So, its SI unit is m3 / kg s2 or Nm2 / kg2
(ii) Planck Constant h
I Approach II Approach
According to Planck: de Broglie:
h
E  h 
mv
E
or h h  mv

Substituting the dimensions of known physical quantities:
 ML2T 2 
 h   h   L M LT1 
T 1 
1
or [h]  [ ML T ] 2

So SI unit of Planck’s constant is kg m2/s which can also be written as (kg m2/s2)  s. But as kg
m2/s2 is joule, so unit of h is joule sec, i.e., J-s.
(iii) Coefficient of Viscosity 
I Approach II Approach III Approach
According to Newton’s Law: Stoke’s Law: Poiseulle’s Formula
dv dV  r 4
F  A F  6 rv 
dy dt 8 l
F F  pr 4
So   
A  dv / dy  6 rv 8l  dV / dt 
Substituting the dimensions of known physical quantities:
 MLT 2   MLT 2   ML1T 2   L4 
   2       
 L   LT 1 / L   L LT1   L L3 /T 
   ML1T1 
So SI or MKS unit of coefficient of viscosity is kg/m s (or g/cm s called poise in CGS system.

Note: From above examples, it is clear that if a physical quantity is given, its dimensions are
unique.
56
UNITS & DIMENSIONS PHYSICS

3. To convert a physical quantity from one system to the other


It is based on the fact that product of numerical value contained in and the unit of physical quantity
remains constant, that is, larger unit has smaller magnitude or
Numerical value unit = constant
n[u] = constant.
If a physical quantity has dimensional formula [MaLbTc] and units of that quantity in two systems are
 M1a Lb1T1c  and  M a2 Lb2T2c  respectively, then
n1[u1] = n2[u2]

 n2  n1
u1 
u2 
a b c
M  L  T 
or n2  n1  1   1   1 
 M 2   L 2   T2 
where n1 and n2 are numerical values in first and second system of units.
Example 16: Find the value of gravitational constant G  6.67 108 cm3 /g s 2 from CGS to MKS System.
Solution: Dimensional formula of G is [M–1L3T–2]
 a = –1, b = 3, c = –2 & n1 = 6.67  10–8
a b c
M  L  T 
As, n2  n1  1   1   1 
 M 2   L 2   T2 
1 3 2
 1g   1cm   1s 
 n2  6.67 10  8
    
 1kg   1m   1s 
but 1cm = 10–2 m and 1g = 10–3 kg
1 3 2
8  103 kg   102 m   1s 
 n2  6.67 10      
 1kg   1m   1s 
1 3
 103   102  8 6
  6.67 10 10 10
8
n2  6.67 10   
3

 1   1 
11
n2  6.67  10 MKS units
i.e. 6.67 108 g 1cm3s 2  6.67 1011 kg 1m3s 2
Alternative Method:
(102 m)3 11 m
3
G  6.67 108  6.67  10
(103 kg) (s2 ) kg s2
i.e. G = 66.67  10–11 MKS units (or SI units)
Example 17: Convert an energy of one joule into erg.
Solution: One joule is absolute unit of energy on m.k.s. system and erg is absolute unit of energy on c.g.s.
system. The dimensional formula of energy is [M1L2T–2]
Therefore, a = 1, b = 2, c = –2
For m.k.s system to c.g.s system
M1 = 1 kg, M2 = 1 g
L1 = 1 M, L2 = 1 cm
T1 = 1 s, T2 = 1 2
n1 = 1 (joule);

57
PHYSICS UNITS & DIMENSIONS

n2 = (no. of ergs) = ?
a b c
M  L  T 
As n2  n1  1   1   1 
 M 2   L 2   T2 
1 2 1 2
 1kg   103 g   102 cm 
2
 1m   1s 
 n2  1        1     1
 1g   1cm   1s   1g   1cm 
n2  103  104  107
Hence, 1 joule = 107 erg
Example 18: Find the value of a force of 100 N on a system based upon the metre, the kilogram and the
minute as the fundamental units.
Solution: Newton is absolute unit of force on mks system.
We have to convert it into a new system.
The dimensional formula of force is [M1L1T–2]. Therefore, the dimensions are:
a = 1, b = 1, c = – 2
a b c
M  L  T 
As, n2  n1  1   1   1 
 M 2   L 2   T2 
1 2 2
 1kg   1s 
1
 1m   1s 
n2  100        100 11  
 1kg   1m   1min.   60s 
n2  100  (60) 2  36  10 4
Hence, 100 N = 36  104 new units of force
Example 19: If unit of length is doubled what is the numerical value of area?
Solution: As unit of length is doubled, unit of Area will become four times. So the numerical value of Area
1
will became one fourth. Because numerical value, n  .
unit
Example 20: Force acting on a particle is 5N. If unit of length and time are doubled and unit of mass is halved
then what is the numerical value of the force in the new unit?
kg  m
Solution: Force = 5
sec2
If unit of length and time are doubled and the unit of mass is halved.
 1/ 2  2  1
Then the unit of force will be =  2 
 times
 (2)  4
Hence the numerical value of the force will be 4 times.

4. To check the dimensional correctness of a given physical relation


This is based on the ‘principle of homogeneity’. According to this principle the dimensions of each term
on both sides of an equation must be the same. (The simple reason for this principle is that mass can be
added to mass to give mass and not to length or time.)
If the dimension of each term on both sides are same, the equation is dimensionally correct, otherwise
not. A dimensionally correct equation may or may not be physically correct.

58
UNITS & DIMENSIONS PHYSICS

mv 2
Example 21: Check the dimensional correctness of the equation, F 
r
Here F = force, m = mass, v = speed and r = radius of the circular path

 M  LT 1 
2

Solution: Dimensionally,  MLT  2

 L2 
i.e.  MLT 2    MLT 2 
As in the above equation dimensions of each term on both sides are same, so this equation is
dimensionally correct.

I
Example 22: Check the dimensional correctness of the equation, T  2
mgL
Here, T = time period, I = moment of inertia, g = acceleration due to gravity and L = length
[ML2 ]
Solution: Dimensionally, [T]   [T]
[M][LT 2 ][L]
As in the above equation the dimension of both sides are same, the given formula is
dimensionally correct. It may or may not be physically correct. However from the theory of
I
physical pendulum we know that T  2 . So the given formula is both dimensionally
mgL
and physically correct.

3 FV 2
Example 23: Pressure, Pr  (where F = force, V = velocity , t = time , x = distance)
 2t 2 x
Check whether this equation may be dimensionally correct or not.
Solution: Dimension of L.H.S = [Pr] = M1L–1T–2
[3] [ F ] [v 2 ]  M L T   L T 
1 1 2 2 2

Dimension of R.H.S     M1L2T 6 


[ ] [t 2 ] [ x] T 2   L
Dimension of L.H.S and R.H.S are not same. So the relation cannot be correct.
1 2
Example 24: For the equation, s  ut  gt . Where symbols have their usual meaning.
2
Check whether this equation may be dimensionally correct or not.
Solution: [L] = [L T–1 T] + [L T–2 T2]
[L] = [L] + [L]
Hence, the equation is dimensionally correct.

5. As a research tool to derive new relations

One of the aims of scientific research is to discover new laws relating different physical quantities. The
theory of dimensions in the light of the principle of homogeneity provides us with a powerful tool of
research in the preliminary stages of any investigation.
If a physical quantity X depends on other physical quantities P, Q and R (say), then we may write
x  P a Qb R c
Then writing dimensional formula for X, P, Q and R and equating the dimensions on either sides give the
values of a, b and c. The substitution of these values in (1) given the new dimensional relation.

59
PHYSICS UNITS & DIMENSIONS

To illustrate this let us consider the following examples:


Example 25: Einstein Mass–Energy Relation
Find relation for energy produced, when mass is converted into energy. Let the energy produced
depend on the mass (m) and speed of light (c).
Solution: Assuming the function to be product of power functions of m and c,
E  Kmx c y …(1)
Where K is a dimensionless constant. If the above relation is dimensionally correct,
Then, [ML2T–2] = [M]x [LT–1]y
or [ML2T–2] = [MxLyT–y]
Equating the exponents of similar quantities, x = 1 and y = 2
So the required physical relation becomes, E  Kmc2
The value of dimensionless constant K is found unity through experiments, so, E  mc 2
This is the famous Einstein mass– energy relation.
Example 26: Stoke’s Law
When a small sphere moves at low speed through a fluid, the viscous force F, opposing the motion, is
found experimentally to depend on the radius r, the velocity of the sphere v and viscosity  of the
fluid.
Find relation for viscous force F using dimensional analysis.
Solution: If the function is product functions of , r and v,
F  k x r y v z ...(1)
where k is dimensionless constant.
[MLT–2] = [ML–1T–1]x [L]y [LT–1]z
[MLT–2] = [Mx L–x +y+z T–x–z]
Equating the exponents of similar quantities
x = 1; –x + y + z = 1 and –x – z = –2
Solving these for x, y and z, we get
So eqn. (1) becomes, F  k rv
On experimental grounds, k  6
so, F  6 rv

Example 27: Planck’s Length


Construct a new physical quantity having dimensions of length in terms of well known universal
constants G, c and h.
Solution: Let QL be the new physical quantity having dimensions of length.
If the function is the product of power functions of G, c and h,
QL  KG xC y h z …(1)
where K is a dimensionless constant of proportionality.
If the above relation is dimensionally correct,
 L   M 1L3T 2 
x y z
 LT 1   ML2T 1 
i.e.  L  M x z L3x y 2 z T2 x y  z 
Equating the exponents of similar quantities
 x  z  0, 3x  y  2z  1 and 2 x  y  z  0
Solving these for x, y and z, we get

60
UNITS & DIMENSIONS PHYSICS

1 3 1
x ; y and z 
2 2 2
So eqn. (1) becomes, QL  KG1/ 2 c 3/ 2 h1/ 2
Assuming constant K to be unity, QL  Gh / c3

Example 28: If time period of simple pendulum depends upon length of pendulum, mass & acceleration due to
gravity, derive relation for time period using methods of dimensions.
Solution: We can say that expression of T should be in this form
T = k (m)a (l)b(g)c
[M0L0T1] = (1) [M1]a [L1]b [L1T–2]c
[M0L0T1] = [Ma Lb+c T–2c]
Comparing the powers of M, L and T,
get a = 0, b + c = 0, – 2c = 1
so a = 0, b = , c =
so T = K M0 L1/2 g–1/2
l
T K
g
Example 29: The time of oscillation (T) depends upon the density ‘d’ radius ‘r’ and surface Tension (s).
Obtain the formula for T by dimensional method.
T  d  r  s
a b c
Solution:
 T  k d  r  s
a b c
....(1)
Taking dimension of each quantity in both sides.
[M0L0T1] = [M1L–3T0]a [L1]b [M1L3T–2]c
 [M0L0T1] = [Ma+c] [L–3a+b] [T–2c]
[M0L0T1] = [Ma+cL–3a + bT–2c]
comparing the dimensions of both sides
a+c=0 …(2)
–3a + b = 0 …(3)
–2c = 1 or c = –1/2 …(4)
Substituting value of c in equation (3)
 1 1
a    0  a
 2 2
1
Now putting a a  in equation (3)
2
1 3
 3    b  0  b
2 2
On substituting value a, b and c in equation (1) T  k  d  r  s
1/2 3/2 1/2

dr 3
 T
s

61
PHYSICS UNITS & DIMENSIONS

Example 30: A gas bubble from an explosion under water oscillates with a period T proportional to , where P
is the static pressure, d is the density of water and E is the total energy of explosion. Find the
values of a, b and c.
Solution: Given that, T  P a d b E c
Equating both sides dimensionally,
[T]  [MT 1T 2 ]a [ML3 ]b [ML2T 2 ]c
[M 0 L0 T 0 ]  [M a  b  c M  a 3b  2c T 2 a  2c ]
Equating the exponents of similar quantities
a  b  c  0,  a  3b  2c  0 and 2a  2c  1
5 1 1
Solving these for a, b, and c, we get, a   ; b  and c 
6 2 3
Example 31: If velocity, force and time are taken to be fundamental quantities find dimensional formula for
(a) Mass and (b) Energy
Solution: Let the quantity be Q then, Q  f ( , F , T )
Assuming that the function is the product of power functions of and T,
Q  K  x F yT z …(1)
where K is a dimensionless constant of proportionality.
[Q ]  [LT 1 ]x [MLT 2 ] y [T]z i.e. [Q ]  [M y Lx  y T  x  2 y  z ] …(2)
(a) Q = mass i.e. [Q] = [M]
So eqn. (2) becomes [M]  [M y Lx  y T  x  2 y  z ]
its dimensional correctness requires
y = 1, x + y = 0 and –x – 2y + z = 0
which on solving yields x = – 1; y = 1; z = 1
Substituting it in Eqn. (1), we get Q  k 1 FT
(b) Q = Energy i.e. [Q] = [ ML2 T–2]
So Eqn. (2) becomes [ML2T 2 ]  [M y Lx  y T  x  2 y  z ]
which in the light of principle of homogeneity yields
y  1, x  y  2 and  x  2 y  z  2
which on solving yields x  y  z  1
So eqn. (1) becomes Q  K FT

LIMITATION OF THEORY OF DIMENSIONS


Although dimensional analysis is very useful it cannot lead us too far as:
(1) If dimensions are given, physical quantity may not be unique as many physical quantities have same
dimensions.
For example, if the dimensional formula of a physical quantity is [ML2 T 2 ] it may be work or energy or torque.
(2) Numerical constants [K] having no dimensions such as (1/2), 1 or 2 etc, cannot be deduced by the
methods of dimensions.
(3) The method of dimensions cannot be used to derive relations other than product of power functions.
1
For example, s  ut    at (or) y  a sin t y  a log  x  ; y  aex etc.
2
(or)
2

62
UNITS & DIMENSIONS PHYSICS

cannot be derived by using this theory. However, the dimensional correctness of these can be checked.
(4) The method of dimensions cannot be applied to derive formula if in mechanics a physical quantity depends
on more than 3 physical quantities as then there will be less number (=3) of equations than the unknowns
(>3). However, still we can check correctness of the given equation dimensionally. For example,
T  2 I / mgL cannot be derived by theory of dimensions but its dimensional correctness can be
checked.
(5) Even if a physical quantity depends on 3 physical quantities, out of which two have same dimensions, the
formula cannot be derived by theory of dimensions, e.g., formula for the frequency of a tuning fork
f  (d / L2 ) cannot be derived by theory of dimensions but can be checked.
(6) Dimensional analysis does not tell whether a given physical quantity is a scalar or a vector.
(7) Dimensional analysis does not deal with quantitative analysis.
Example 32: Can Pressure (P), density () and velocity (v) be taken as fundamental quantities?
Solution: P,  and v are not independent, they can be related as P   v 2 ,so they cannot be taken as
fundamental variables.
To check whether the ‘P’, ‘’, and ‘V’ are dependent or not, we can also use the following
mathematical method:
[P] = [M1L-1T-2]
[] = [M1L-3 T0]
[v] = [M0L1T-1]
Check the determinant of their powers:
1 1 2
1 3 0 = 1 (3) – (–1) (–1) – 2 (1) = 0,
0 1 1
So, these three terms are dependent.
THEORY OF ERRORS
Errors in measurement
The measuring process is essentially a process of comparison. To measure any physical quantity, we
compare it with a standard (unit) of that quantity. No measurement is perfect as the errors involved in the
process connot be removed completely.
Hence inspite of our best efforts, the measured value of a quantity is always some way different from its
actual values, or true value. This difference in the true value and the measured value of a quantity is
called error of measurement.
Least Count
The minimum measurement that can be actually taken by an instrument is called the least count. The
least count of a metre scale graduated in millimetre mark is 1 mm. The least count of a watch having
second’s hand is 1 second.
Why error? Suppose we have to measure the length of a rod. How can we!
(a) Let us use a centimeter scale: (a scale on which only cm. marks are there)

We will measure length = 4 cm.

63
PHYSICS UNITS & DIMENSIONS

Although the length will be a bit more than 4, but we cannot say its length to be 4.1 cm or 4.2
cm., as the scale can measure upto cms only, not closer than that.
This scale can measure upto cms accuracy only.
So we’ll say that its least count is 1 cm.
(b) Lets use an mm scale: (a scale on which mm. marks are there)

We will measure length l = 4.2 cm., which is a more closer measurement. Here also if we
observe closely, we’ll find that the length is a bit more than 4.2, but we cannot say its length to
be 4.21, or 4.22, or 4.20 as this scale can measure upto 0.1 cms (1 mm) only, not closer than that.
This scale can measure upto 0.1 cm accuracy.
Its least count is 0.1 cm.
Max uncertainty in l can be = 0.1 cm
Max possible error in l can be = 0.1 cm
(c) We can use Vernier callipers: (which can measure more closely, upto 0.01 cm)
Then we’ll measure length l = 4.23 cm which is more closer measurement.
It can measure upto 0.01 cm accuracy
Least count = 0.01 cm
Max uncertainty in l can be = 0.01cm
Max possible error in l can be = 0.01cm
To get further more closer measurement:
(d) We can use Screw Gauge: (which can measure more closely, upto 0.001 cm)
We’ll measure length l = 4.234 cm.
Max possible uncertainty (error) in l can be = 0.001 cm
Least count of the instrument = 0.001 cm
Accuracy
The accuracy of a measurement is a measure of how close the measured value is to the true value of the
quantity. The accuracy of a measurements is also determined by the number of significant figures. The
larger is the number of significant figures, more accurate is the measurement.
The accuracy in measurement may depend on several factors like personal errors; imperfection in
technique/procedure; instrumental errors and errors due to external causes, random errors and so on. Poor
accuracy involves errors that can often be corrected. As we reduce the errors, measurement’s accuracy
increases.
Precision
The precision of a measurement is determined by the least count of a measuring instrument. The smaller
is the least count larger is the precision of the measurement.
Precision tells us to what resolution or limit the quantity is measured by a measuring instrument.
Precision describes the limitation of the measuring instrument. Infact, precision is determined by the least
count of the measuring instrument. Smaller the least count, greater is the precision. e.g. If the length of a
block is 4.37 cm and its measurement with a meter scale is l1 = 4.4 cm, while with vernier callipers is l1 =
4.43 cm, then first measurement is more accurate while the second measurement is more precise.
Order of magnitude
The order of magnitude of a physical quantity is that power of 10 which is closest to its magnitude. It
gives an idea about how big and how small a given physical quantity is?

64
UNITS & DIMENSIONS PHYSICS

A number N can be expressed as N = n  10x.


If 0.5 < n < 5, then x will be the order of magnitude of N.
Example 33: (a) Speed of light is 3  108 ms–1. What is its order of magnitude?
(b) Mass of electron is 9.1  10–31 kg. What is its order of magnitude?
Solution:
(a) Because N = 3  108
 n = 3 (0.5 < n < 5)
So, Order of magnitude is 8.
(b) Because N = 9.1  10–31 kg
 N = 0.91  10–30 kg
n = 0.91 (0.5 < n < 5)
So, Order of magnitude is –30.

Significant Figures

Definition: It is defined as the number of certain digits in a measurement plus the first uncertain digit.
e.g. If the length of an object is 6.53 cm, it has three significant figures. Here, 6.5 is reliable while the
third digit 3 is uncertain.
Common rules for counting significant figures in a reported measurement :
Rule 1. All non zero digits in a measurement are significant.
For example: x = 7635 m has four significant figure. Again m = 126 kg has only three significant
digits.
Rule 2. All zeros occurring between two non zero digits in a measurement are significant no matter
where the decimal point is, if at all.
For example: m = 1009 kg has four significant figures. Again t = 1.0801 s has five significant
figures.
Rule 3. If the measurement is less than one, the zero (s) on the right of decimal point and to the left of
first non– zero digit are not significant.
For example: in 0.005704 m, the underlined zeros are not significant. The zero between 7 and 4
is significant. The number of significant figures is 4.
Rule 4. In a measurement without a decimal point, the terminal or trailing zeros are not significant.
For example, l =1620 metre has three significant figures, the trailing zero being not significant.
Rule 5. However, in a measurement with decimal point the trailing zeros are significant.
For example, l = 1.600 m has four significant figure. Similarly, t = 0.00013000 s has five
significant figures. The underlined zeros are not significant as per Rule 3.
Rule 6. A choice of change of different units does not change the number of significant figures in a
measurement.
For example, the length l = 5.608 cm has four significant digits. In different units, the same
length can be written as l = 56.08 mm, l =0.05608 m; l = 0.00005608 km; l = 56080 m. All
these numbers have the same numbers of significant figures, namely four, the digits 5,6,0 and 8.
This shows that the location of decimal point is of no consequence in determining the number of
significant figures.
Rule 7. The digits 0 conventionally put on the left of a decimal for a measurement less than 1 is never
significant.
For example, l = 0.3570 m has four significant digits. The first zero before decimal point is not
significant.

65
PHYSICS UNITS & DIMENSIONS

Example 34: Count total number of significant figures in 3.0800 m.


Solution: Significant figures = Five, as trailing zeros after decimal place are significant.
Example 35: Count total number of significant figures in 0.00418 kg.
Solution: Significant figures = Three, as leading zeros are not significant.
Example 36: Count total number of significant figures in 3500 s.
Solution: Significant figures = Two, the trailing zeros are not significant.
Example 37: Count total number of significant figures in 300.00 m.
Solution: Significant figures = Five, trailing zeros after decimal point are significant.
Example 38: Count total number of significant figures in 5.003020 J.
Solution: Significant figures = Seven, the trailing zeros after decimal place are significant.
Example 39: Count total number of significant figures in 6.020 × 1023 articles per mole.
Solution: Significant figures = Four; 6, 0, 2, 0 ; remaining 23 zeros are not significant.
Example 40: Count total number of significant figures in 1.60 × 10–19 C.
Solution: Significant figures = Three; 1, 6, 0; remaining 19 zeros are not significant.

Rounding Off
The result of computation with approximate measurements, which contain more than one uncertain digit,
should be rounded off. While rounding off measurements, we use the following rules by convention:
Rule 1. If the digit to be dropped is less than 5, then the preceding digit in left remains unchanged.
For example, is rounded off to 7.8 m. Again, is rounded off to 3.9m.
Rule 2. If the digit to be dropped is greater than 5, then the preceding digit is raised by one.
For example, m = 6.88 kg is rounded off to 6.9 kg. Again, t = 12.78 s is rounded off to 12.8 s.
Rule 3. If the digit to be dropped is 5 following by digits other than zero, then preceding digit is raised by one.
For example, m = 16.351 kg is rounded off to 16.4 kg.
Again l = 6.758 m is rounded off to 6.8 m.
Rule 4. If the digit to be dropped is 5 or 5 followed by zeros then the preceding digit remains unchanged,
if it is even.
For example, t = 3.250 s becomes 3.2 s on rounding off, Again l = 12.650 m becomes 12.6 m on
rounding off.
Rule 5. If the digit to be dropped is 5 or 5 followed by zeros, then the preceding digit is raised by one, if
it is odd.
For example, t = 3.750 s is rounded off to 3.8 s. Again, m =16.150 kg is rounded off to 16.2 kg.

Note:
1. For calculations, a number known accurately to many significant digits can be rounded
off to an approximate value. For example, speed of light in vacuum is c = 2.99792458 
108 m/s. It is rounded off to c = 3  108 m/s.
2. The value of   3.1415926 is known to a large number of significant figures,
However, in calculations, we may take   3.142 or 3.14, as per our requirement.

Rules for algebraic operations

(a) Addition and Subtraction: The sum or difference of two measurements has significant figures only
in those places where these are in the least precise amongst the given measurements. e.g. when we

66
UNITS & DIMENSIONS PHYSICS

add 3.213 cm, 40.5 cm and 2.0123 cm, we get 45.7253 cm. But the least precise of the given
measurements is 40.5 cm, therefore, the final answer must be 45.7 cm.
(b) Multiplication and division: The product or the division of two measurements does not have more
significant figures than are present in the least precise of the given measurement. e.g., when we
multiply 4.08 m3 with 16 kg m–3, we get 65.28 kg. But the least precise of the two given
measurements is 16 kg m–3 , therefore, the final answer is 65 kg.

TYPES OF ERRORS
The errors in measurement can be broadly classified as
1. Systematic Errors 2. Random Errors 3. Gross Errors

(a) Systematic Errors

The systematic errors are those errors that tend to be in one direction, either positive or negative. Infact,
the causes of systematic errors are known. Therefore, such errors can be minimised.
Some of the sources of systematic errors are:
(i) Instrumental errors
(ii) Imperfection in experimental technique or procedure.
(iii) Personal errors
(iv) Error due to external causes.
(v) Zero Error

(b) Random Errors


The random errors are the errors which occur irregularly. They are random with respect to sign and size,
i.e. sometimes, random errors may be positive and the other time, they may be negative. Similarly,
sometimes, the random errors may be big and the other time, they may be small.
The random errors may arise due to random and unpredictable variations in experimental conditions e.g.
temperature, pressure, voltage supply, mechanical vibrations etc. These may also be due to personal
errors by the observer doing the experiment.
These errors can be minimised by performing the experiment large number of times.

(c) Gross Errors

These errors arise on account of shear carelessness of the observer. For example:
(i) Reading an instrument without setting it properly.
(ii) Taking the observations wrongly without caring for the sources of errors and the precautions.
(iii) Recording the observations wrongly.
(iv) Using wrong values of the observations in calculations.
These errors can be minimised only when the observer is sincere and mentally alert.

Permissible Error or least count error


The permissible error is the error which arises an account of limitations on the measuring abilities of
various instruments used in an experiment. This error persists even when we carefully avoid gross errors
and select the most accurate instruments for measurement.
For example suppose length of an object is measured with a metre rod with least count equal to 0.1 cm. If
the measured length is 62.5cm, it has to be recorded as (62.5  0.1) cm stating the limits of error.
Similarly, suppose time period of a pendulum is measured to be 2.0 s, using a stop watch of least count

67
PHYSICS UNITS & DIMENSIONS

0.1 s, it has to be recorded as (2.0  0.1) s, meaning thereby that time period is in the range of 1.9 s, and
2.1 s.

ABSOLUTE ERROR, RELATIVE, ERROR AND PERCENTAGE ERROR


(a) Absolute error
Absolute error in the measurement of a physical quantity is the magnitude of the difference between the
true value and the individual measured value of the quantity.
Let a physical quantity be measured n times.
Let the measured values be a1 , a2 , a3 .......an . The arithmetic mean of these values is
a1  a2  ......an
am  …(1)
n
in
1
or am   ai …(2)
n i 1
Usually, the arithmetic mean am is taken as the best possible/ true value of the quantity, if the same is not
known otherwise.
By definition absolute errors in the individual measured values of the quantity are
 a1  am  a 1
 a2  am  a 2
.......................
 an  am  a n
The absolute errors may be positive in certain cases and negative in certain other cases. This is because
individual measurements are as likely to overestimate as to underestimate the true value of the quantity.

(b) Mean absolute error


It is the arithmetic mean of the magnitudes of absolute errors in all the measurements of the quantity. It is
represented by  amean . Thus
| a1 |  | a2 | ....| an |
 amean 
n
in
1
 amean    | ai | …(3)
n i 1
Hence the final result of measurement may be written as a  am  amean
This implies that any measurement of the quantity is likely to lie between (am   ameam ) and
(am   ameam ) .

(c) Relative error or Fractional error

The relative error or fractional error of measurement ( a ) is defined as the ratio of mean absolute to the
mean value of the measured quantity.
Mean absolute error amean
Thus relative error or fractional error is,  a  
mean value am
When the relative/Fractional error is expressed in percentage, we call it percentage error.
amean
Thus, Percentage error,  a  100   100%
am

68
UNITS & DIMENSIONS PHYSICS

PROPAGATION OR COMBINATION OF ERRORS


To calculate the net error in the results, we should study how errors propagate in different mathematical
operations. We discuss below the propagation of errors in the five different mathematical operations:
(a) Error in a sum
Suppose x  a  b
Therefore, the maximum absolute error in x is,  x   (a  b)
Hence maximum absolute error in sum of the two quantities is equal to sum of the absolute errors in
the individual quantities.
(b) Error in a difference
Suppose x  a  b
Therefore, the maximum absolute error in x is, x   (a  b)
i.e. maximum absolute error in difference of two quantities is equal to sum of the absolute error in
the individual quantities.
(c) Error in a product
Suppose x  a  b
x  a b 
Hence maximum possible values of     
x  a b 
Hence maximum fractional error or relative error in product of quantities is equal to sum of the
fractional or relative errors in both individual quantities.
(d) Error in a quotient
a
Suppose x 
b
x  a b 
Therefore, the maximum value of     
x  a b 
Hence the maximum values of fractional or relative error in division of quantities is equal to sum of
the fractional/relative errors in the individual quantities.

(e) Error in an equation

an
Suppose x 
bm
x  a b 
  n m  …(1)
x  a b 
It follows from (1) that fractional error or relative error in a quantity raised to power (n) i.e. n should
be measured with highest degree of accuracy i.e., with least error.
al bm
In general, suppose X 
cn
X  a   b   c 
  l  m  n  
X  a   b   c 

69
PHYSICS UNITS & DIMENSIONS

Example 41: The error in the measurements of radius of the sphere is 0.3%. What is the permissible error in its
surface area.
Solution: Surface area of sphere A = 4r2
There is no error involved in constant 4.
Fractional error = 2 r
r
A  r 
% error = 100  2   100   2  0.3  0.6%
A  r 
Example 42: The length and breadth of rectangle are (5.7 ± 0.1) cm and (3.4 ± 0.2) cm. Calculate area of the
rectangle with error limits.
Solution: Here, l = (5.7 ± 0.1) cm; b = (3.4 ± 0.2) cm
Area, A = l × b = 5.7 × 3.4 = 19.38 cm2 = 19.0 cm2
(rounding off to two significant figures)
A  l b   0.1 0.2   0.34  1.14 
          
A  l b   5.7 3.4   5.7  3.4 
A 1.48 1.48
  A   A
A 19.38 19.38
1.48
 19.38  1.48 cm2
19.38
A = ± 1.5 cm2 (rounding off to two significant figures)
 Area = (19.0 ± 1.5) cm2
Example 43: Two resistors of resistances R1 = (100  3) ohm and R2 = (200  4) ohm are connected
(a) in series, (b) in parallel. Find the equivalent resistance of the (a) series combination (b)
parallel combination.
Solution:
(a) The equivalent resistance of series combination
R S  R1  R 2  (100  3)ohm  (200  4)ohm  300  7ohm
(b) The equivalent resistance of parallel combination
R1 R2 200
RP    66.7ohm
R1  R2 3
RP  R R R 
   1  2  S 
RP  R1 R2 RS 
 R R R 
RP    1  2  S  RP
 R1 R2 RS 
 3 4 7   28 
RP        66.7       66.7   6.23 
 100 100 300   300 
So, RP   66.7  6.23 

70
UNITS & DIMENSIONS PHYSICS

Example 44: It is claimed that two cesium clocks, if allowed to run for 100 years, free from any disturbance,
may differ by only about 0.02s. What does this imply for the accuracy of the standard caesium
clock in measuring a time interval of 1s?
Solution: Time interval of 100 y = (100  365.25  24  60  60)s = 3155760000s
Difference in the two clocks = 0.02s
Measured time interval in seconds = (3155760000  0.02)s
Measured time interval in seconds = 3155760000.02 or 3155759999.98s
Both these values of measured time contain 12 significant figures and as such the accuracy of
standard caesium clock is 1 part in 1011 to 1012.
Example 45: The length, breadth and thickness of a rectangular sheet of metal are 4.234m,1.005m, and 2.01
cm respectively. Give the area and volume of the sheet to correct significant figures.
Solution: Length of the sheet, l = 4.234 m
Breadth of the sheet, b = 1.005m
Thickness of the sheet t = 2.01 cm = 0.0201m
Area of the sheet = 2(l  b + b  t + l  t)
= 2 (4.234  1.005 + 1.005  0.0201+4.234  0.0201) m2
= 2(4.25517 + 0.0202005 + 0.0851034) m2 = 8.72 m2 (rounded off to three significant figures)
Volume of the sheet = lbt = (4.234  1.005  0.0201) m3
= 0.0855289 m3 = 0.0855 m3 (rounded off to three significant figures)
Note that the number of significant figures (three in both the cases) in the result cannot be greater than
the number of significant figures (three) in the least accurate factor (0.0201m) in the calculations.
Example 46: The mass of a box measured by a grocer’s balance is 2.3 kg. Two gold pieces of masses 20.15 g
and 20.17 g are added to the box. What is: (a) the total mass of the box (b) the difference in the
masses of the pieces to correct significant figures?
Solution: Mass of the box = 2.3 kg
Mass of one gold piece = kg = 0.02015 kg
Mass of the other gold piece = 0.02017kg
(a) Total mass of the box = 2.3 kg + 0.02015 kg + 0.02017 kg
= 2.34032 kg = 2.3 kg
(b) Difference in the masses of the pieces = 20.17g – 20.15 g = 0.02 g
Example 47: The diameter of a ball was measured five times with the aid of a micrometer whose absolute
error (dinst) = 0.01 mm. The results of measuring the diameter of the ball are d1 = 5.27 mm, d2 =
5.30mm, d3 = 5.28 mm, d4 = 5.32 mm and d5 = 5.28 mm. Find (i) mean value of ball diameter (ii)
mean absolute error (iii) result of measurement (iv) relative error (v) percentage error. Also,
express the result in terms of percentage error.
Solution:
d1  d2  d3  d4  d5
(i) The mean value of the ball diameter, i.e. dm   5.29 mm
5
(ii) The absolute errors in the measurements are
d1  dm  d1  0.02 mm
d2  dm  d2  0.01 mm
d3  dm  d3  0.01 mm
d4  dm  d4  0.03 mm

71
PHYSICS UNITS & DIMENSIONS

d5  dm  d5  0.01 mm
d1  d2  d3  d4  d5
Mean absolute error, dmean   0.02 mm
5
(iii) Since the mean absolute error (dmean) is greater than the instrumental error (dinst), the result of
measurement is
d  dm  dmean   5.29  0.02 mm
(As a rule, we take either dmean or dinst, depending upon which of these errors is greater)
d mean 0.02
(iv) Relative error,   0.04
dm 5.29
d mean
(v) Percentage error, 100  0.04 100  4%
dm
value of d expressed in terms of percentage error = (5.29  4%) mm
Example 48: A physical quantity P is related to four observables a, b, c and d as follows:
a 3b 2
P . The percentage errors of measurement in a, b, c and d are 1%, 3%, 4% and 2%
cd
respectively. What is the percentage error in the quantity P? If the value of P calculated using the
above relation turns out to be 3.763, to what value should you round off the result?
a 3b 2
Solution: We are given that, P 
cd
Applying the formula for the combination of errors,
P a b 1 c d
3 2  
P a b 2 c d
a 1 b 3 c 4 d 2
As  ,  ,  ,  ,
a 100 b 100 c 100 d 100
P 1 3 1 4 2 13
 3  2    
P 100 100 2 100 100 100
P
Thus, percentage error in P  100
P
13
 100  13%
100
The result should be rounded off to 3.8 as there is uncertainty even in regard to the second
significant figure, i.e. 7 in 3.763.

72
UNITS & DIMENSIONS PHYSICS

POINTS TO BE REMEMBERED
1. The absolute error in each measurement is equal to the least count of the measuring instrument.
2. The unit and dimensions of the absolute error are same as that of quantity itself.
3. Absolute error is not dimensionless quantity.
4. Relative error is dimensionless quantity.
5. Smaller the least count, higher is the precision of measurement.
6. Larger the number of significant figures after the decimal in a measurement, higher is the precision
of measurement.
7. Significant figures do not change if we measure a physical quantity in different units.
8. Significant figures are the number of digits upto which we are sure about their precision.
9. If a measurement is without a decimal and ends in one or more zeros, then all the zeros at the end of
the measurement may not be significant. To make the number of significant figures clear, it is
suggested that the measurement may be written in exponential form. For example: 20300 may be
expressed as 203.00 × 102, to suggest that all the zeros at the end of 20300 are significant.
10. 1 inch = 2.54 cm
11. 1 foot = 12 inches = 30.48 cm
12. 1 yard = 0.9144 m
13. 1 slug = 14.59 kg
14. 1 barn = 10–28 m2
15. 1 litre = 103 cm3 = 10–3 m3
5
16. 1 km/h = m/s & 1 m/s = 3.6 km/h
18
17. 1 g/cm3 = 1000 kg/m3
18. 1 atm. = 76 cm of Hg = 1.013 × 105 N/m2 & 1 N/m2 = Pa
19. When we add or subtract two measured quantities, the absolute error in the final result is equal to the
sum of the absolute errors in the measured quantities.
20. When we multiply, or divide two measured quantities, the relative error in the final result is equal to
the sum of the relative errors in the measured quantities.
21. A solar day is the time interval between successive appearances of the Sun overhead.
22. Do not use the slang SI system as the word system is contained in SI itself.
23. A cesium clock was adopted in 1999 whose variation is less than one second in 20 million years.
24. A very precise clock of least count 0.00001 s, the all - optical atomic clock is under development.
25. The standard platinum metre bar fabricated was shorter by 1.8  10–4 m than the 10–7 part of a
quadrant of the terrestrial meridian.
26. A tropical year is defined as the time interval between two successive passages of the earth through
the vernal equinox.
27. The period of earth’s rotation and hence the length of a day is increasing at the rate of 7s per year.
28. A sidereal day is about 4 minute shorter than the solar day.
29. Significant figures are lost in calculations involving difference of two close measurements.

73
PHYSICS UNITS & DIMENSIONS

CLASS ASSIGNMENT

Questions based on Measurement of Mass, Length & Time


1. How much longer than a microsecond is a millisecond?
2. The radius of gold nucleus is 41.3 fermi. Express its volume in m3.
3. The shadow of a pole standing on a level ground is found to be 45 cm longer when the sun’s altitude is
30 than when it was 60. Determine the height of pole.
4. A laser light beamed at the moon takes 2.56 s to return after reflection at the moon’s surface. How much
is the radius of the lunar orbit around the earth?
5. What is the distance in km of a quasar from which light takes 3.0 billion years to reach us?
6. In a submarine equipped with SONAR, the time delay between generation of a probe wave and the
reception of the echo after reflection from an enemy submarine is found to be 77.0s. What is the distance
of the enemy submarine? Speed of sound in water is 1450 ms–1.
7. The parallax of a heavenly body measured from two points diametrically opposite on equator of earth is
1.0 minute. If the radius of earth is 6400 km, find the distance of the heavenly body from the centre of
earth in AU, taking 1AU = 1.5  1011m.
8. The moon is observed from two diametrically opposite points A and B on earth. The angle  subtended
at the moon by the two directions of observation is 1054. Given that the diameter of earth is about 1.276
 107m, calculate the distance of moon from earth.
9. The sun’s angular diameter is measured to be 1920. The distance of the sun from the earth is 1.496 
1011m. What is the diameter of the sun?
10. A man wishes to estimate the distance of a nearby tower from him. He stands at a point A in front of the
tower C and spots a very distant object O in line with AC. He then walks perpendicular to AC upto B, a
distance of 100 m and looks at O and C again. Since O is very distant, the direction of BO is practically
the same as AO, but he finds the line of sight of C shifted from the original line of sight by an angle  =
400 ( is known as parallax). Estimate the distance of the tower C from his original position A.
11. One gram mole of an ideal gas at NTP occupies 22.4 litres (molar volume). What is the ratio of molar volume
to atomic volume of hydrogen? Take size of hydrogen molecule to be 1 Å. Why is the ratio so large?
12. Calculate average atomic mass density of a sodium atom, assuming its size to be 2.5Å. Compare it with
density of sodium in its crystalline phase (970 kg m–3). Are the two densities of the same order of magnitude?
13. The sun is a hot plasma (ionized matter) with its inner core at a temperature exceeding 10 7 K, and its
outer surface at a temperature of about 6000 K. At such high temperatures, no substance remains in a
solid or liquid phase. In what range do you expect the mass density of the sun to be? In the range of
densities of solids, liquids or gases? Check if your guess is correct from the following data: mass of sun
= 2.0  1030 kg; radius of the sun = 7.0  108m.
14. The unit of length convenient on nuclear scale is a fermi, 1 f = 10–15 m. Nuclear sizes obey roughly the
following empirical relation. r = r0 A1/3, where r is the radius of the nucleus and r0 is a constant equal to 1.2f.
Show that the rule implies that nuclear mass density is constant for different nuclei. Estimate the mass density
of the sodium nucleus. Compare it with average mass density of sodium atom (4.67  103 kg/m3).
15. The photograph of a house occupies an area of 1.75 cm2 on a 35 mm slide. The slide is projected on to a
screen, and the area of the house on the screen is 1.55 m2. What is the linear magnification of the
projector screen arrangement?
16. A student measures the thickness of a human hair by looking at it through a microscope of magnification
100. He makes 20 observations and finds that the average width of the hair in the field of view of the
microscope is 3.5 mm. What is his estimate on the thickness of hair?

74
UNITS & DIMENSIONS PHYSICS

17. A drop of olive oil of radius 0.25 mm spreads into a circular film of diameter 20 cm on the water surface.
Estimate the size of an oil molecule.
Questions based on Dimensional Analysis

 
18. (a) In the following equation: 2
= Fv+ 2 , find dimensional formula for [] and []. Here t = time, F =
t x
force, v = velocity & x = distance.
(b) Write the dimensions of a/b in the relation F  a x  bt 2 where F is force, x is distance and t is time.
a  t2
(c) Write the dimensions of a/b in the relation P  where P is pressure, x is distance and t is time.
bx
F
19. (a) In the equation,  = sin (t), find the dimension of  and Here V = velocity, F = force & t = time.
V2
(b) Find the dimensions of  and k in equation.
y  a sin t  kx  where t is time, y, x and a represent distance.
(c) A book with many printing errors contains four different formulae for the displacement y of a particle
undergoing a certain periodic motion:
2 t a t a  2 t 2 t 
(i) y  a sin (ii) y  a sin vt (iii) y  sin (iv) y  sin T  cos T 
T T a 2  
Here, a is maximum displacement of particle, v is speed of particle, T is the time period of motion. Rule
out the wrong formulae on dimensional grounds.
x2  
20. The force of interaction between two atoms is given by F   exp    ; where x is the distance, k
  kt 
is the Boltzmann constant and T is temperature and  and  are two constants. The dimension of  is:
[JEE (M) 2019]
(A) M2L2T–2 (B) M2LT–4
(C) M0L2T–4 (D) MLT–2
 n2  n1 
21. In the formula; N = – D   , D = diffusion coefficient, n1 and n2 is number of molecules in unit
 x2  x1 
volume at distances x1 and x2along X-axis. N is number of molecules passing through per unit area per
unit time. Calculate dimensional equation of D.
22. The centripetal force F acting on a particle moving uniformly in a circle may depend upon mass (m),
velocity (v) and radius (r) of the circle. Derive the formula for F using the method of dimensions.
23. If time period of simple pendulum depends upon length of pendulum, mass & acceleration due to
gravity, derive relation for time period using methods of dimensions.
24. Expression for time in terms of G (universal gravitational constant), h (Planck constant) and c (speed of
light) is proportional to: [JEE (M) 2019]
Gh hc5
(A) (B)
c3 G
c3 Gh
(C) (D)
Gh c5

75
PHYSICS UNITS & DIMENSIONS

25. A gas bubble from an explosion under water oscillates with a period T proportional to P a d b E c , where P is the
static pressure, d is the density of water and E is the total energy of explosion. Find the values of a, b and c.
26. Construct a new physical quantity having dimensions of length in terms of well known universal
constants G, c and h.
27. Using the method of dimensional analysis, derive an expression for rate of flow (V) of a liquid of
coefficient of viscosity  flowing through a pipe of length l, radius r under a pressure difference P.
28. Reynold number (R) determines whether or not the flow of a liquid through a pipe is streamlined. This
constant is a combination of velocity (), density () and coefficient of viscosity (). Given that R varies
directly as diameter D of the pipe, derive the formula for R using the method of dimensions.
1
29. Derive dimensionally, the relation s  ut  at 2 , where the symbols have their usual meanings.
2
30. Can Pressure (P), density (  ) and velocity (v) be taken as fundamental quantities?
31. Using dimensional analysis, check the accuracy of the following relations:
a
(i) Snth  u   2n  1 (ii)   h / mv (iii) E = mc2
2
32. The escape speed from the surface of earth is given by v  2GM , where M is mass and R is the radius
R
of earth. Check the correctness of the formula.
2 cos 
33. Check the correctness of the relation h  , where h is height,  is surface tension,  is angle of
r 2 dg
contact, r is radius, d is density and g is acceleration due to gravity.

34. Check by the method of dimensions, the correctness of formula v  1 K , where v is the velocity of
 d
longitudinal waves,  is wavelength of wave, K is the coefficient of volume elasticity and d is the
density of the medium.
35. Convert the value of gravitational constant G  6.67 108 cm3 /g s 2 from CGS to MKS System.
36. Convert an energy of one joule into erg.
37. Find the value of a force of 100 N on a system based upon the metre, the kilogram and the minute as the
fundamental units.
38. If the units of force, energy and velocity are 10N, 100 J and 5 ms–1, find the units of length, mass and time.
39. If the unit of force were kilonewton, that of time millisecond and that of power kilowatt, what would be
the units of mass and length?
Questions based on Error Analysis
40. The length, breadth and thickness of a rectangular sheet of metal are 4.234m, 1.005m, and 2.01 cm
respectively. Give the area and volume of the sheet to correct significant figures.
41. The mass of a box measured by a grocer’s balance is 2.3 kg. Two gold pieces of masses 20.15 g and
20.17 g are added to the box. What is: (a) the total mass of the box (b) the difference in the masses of the
pieces to correct significant figures?
42. It is claimed that two cesium clocks, if allowed to run for 100 years, free from any disturbance, may
differ by only about 0.02s. What does this imply for the accuracy of the standard cesium clock in
measuring a time interval of 1s?
43. The diameter of a ball was measured five times with the aid of a micrometer whose absolute error (dinst) =
0.01 mm. The results of measuring the diameter of the ball are d1 = 5.27 mm, d2= 5.30mm, d3 = 5.28 mm,

76
UNITS & DIMENSIONS PHYSICS

d4 = 5.32 mm and d5 = 5.28 mm. Find (i) mean value of ball diameter (ii) mean absolute error (iii) result of
measurement (iv) relative error (v) percentage error. Also, express the result in terms of percentage error.
44. We measure the period of oscillation of a simple pendulum. In successive measurements, the readings
turn out to be 2.63 s, 2.56 s, 2.42 s, 2.71 s and 2.80 s. Calculate relative error and percentage error.
45. Two rods have lengths measured as (1.8  0.2)m and (2.3  0.1)m . Calculate their combined length
with error limits.
46. The original length of a wire is (153.7  0.6)cm . It is stretched to (155.3  0.2)cm . Calculate the
elongation in the wire with error limits.
47. Two resistors of resistances R1  100  3 ohm and R 2   200  4 ohm are connected (a) in series,
(b) in parallel. Find the equivalent resistance of the (a) series combination (b) parallel combination.
48. Calculate the focal length (f) of a spherical mirror from the following calculations: u = (50.1  0.5) cm,
v = (20.1  0.2) cm.
49. The radius of a sphere is measured with an error of 2%. What would be the error in volume of the sphere?
l2 h
50. The radius of curvature of a concave mirror, measured by a spherometer, is given by R =  . The
6h 2
value of l and h are 4.0 cm and 0.065 cm respectively, where l is measured by a meter scale and h by a
spherometer. Find the relative error in the measurement of R.
51. The percentage errors in quantities P, Q, R and S are 0.5%, 1%, 1.5% and 5.5% respectively in the
P 2Q 2
measurement of a physical quantity A  . The maximum percentage error in the value of A will
RS
be: [JEE (M) 2018]
(A) 6.5 % (B) 8.5 %
(C) 6.0 % (D) 7.5 %
52. A person measures the depth of a well by measuring the time interval between dropping a stone and
receiving the sound of impact with the bottom of the well. The error in his measurement of time is
T = 0.01 seconds and he measures the depth of the well to be L = 20 meters. Take the acceleration due
to gravity g = 10ms–2 and the velocity of sound is 300 ms–1. Then the fractional error in the measurement
L is closest to: [JEE (M) 2017]
(A) 0.2% (B) 3%
(C) 5% (D) 1%
53. In the given set of measurements which parameter is to be measured with extra or utmost care?
54. Calculate the relative error in the measurement of x = 11n.
55. The following observations were made during an experiment to find acceleration to gravity using simple
pendulum of length 90 cm. The time for 20 oscillations is 36.0 s. Length is being measured to an accuracy of
0.1 cm and time is measured with a watch of resolution 0.2 s. Find percentage error in the measurement of g.
56. The diameter of a circle is 1.06cm. Calculate area of circle with due regard to significant figures.
57. Why do we have different units for the same physical quantity?
58. The volume of sphere is 1.76 m3. What is the volume of 25 such spheres upto correct significant figures?
59. What is difference between voltmetre and voltmeter?
60. To estimate the size of a molecule we cannot use optical microscope. Why?
61. The length and breadth of a rectangle are measured as l  l   6.0  0.1 m , b  b   4.0  0.1 m .
Calculate the perimeter with error limits.

77
PHYSICS UNITS & DIMENSIONS

HOME ASSIGNMENT

Questions based on Basics of Measurement


Conceptual / Subjective Problems
1. What is the difference between mN, Nm and nm?
2. The numeric value of a quantity is n1 when expressed with unit u1. What is its numeric value for another
unit which is x times large.
3. The energy of a photon is given by E = hv. If for a photon wavelength = 6000Å, find its energy in eV.
Given 1eV = 1.6  10–19 J & h = 6.6  10–34 J/s.
4. What is the Gaussian system of units?
5. Is the definition of a physical quantity for which no method of measurement is known, has some meaning?
6. Why is the need of selecting some units as fundamental units?
7. Explain this statement clearly:
To call a dimensional quantity large or small is meaningless without specifying a standard for
comparison. In view of this, reframe the following statements wherever necessary:
(a) atoms are very small objects
(b) a jet plane moves with great speed
(c) the mass of Jupiter is very large
(d) the air inside this room contains a large number of molecules
(e) a proton is much more massive than an electron
(f) the speed of sound is much smaller than the speed of light.
8. S.I has broader base. Comment on the statement.
9. Give two advantage of S.I system.
10. Define unit. What is the need of selecting some units as fundamental units?
11. What is the difference between mN, Nm and nm?
Objective Problems
12. Standard unit should not be
(A) well defined (B) change with time
(C) easily accessible (D) easily reproducible
13. The unit for nuclear dose given to a patient is
(A) Fermi (B) Rutherford
(C) Curie (D) Roentgen
14. For which of the following physical quantity no method of measurement is known?
(A) Force (B) Gravitational Force constant
(C) Velocity (D) Entropy
15. Assertion: Mass, length and time are fundamental quantities
Reason: Mass, length and time are independent of one another.
(A) if both assertion and reason are true and the reason is correct explanation of the assertion
(B) If both assertion and reason are true, but reason is not the correct explanation of the assertion
(C) If assertion is true, but the reason is false
(D) If assertion is false, but the reason is true

78
UNITS & DIMENSIONS PHYSICS

Questions based on Measurement of length


Conceptual / Subjective Problems

16. How many astronomical units (A.U.) make 1 parsec?


17. The earth – moon distance is about 60 earth radius. What will be the diameter of the earth (in degrees) as
seen from the moon?
o
o 1
18. Angular diameter of the earth as seen from the moon is 2 . Moon is seen to be of   diameter from the
2
earth. Compare the diameter of the moon and the earth.
19. During a complete solar eclipse, the moon almost entirely covers the sphere of the sun. Write the relation
between the distances and sizes of the sun and the moon.
20. Estimate the diameter of a thread if you have a metre scale.
21. Suggest a way to measure:
(a) the thickness of a sheet of paper, (b) the distance between the sun and the moon.
22. What is the advantage in choosing the wavelength of light radiation as a standard of length?
23. How many meters are there in one X–ray unit?
24. Which method is used for measuring the distance of a star at a distance more than 100 light years away
from earth?
25. What is difference between 1AU and 1Å?
26. How many light years are there in one meter?
27. The radius of gold nucleus 41.3 fermi. Express its volume in m3.
28. The radius of atom is of the order of 1 Å and radius of nucleus is of the order of fermi. How many
magnitudes higher is the volume of atom as compared to the volume of nucleus?
29. Density of iron is  kg/m3 and mass of an iron atom is m kg. If the atoms are spherical and tightly
packed, find the distance between the centres of adjacent atoms.
30. For a given base line will a distant star or a nearby star, show greater parallax?
31. It is convenient to express distance of stars in terms of light year rather than in kilometer. Why?
32. Which of the two is greater in magnitude –number of seconds in a day or number of A.U. in a light year?
33. How can we find the distance of the moon by parallax method?
34. Parallax of a heavenly body measured from two points diametrically opposite on equator of earth is 2.0
minute. Calculate the distance of heavenly body in light years. Given radius of earth is 6400 Km. What is
the limitation of this method?
35. What do you mean by parallax? The Sun’s angular diameter is measured to be 1920”. The distance D of
the Sun from the earth is 1.496 × 1011 m. What is the diameter of the sun in AU?
36. Describe a method to measure the size of an oleic acid molecule.
37. Define parallax. Explain this common observation clearly. “If you look out of the window of a fast moving
train, the nearby trees, houses etc. seem to move rapidly in a direction opposite to the train’s motion, but the
distant objects (hill tops, the moon, the starts etc) seem to be nearly stationary.”
38. Explain how you will measure distance of a nearby star.
Objective Problems
39. One Mach number is equal to
(A) Velocity of light (B) Velocity of sound 332 m/sec
(C) 1 km/sec (D) 1 m/sec

79
PHYSICS UNITS & DIMENSIONS

40. Assertion: Å and AU are different units of length


Reason: Å (angstrom) is small unit of length. While AU is big unit of length
(A) if both assertion and reason are true and the reason is correct explanation of the assertion
(B) If both assertion and reason are true, but reason is not the correct explanation of the assertion
(C) If assertion is true, but the reason is false
(D) If assertion is false, but the reason is true
41. Which method is used for measuring the distance of a star at a distance less than 100 light years away from
earth?
(A) Kepler’s law of periods (B) echo method
(C) Parallax method (D) Shadow method
42. Find the value of one light year in giga meter.
(A) 9.46  105 (B) 9.46  106
(C) 9.46  107 (D) 9.46  108
43. Which method is best suitable to measure the thickness of a sheet of paper?
(A) laser method (B) screw Gauge
(C) Sonar method (D) parallex method
44. A new unit of length is chosen such that the speed of light in vacuum is unity. What is the distance between
the Sun and the Earth in terms of the new unit if light takes 8 min and 20 s to cover this distance?
(A) 200 (B) 300
(C) 400 (D) 500
45. Angular diameter of moon is
(A) 0.6o (B) 0.5o
(C) 0.7o (D) 0.8o
46. Red corpuscles of human blood stream are known to be flattened discs. Blood count shows RBCs of the order
of 5  106 in each cubic millimetre of blood, what is the order of total number of red corpuscles it contains?
(A) 108 (B) 1011
(C) 1010 (D) 1013

Questions based on Measurement of mass


Conceptual / Subjective Problems
47. Define atomic mass unit (a.m.u). or Express unified atomic mass unit in kg.
48. Can a body have zero mass and zero weight?
49. How many kilograms are there in (I) a slug (ii) one u?
50. How many amu make 1kg?
51. How many metric tonnes are there in a teragram?
Objective Problems
52. How many kilograms are there in (i) a slug (ii) one u?
(A) 14.57 kg, 1.66  10–27 kg (B) 14.57 kg, 1.66  10–31 kg
(C) 15.57 kg, 1.66  10–27 kg (D) 14.57 kg, 1.66  10–31 kg
53. Order of magnitude of mass of proton is
(A) 10–28 kg (B) 10–27 kg
(C) 10–26 kg (D) 10–25 kg
54. The fundamental unit of the quantity of matter is
(A) Kg (B) mol.
(C) gm (D) meter

80
UNITS & DIMENSIONS PHYSICS

Questions based on Measurement of time


Conceptual / Subjective Problems
55. Which type of phenomenon can be used as a measure of time? Give three examples.
56. Mankind has existed for about 106 years whereas the universe is about 1010 years old. If the age of the
universe is taken to be one day, how many seconds has the mankind existed?
57. Human heart is an inbuilt clock. Comment.
58. What is the basic rule to measure time? Name any four clocks based on this rule.
Objective Problems
59. How many number of times the human heart beats in the life of 60 years of a man. Given that the heart
beats once in 0.8 s?
(A) 2.3668  108 (B) 2.3668  109
(C) 2.3668  1010 (D) 2.3668  1011
60. Mankind has existed for about 10 years whereas the universe is about 1010 years old. If the age of the
6

universe is taken to be one day, how many seconds has the mankind existed?
(A) 7.54 (B) 9.54
(C) 8.64 (D) 9.85
61. Human life expectancy is of the order of :
(A) 105 s (B) 107 s
(C) 109 s (D) 1015 s
62. Chronometer is used to measure
(A) time (B) mass
(A) density (B) distance
63. Radioactive dating is used to measure
(A) time (B) gravitational mass
(C) Inertial mass (D) distance

Questions based on Dimensional formulae & Dimensional equation


Conceptual / Subjective Problems
64. The position of a particle moving along x–axis depends on time according to the relation x  at 2  bt 3
where t is time in sec. and x is in meters. What are the units and dimensional formulae of a and b.
c
65. The velocity v of a particle depends upon time t according to the relation v  a  bt  . Write the
d t
dimensional formulae of a, b, c and d.
66. Write the dimensional formula of a/b in the relation F  a x  bt where F is force, x is distance and t
2

is time.
a  t2
67. Write the dimensional formula of a/b in the relation P  where P is pressure, x is distance and t is time.
bx
x2  b
68. Write the dimensional formula of a and b in the relation P  , where P is power, x is distance and
at
t is time.
b  x2
69. Write the dimensional formula of ab in the relation E  , where E is the energy, x is the distance
at
and t is the time.
70. Give the dimensional formula of Gravitational constant.

81
PHYSICS UNITS & DIMENSIONS

71. Using the principle of homogeneity find which one is correct?


4 2 a 2 4 2 a 3
(i) T 2  (ii) T 2 
G GM
Here T is time period, G is gravitational constant, M is mass and a is radius of earth.
72. Find the dimension of a/b in the equation:
a  t2
P , where P is pressure, x is distance and t is time.
bx
73. (a) If y  A sin  kx  t  where x & t stand for distance and time respectively. Obtain the dimensional
formula for  & k.
 Pr 4
(b) What will be the dimensions of  in the equation V  . Here V = volume per unit time, P =
8 l
Pressure, l = length and r = radius.
74. Give an example of a physical quantity which has neither a unit but nor dimensions.
75. If one says that the area of disc of radius r and diameter d is r2d, how you would refute?
Objective Problems
76. Volt/metre is the unit of
(A) Potential (B) Work
(C) Force (D) Electric intensity
Ax1 / 2
77. The potential energy of a particle varies with distance x as U = where A and B are constants.
x2  B
The dimensional formula for A  B is
(A) [M1L7/2T-2] (B) [M1L11/2T-2]
(C) [M1L5/2T-2] (D) [M1L9/2T-2]
a sin   b cos
78. If x = , then
ab
(A) The dimensions of x and a are same (B) the dimensions of a and b are not same
(C) x is dimensionless (D) none of the above
79. The equation of wave is given by
x 
y  A sin    K  ,
v 
where ω is angular velocity and v is linear velocity. The dimensional formula of K is :
(A) [LT] (B) [T]
-1
(C) [T ] (D) [T2]
C
80. If A  B  , the dimensions of ‘B’ and ‘C’ are [LT-1] and [M0LT0], respectively. Find the
DE
dimensions of A, D and E :
(A) A = [M0L0T-1], D = [T], E = [LT] (B) A = [MLT0], D = [T2], E = [T2]
(C) A = [LT-1], D = [MT], E = [MT] (D) A = [LT-1], D = [T], E = [T]
 0 lV
81. If X = , where ε0 is the permittivity of free space, l is length V is potential difference and t is time.
t
The dimensions of X are the same as that of

82
UNITS & DIMENSIONS PHYSICS

(A) Charge (B) Resistance


(C) Voltage (D) Current
𝑒4
82. The dimensional formula of is where e = charge, mp, me are masses, c = speed of light and G
𝜀02 𝑚𝑝 𝑚𝑒2𝑐 3𝐺
= gravitational constant.
(A) [M] (B) [T]
(C) [AT] (D) [A4T-2M]
83. The workdone by a battery is W = εΔq, where Δq = charge transferred by battery ε = emf of the battery.
What are is the dimensional formula of emf of battery?
(A) [A-2M0L0T-2] (B) [A-2ML2T-3]
(C) [A0M2T-3] (D) [A-1ML2T-3]
84. The dimensions of the quantities in one or more of the following pairs are the same. Identify the pair/pairs
a. Torque and work
b. Angular momentum and Plank’s constant
c. Energy density and Young’s modulus
d. Light year and wavelength
(A) ‘a’ only (B) ‘a’ and ‘b’
(C) ‘a’, ‘b’ and ‘c’ (D) ‘a’, ‘b’, ‘c’ and ‘d’.
85. Given that y = a sin ωt + bt + ct cos ωt
2

The unit of abc is same as that of :


y
(A) y (B)
t
2 3
 y  y
(C)   (D)  
t t
86. Let x, y and z be three physical quantities having different dimensions. Which of the following
mathematical operations must be meaningless?
x x y
(A) z (B)
y x y
(C) x y  z (D) z  x  y
2 3 2 3

Questions based on Checking the correctness


Conceptual / Subjective Problems
 2 3 4
87. A function f () is defined as: f    1      ......................
1! 2! 3! 4!
Why is it necessary for  to be a dimensionless quantity?
88. A famous relation of Physics relates moving mass m to the rest mass m0 of a particle in terms of its speed
u and the speed of light c. (This relation first arose as a consequence of special theory of relativity due to
Albert Einstein). A body recalls the relation almost correctly but forgets where to put the constant c. He
m0
writes m  . Guess where to put the missing c.
1  
2 1/2

89. Using the method of Dimensional analysis, check the dimensional correctness of the following relations
(a) v 2  u 2  2as where v is final velocity, u is initial velocity, a is acceleration & s is distance covered

83
PHYSICS UNITS & DIMENSIONS

1 T
(b) v  , where v is frequency measure in sec–1, l is length, T is tension (Force) &  is mass per
2 
unit length (kg/m)
(c) V  Avt where V is volume, A is area, v is velocity, t is time & h is height
 cos 
(d) h  , where  is surface tension measured in N/m, r is radius,  is density & g is acceleration
r2 g
due to gravity
S
(e) F  m(a  ) , where F is force, a is acceleration, S is distance & v is velocity
v
p
(f) v  , where v is velocity, p is pressure &  is density

a
(g) Snth  u   2n 1 , where Snth = displacement in nth second, a = acceleration, n = time
2
I
90. Test by the method of dimensions the validity of the equation t  2 , where t is the time period of
k
torsional oscillation of a body of moment of inertia I about the axis of rotation and k is the torque per unit
radian twist due to torsional reaction of the suspension.
91. An apprentice engineer found that the volume V of water which passes any point of a canal during time t
is connected with area the cross –section a of the canal and the velocity v of water by the relation V =
kavt. Verify the correctness of the relation.
1 B
92. Check the dimensional consistency of the formula, v  , where v is the velocity of the longitudinal
 
waves, B is the bulk modulus and  is the density of the medium,  is the wavelength of the waves.
3g
93. Check the correctness of the relation,  , where the letters have their usual meaning.
4 RG
94. A flywheel of moment of inertia I is set into rotation by the descent of a mass m. If  is the angular
velocity of the flywheel and v is the linear velocity of mass when it has ascended through a distance h.
1 1
mgh  mv 2  I  2
2 2
Show by your knowledge of dimensions that this equation is dimensionally correct.
mgl 3
95. Test by method of dimensions the correctness of the equation,   , where  is the depression
4bd 3Y
produced at the centre of a bar of length l, breadth b and depth d, placed symmetrically on two knife
edges near its ends and loaded in the middle by a mass m, and Y is the Young’s modulus of the material
of the bar.

84
UNITS & DIMENSIONS PHYSICS

Objective Problems
96. A circular railway track of radius r is banked at angle θ so that a train moving with speed v can safely go
round the track. A student writes :
tan θ = rg/v2. Why this relation is not correct?
(i) Equality of dimensions does not guarantee correctness of the relation
(ii) Dimensionally correct relation may not be numerically correct
(iii) The relation is dimensionally incorrect
(A) (i) & (ii) (B) (ii) & (iii)
(C) (iii) & (i) (D) (i), (ii) & (iii)

Questions based on Derivation of formulae


Conceptual / Subjective Problems
97. The frequency of vibration (v) of a string may depend upon length (l) of the string, tension (T) in the
string and mass per unit length (m) of the string. Using the method of dimensional analysis, derive the
formula for v.
98. A planet moves round the Sun in a circular orbit. Assuming that the period of revolution t of the planet
depends upon the radius R of its orbit, the mass M of the planet and the universal constant G, prove
R3
dimensionally that t 
2
.
GM
99. The viscosity  of a gas depends on the mass, the effective diameter and the mean speed of the
molecules. Use dimensional analysis to find  as a function of these variables.
100. A jet of water of cross sectional area A and velocity v impinges normally on a stationary flat plate. The
mass per unit volume of water is . By dimensional analysis, determine an expression for the force F
exerted by the jet against the plate.
101. A body of mass m hung at one end of a spring executes SHM. The force constant of the spring is K while
2 m
its time period of vibration is T. Prove by dimensional method that equation T  is incorrect.
K
Derive the correct relation for the time period.
102. Assuming that the mass m of the largest stone that can be moved by a flowing river depends on velocity
v, the density  and acceleration due to gravity g, show that m varies directly as the sixth power of
velocity of flow.
103. The height h to which a liquid rises in capillary tube of radius r depends, in addition to r, on (i) surface
tension s (ii) density  of the liquid and (iii) g. Is it possible to obtain dimensionally a relation for h
without the experimental information that h is inversely proportional to r? What is its relation?
104. The Reynolds number (Re) for a liquid flowing through a tube depends upon its density , coefficient of
viscosity , its speed v and diameter D of the tube. Obtain an expression for R e, given Re is directly
proportional to D.
105. If dimensions of length are expressed as GxCyhz where G, C and h are the universal gravitational constant,
speed of light and Plank’s constant respectively what are the values of x, y and z?
106. If the velocity of light C, the constant of gravitation G and Plank’s constant h, be chosen as fundamental
units, find the value of a gram, a centimeter and a second in terms of new units of mass, length and time
respectively. Given C = 3  1010 cm/s. G = 6.67  10–8 dyne cm2/g2, h = 6.6  10–27 ergs.

85
PHYSICS UNITS & DIMENSIONS

107. A calorie is a unit of heat energy and it equals about 4.2 J, where1 J = 1 Kgm2s– 2. Suppose we employ a
system of units in which the unit of mass equals  kg, the unit of length equals m, the unit of time is
γs. Show that a calories have a magnitude 4.2  1 2 2 in terms of new units.
108. Experiments show that frequency of tunning fork depend on length L of its prong, density d, and young’s
modulus Y of material of tunning fork. Derive expression for the frequency by the dimensional method.
109. If dimensions of length are expressed as G x c y h z . Where G, c, h are universal gravitational constant,
velocity of light and planks constant respectively. Find the values of x, y, z.
110. Derive by method of dimensional analysis, an expression for the volume of liquid flowing out per second
through a narrow pipe. Assume that the rate of flow of liquid depends on
(i) The coefficient of viscosity  of liquid
(ii) The radius r of the pipe
(iii) The pressure gradient (P/1) along the pipe. Take K = /8.
111. The coefficient of viscosity  depends : (1) on mass (m) (2) the effective diameter (d) and (3) the mean
speed (v) of the gas molecules. Find the relation between them using dimensional analysis.
112. What do you mean by dimension? Turpentine oil is flowing through a tube of length l and radius r. The
pressure difference between the two ends of the tube is P. The viscosity of the oil is given by:
𝑃 (𝑟 2 − 𝑥 2 )
𝜂=
4𝑣𝑙
Where v is velocity of oil at a distance x from the axis of the tube. What are the dimensions of 𝜂?
Objective Problems
113. The speed (v) of ripples on the surface of water depends on surface tension (σ), density (ρ) and
wavelength (λ). The square of speed (v) is proportional to
(A) σ/ρλ (B) ρ/σλ
(C) λ/σρ (D) σρλ
114. The frequency ‘f’ of a mass ‘m’ suspended from a spring of force constant k is given by f = Cmxky where
C is a dimensionless constant. The values of x and y are :
1 1 1 1
(A) x  ; y (B) x   ; y  
2 2 2 2
1 1 1 1
(C) x  ; y   (D) x   ; y 
2 2 2 2
115. The volume V of the liquid crossing through a tube is related to the area of cross section A, velocity v
and time t as V  A v  t  . If  = 1 then, which of the following relations is correct?
(A)      (B)     
(C)      (D)     
116. In a system of units, the Planck’s constant (h), the gravitational constant (G) and the speed of light (c) are
taken as the fundamental units. What is the dimensional formula of force in this system of units?
0 1 4 1 0 4
(A) [h G c ] (B) [h G c ]
1 4 0 4 2 2
(C) [h G c ] (D) [h G c ]

86
UNITS & DIMENSIONS PHYSICS

117. The velocity of surface waves depends upon surface tension (S), coefficient of viscosity (  ) and density (
 ). The relation for velocity is
s2 s
(A) (B)
 
 
(C) (D)
s2 
Questions based on Conversion of one system
Conceptual / Subjective Problems
118. Which of the following represents an acceleration of 10 cm/s²?
(a) 10 Km/hr² (b) 1.29  106 m/hr² (c) 3.402  107 m/s².
119. A calorie is a unit of heat or energy and it equals about 4.2 J where 1J = 1 kg m2s–2. Suppose we employ
a system of units in which the unit of mass equals a kg, the unit of length equals b m, the unit of time is
g s. Show that a calorie has a magnitude 4.2 a –1b–2g2 in terms of the new units.
120. The speed of light in air is 3.00  108 ms–1. The distance travelled by light in one year (i.e., 365 days =
3.154  107 s) is known as light year. A student calculates one light year = 9.462  1015 m. Do you agree
with the student? If not, write the correct value of one light year.
121. If velocity of light in air (= 3  108 m/s), acceleration due to gravity (= 9.8 m/s2) and density of mercury
at 0oC (=13600 Kg/m3) be chosen as fundamental units, find the units of mass, length and time.
122. Find the value of 60J/min. on a system which has 100g, 100cm and 1min as fundamental units.
123. The density of a material in cgs system is 8 g cm–3. In a system of units in which unit of length is 5cm
and unit of mass is 20g, what is the density of the material?
124. Find the value of 20J on a system which has 10cm, one kg and 1/2 minute as the fundamental units of
length, mass and time respectively.
125. Find the value of a force of 100 dyne on a system based on metre, the kg and the minute as the
fundamental units.
126. The speed of sound in air at room temperature is 332m/s. If the unit of length is 1km and that of time is
1hour, calculate this speed in km/hr.
127. The magnitude of a force is 36 units if 1kg, 1m and 1 min are taken as fundamental units. Find the value
of this force in cgs system.
128. A new unit of length is chosen such that the speed of light in vacuum is unity. What is the distance
between the sun and earth in terms of the unit, if light takes 8 min and 20 sec, to cover the distance?
129. If 500g be the unit of mass, 50sec. the unit of time and acceleration due to gravity (980 cm/s2) be the unit
of acceleration, find what will be the new unit of energy?
130. New system of units is so chosen that the units of mass equals  Kg, the unit of length equals  m and
unit of time is  s. How much will 5 J measure in this new system.
131. If in a new system of units called star units 1kg* =10 kg, 1m* = 1km, 1s* = 1min, what will be the value
of 1J of energy in the new system.
Objective Problems
132. Density of wood is 0.5g/cc in the CGS system of units. The corresponding value in MKS units is
(A) 500 (B) 5
(C) 0.5 (D) 5000

87
PHYSICS UNITS & DIMENSIONS

133. If velocity of light in air (=3x108 m/s), acceleration due to gravity (=10m/s 2) and density of mercury at
00C (=13600Kg/m3) be chosen as fundamental units, what is the units of mass?
(A) 3 107 (B) 9 1015
(C) 9.911051 (D) 3
134. In the above question, what is the unit of length?
(A) 3 107 (B) 9 1015
(C) 3.672 1029 (D) 3
135. In Q133, what is the unit of time?
(A) 3 107 (B) 9 1015
(C) 3.672 1029 (D) 3
136. Find the value of 60J/min. on a system which has 100g, 100cm and 1min as fundamental units.
(A) 6 (B) 2.16  104
(C) 2.16  106 (D) 6000
-3
137. The density of a material in cgs system is 8gcm . In a system of units in which unit of length is 5cm and
unit of mass is 20g, what is the density of the material?
(A) 5 (B) 50
(C) 500 (D) 5000
138. If the unit of force were 10 N, that of power 10 watt and that of time 10 -3 s what would be the units of
3 3

length?
(A) 103 (B) 102
(C) 101 (D) 1
139. Convert 1 MW power to a new system having base units of mass, length and time as 10 kg, 1 dm and
1 minute respectively.
(A) 2.16 10 (B) 2.16  10
5 12

(C) 2.16 10 (D) 2.16 10


7 8

Questions based on Significant figures


Conceptual / Subjective Problems
140. Differentiate between 2.00cm and 2.0cm using the concept of significant figures.
141. A jeweller puts a diamond in a box weighing 1.2 kg. Find the total weight of the box and diamond with
due regard to significant figures, if the weight of the diamond is 5.42 g.
142. Express 3.0  10–4 – 1.7  10–6 with proper significant figures.
143. For the period 1960–1983, the metre was defined to be 1,650, 763.73 wavelength of a certain orange –red
light emitted by krypton atoms. Compute the distance of the wavelength in nanometers to be correct
number of significant figures.
144. What is the number of significant figures in 0.06070?
145. Solve with due consideration of significant figures:
1.4 108
(i) (ii) 3.8 106  4.2 105
2.6 106
146. State and explain four rules for counting significant figures in a measurement.

88
UNITS & DIMENSIONS PHYSICS

Objective Problems
147. Assertion : The number of significant figures in 0.001 is 1 while in 0.100 in 3.
Reason : Zeros before a non –zero significant digit are not counted while zeros after a non –zero
significant digit are counted
(A) If both assertion and reason are true and the reason is correct explanation of the assertion
(B) If both assertion and reason are true, but reason is not the correct explanation of the assertion
(C) If assertion is true, but the reason is false
(D) If assertion is false, but the reason is true
148. The value of resistance is 10.845 ohm and the current is 3.23 ampere. The potential difference is
35.02935 volt. The value of potential difference in terms of significant figures would be :
(A) 35 V (B) 35.0 V
(C) 35.029 V (D) 35.03 V
149. The mass of a box is 2.3 kg. Two marbles of masses 2.15 g and 12.39 g are added to it. The total mass of
the box to the correct number of significant figures is
(A) 2.340 kg (B) 2.3145 kg.
(C) 2.3 kg (D) 2.31 kg
150. Each side of a cube is measured to be 5.402 cm. The total surface area and the volume of the cube in
appropriate significant figures are :
(A) 175.1 cm 2 , 157 cm 2 (B) 175.1 cm 2 , 157.6 cm 3
(C) 175 cm 2 , 157 cm 2 (D) 175.08 cm 2 , 157.639 cm 3

Questions based on Accuracy and Precision


Conceptual / Subjective Problems
151. “Accurate measurement of a physical quantity may not be a precise measurement”. Explain the statement.
152. Which of the following readings is the most accurate:
(i) 5000 m (ii) 5  102 m (iii) 5  103 m?
Objective Problems
153. The accuracy of a clock is one part in 1010. The maximum difference between two such clocks operating
for 1010 s is :
(A) 1s (B) 2 s
(C) 10 s (D) 1010 s

Questions based on Various types of errors


Conceptual / Subjective Problems
154. We measure the period of oscillation of a simple pendulum. In successive measurements, the readings
turn out to be 2.63 s, 2.56 s, 2.42 s, 2.71 s and 2.80 s. Calculate relative error and percentage error.
155. In an experiment, the refractive index of glass was found to be 1.45, 1.56, 1.54, 1.44, 1.53 s. Calculate (i)
Mean absolute error (ii) Relative error (iii) Percentage error.
Objective Problems
156. A student measured the diameter of a wire using a screw gauge with least count 0.001 cm and listed the
measurements. The correct measurement is :
(A) 5.320 cm (B) 5.3 cm
(C) 5.32 cm (D) 5.3200 cm

89
PHYSICS UNITS & DIMENSIONS

157. The least count of a stop watch is 0.1 s. The time of 20 oscillations of the pendulum is found to be 20 s.
The percentage error in the time period is :
(A) 0.25% (B) 0.5%
(C) 0.75% (D) 1.0%
158. The random error in the arithmetic mean of 100 observations is x, then random error in the arithmetic
mean of 400 observations would be:
1
(A) 4x (B) x
4
1
(C) 2x (D) x
2
159. The mean time period of second’s pendulum is 2.00 s and mean absolute error on the time period is
0.05s. To express maximum estimate of error, time period should be written as :
(A)  2.00  0.01 s (B)  2.00  0.025 s

(C)  2.00  0.05 s (D)  2.00  0.10 s

Questions based on Propagation of errors


Conceptual / Subjective Problems
160. Two rods have lengths measured as (1.8  0.2)m and (2.3  0.1)m . Calculate their combined length
with error limits.
161. The initial and final temperatures of a liquid are measured to be (67.7  0.2)C and (76.3  0.3)°C .
Calculate the rise in temperature.
162. The original length of a wire is (153.7  0.6)cm . It is stretched to (155.3  0.2)cm . Calculate the
elongation in the wire with error limits.
163. The relative density of a material is found by weighing it first in air and then in water. If the weight in air
is (10.0  0.1)g and weight in water is (5.00  0.1)g, find the maximum permissible percentage error in
relative density.
164. A body travels uniformly a distance of (16.0  0.4) m in time (4.0  0.3)s. Calculate its velocity with
error limits. What is the percentage error in velocity?
165. The radius of a sphere is measured with an error of 2%. What would be the error in volume of the sphere?
166. The radius of a sphere is measured to be (2.1  0.05)cm . Calculate its surface area with error limits.
167. In an experiment to determine the specific resistance r of a meterial, the following data was obtained.
 r2R
Resistance, R = (64  2), length, l = (156.0  0.1) cm, radius r = (0.26  0.02)cm. Given   ,
l
find the percentage error in r.
168. The following observations are made during an experiment to find the value of g using simple pendulum.
l = 90.0 cm, time (t) for 20 vibrations = 36.0 s. Find the percentage error in the measurement of g. Given
l
that T  2 . Length is being measurement to an accuracy of 0.1cm and time to 0.2s.
g
169. Calculate the focal length (f) of a spherical mirror from the following calculations : u = (50.1  0.5) cm,
v = (20.1  0.2)cm.

90
UNITS & DIMENSIONS PHYSICS

rh g
170. The surface tension (T) is measured by capillary rise formula T  . The quantities , g and  are
2cos 
taken from the table of constants while height and diameter are measured as h = (3.00 + 0.01) cm and
D = (0.250  0.001)cm. Find the percentage error in T.
l2 h
171. The radius of curvature of a concave mirror, measured by a spherometer, is given by R   . The
6h 2
value of l and h are 4.0 cm and 0.065 cm respectively, where l is measured by a meter scale and h by a
spherometer. Find the relative error in the measurement of R.
172. A beam of length ‘L’, breadth ‘b’ and thickness ‘d’ when loaded by a weight Mg in the middle, a
depression ‘e’ is produced in it. By measuring this depression ’e’, the value of Young’s modulus of the
MgL3
material of the beam can be calculated by using the expression Y  . Following are the values of
4bd 3e
different physical quantities obtained in one set of observations on this experiment: M = 1000gms, L =
200cm, b = 2.54cm, d = 0.620cm, e = 0.1764cm. If ‘M’ is measured by spring balance, ‘L’ by meter
scale, ‘b’ by vernier callipers, ‘d’ by screw gauge and ‘e’ by spherometer, then what will be maximum
possible % error in Y?
173. What is an error? Calculate the resistance if potential difference V is  20  1  V when applied across a
resistance R gives a current of  2.5  0.5 A.
174. If a = 10.0  0.1 and b = 5.0  2% then what is the percentage error in a  b ?
2
175. A physical quantity is related to four measurable quantities a, b, c, d as P  a b c d
2 3 5/2
percentage error
in the measurement of a, b, c, d are 1%, 2%, 3% and 4% respectively. What is the percentage error in
the quantity P?
176. The specific resistance is  of a thin wire of radius r cm, resistance R and length L cm.
If r  0.26  0.02cm, R  32  1 and L  78  0.01cm, find the percentage error in .

l
177. The period of oscillation of a simple pendulum is T = 2 . Measured value of L is 20.0 cm known to
g
1 mm accuracy and time for 100 oscillations of the pendulum is found to be 90 s using a wrist watch of
1s resolution. What is the accuracy in the determination of g?
178. How does the error propagate in sum of two physical quantities?
179. Discuss the process of combination of errors when result involves the ratio of two quantities i.e. X = A/B.
180. In an experiment in determining the density of a rectangular block, the dimensions of the block are
measured with a vernier caliper with a least count of 0.01 cm and its mass is measured with a beam
balance of least count 0.1 g, L = 5.12 cm, b = 2.56 cm, t = 0.37 cm and m = 39.3 g. Report correctly the
density of the block.
181. How can we estimate the error in the power of different measured quantities? Deduce the general rule for
evaluating the error in combined calculations.

91
PHYSICS UNITS & DIMENSIONS

Objective Problems
182. Assertion : If error in measurement of mass is 2% and that in measurement of velocity is 5%, then error
in measurement of kinetic energy is 6%.
∆𝐾 ∆𝑚 ∆𝑣
Reason : Error in kinetic energy is 𝐾 = ( 𝑚 + 2 𝑣 )
(A) If both assertion and reason are true and the reason is correct explanation of the assertion
(B) If both assertion and reason are true, but reason is not the correct explanation of the assertion
(C) If assertion is true, but the reason is false
(D) If assertion is false, but the reason is true
183. A distance of about 50cm is measured using a metre stick having mm division. The percentage error is
(A) 0.2% (B) 0.4%
(C) 0.02% (D) 0.002%
184. Specific resistance of a thin circular wire of radius r cm, resistance R ohms and length L is given by  =
πr2R/L. If r = 0.26 ± 0.01cm, R = (30±2) ohm and L=(75.00±0.01)cm, find the percentage errors in δ.
(A) 7.15 (B) 10.3
(C) 14.3 (D) 20.6
185. A beam of length ‘L’, breadth ‘b’ and thickness ‘d’ when loaded by a weight Mg in the middle, a
depression ‘e’ is produced in it. By measuring this depression ’e’, the value of Young’s modulus of the
material of the beam can be calculated by using the expression Y=MgL 3/4bd3e. Following are the values
of different physical quantities obtained in one set of observations on this experiment:
M = 1000  5g, L = 200  0.1 cm, b = 2.54  0.01cm, d = 0.620  0.001cm, e = 0.175  0.005cm. If ‘M’
is measured by spring balance, ‘L’ by meter scale, ‘b’ by Vernier callipers, ‘d’ by screw gauge and ‘e’ by
spherometer, then what will be maximum possible % error in Y?
(A) 1.38 (B) 2.38
(C) 4.38 (D) 4.10
186. The length, of a cylinder is measured with a metre rod having least count 0.1 cm. Its diameter is
measured with Vernier calipers having least count 0.01 cm. Given that length is 5.0 cm and radius is
2.0 cm. The percentage error in the calculated value of the volume will be :
(A) 1% (B) 2%
(C) 2.5% (D) 4%
187. The length of a rod is 11.05  0.05 cm. What is the total length of two rods?
(A)  22.1  0.05 cm (B)  22.1  0.1 cm
(C)  22.10  0.05 cm (D)  22.10  0.01 cm

Questions based on Rounding off


Conceptual / Subjective Problems
188. It is advised to measure the difference between two nearly equal quantities directly instead of measuring
these quantities and finding their difference. Why?

92
UNITS & DIMENSIONS PHYSICS

ANSWER KEY
Ques Answers Ques Answers Ques Answers Ques Answers
1 kg = 1.82  107 unit of mass
2 (n1 /x) 64 m/s2, [LT–2], m/s3, [LT–3] 106 1 m = 2.46  1034 unit of length 148 B
1 s = 7.42  1042 unit of time
3 2.1 eV 65 [LT–1], [LT–2], [L], [T] 109 x = –1/2, y = 3/2, z = –1/2 149 C

kPr  Pr
4 4
4 CGS 66 [L–1/2 T 2] 110 Q  150 B
L 8 L
M
5 Yes, entropy 67 [MT–2] 111  v 152 5000 m
d2
11 B 68 [M–1T2], [L2] 112 [ML–1T–1] 153 A
12 D 69 [M–1L2T] 113 A 154  4%
13 D 70 (a) [M–1L3T–2]; (b) Angle, rad, [M0 L0T0] 114 D 155 3%
14 A 71 (i) Incorrect; (ii) correct 115 D 156 A
mN = milli newton; Nm
15 72 [M1 L0T–2] 116 A 157 B
= newton meter
16 2.07  105 A.U 73 (a) [L–1]; (b) [ML–1T–1] 117 B 158 B
23 10–13m 74 relative density 118 b 159 C
Refute – Prove to be wrong, dimensional 1.21  1050 kg, 9.3 1015 m,
26 1.057  10–16 75 121 160 (4.3  0.3)m
wrong 3.1  107 s
27 2.95  10–40 76 D 122 2.16  106 161 (8.6  0.5)oC
1/3
 3m 
29   77 B 123 50 new system 162 (1.6  0.8)cm
 0.5   
35 1.39  109 m 78 C 124 18  105 units 163 5%
(55.4 
39 B 79 B 125 3.6 166
2.6)cm2
40 A 80 D 126 1195.2 km/h 167 18.52%
41 C 81 D 127 103 dyne 168 1.22%
42 B 82 B 128 500 new units of length 169 (14.3  0.4)cm
43 B 83 D 129 1.2  1012 170 0.7%
44 D 84 D 130 A 171 0.08
45 B 85 D 131 C 172 3.1 %
46 D 86 B 132 B 173 (8  2)
(a) correct; (b) correct; (c) correct; (d) Incorrect;
47 1.66  10–27 kg 89 133 A 174 3%
(e) Incorrect; (f) correct; (g) correct
49 14.57 kg, 1.66  10–27 kg 91 Correct 134 C 175 23.5 %
50 6  1026 92 Incorrect 135 B 176 18.5 %
51 106 93 Correct 136 A 177 26.3 cm s–2
52 A 95 Correct 137 B 180 8.10  0.29
53 B 96 A 138  2  5   1 2 2
182 D

k T
54 B 97 v 139 3.6  10–4 183 B
l m
mv
56 8.64 s 99  k 141 1.2 kg 184 C
d2
59 B 100 F  k  Av 2 142 3.0  10–4 185 C

m
60 C 101 T  2 143 605.780211 nm 186 C
k
k
61 C 103 No, h 144 Four 187 D
r g
v D
62 A 104 Re  145 (a) 5.4  101 (b) 4.6  10–5

63 A 105 (1/2, –3/2, 1/2) 147 A

93
PHYSICS UNITS & DIMENSIONS

OBJECTIVE QUESTIONS BANK

Ax1/2
1. The potential energy of a particle varies with distance x as U  where A and B are constants.
x2  B
The dimensional formula for A x B is
(A) M1L7/2T–2 (B) M1L11/2T–2
(C) M1L5/2T–2 (D) M1L9/2T–2
 0 IV
2. If X  , where 0 is the permittivity of free space, l is length V is potential difference and t is
t
time. The dimensions of X are the same as that of
(A) charge (B) resistance
(C) voltage (D) current
3. The accurate value of length of a pencil is 6.04 cm. Two students A and B measure his length, one by a
metre scale and another by a Vernier calipers respectively. The length measured by A is 5.9 cm while the
length measured by B is 6.37cm. Then the accuracy is more in measurement of :
(A) A (B) B
(C) neither of A nor of B (D) A and B equally
a sin   b cos 
4. If x  , then
ab
(A) the dimensions of x and a are same (B) the dimensions of a and b are not same
(C) x is dimensionless (D) none of the above
5. The speed (v) of ripples on the surface of water depends on surface tension (), density () and
wavelength (). The square of speed (v) is proportional to
(A) / (B) /
(C) / (D) 
x 
6. The equation of wave is given by y  A sin    K  , where  is angular velocity and y is linear
y 
velocity. The dimensions of K are :
(A) LT (B) T
(C) T –1 (D) T2
7. A circular railway track of radius r is banked at angle  so that a train moving with speed v can safely go
round the track. A student writes : tan = rg/v2. Why this relation is not correct?
(i) Equality of dimensions does not guarantee correctness of the relation
(ii) Dimensionally correct relation may not be numerically correct
(iii) The relation is dimensionally incorrect
(A) (i) &(ii) (B) (ii)&(iii)
(C) (iii)& (i) (D) (i), (ii) & (iii)
e4
8. The dimensional formula of is (where e = charge, mp, me are masses, c = speed of light
 02 mp me 2c3G
and G = gravitational constant)
(A) [M] (B) [T]
(C) [AT] (D) [A4T–2M]

94
UNITS & DIMENSIONS PHYSICS

9. 9.25 g of substance occupies a volume 1.5cm2. Express density by seeking the idea of appropriate
significant figures.
(A) 6.2 g/cm3 (B) 6.17 g/cm3
(C) 6.612 g/cm3 (D) 6.1625 %
10. The workdone by a battery is W = q where q = charge transferred by battery  = emf of the battery.
What are dimensions of emf of battery?
(A) [A–2M0L0T–2] (B) [A–2ML2T–3]
0 2 –3
(C) [A M T ] (D) [A–1ML2T–3]
11. The dimensions of h/e, where h is planck’s constant and e is electronic charge are same as that of
(A) magnetic flux (B) electric flux
(C) electric field (D) magnetic field
12. A thin copper wire of length l metre increases in length by 2% when heated through 10 o C. What is the
percentage increase in area when a square copper sheet of length l metre is heated through 10o C?
(A) 4% (B) 8%
(C) 16% (D) none of the above
13. The percentage error in the volume of a sphere if percentage error in its diameter is 2% :
(A) p x 6% (B) 4p %
(C) 6 % (D) 3%
14. The number of significant figures in the measurement 5418000 m are:
(A) 4 (B) 5
(C) 6 (D) 7
15. The value of resistance is 10.845 ohm and the current is 3.23 ampere. The potential difference is
35.02935 volt. The value of potential difference in terms of significant figures would be :
(A) 35 V (B) 35.0 V
(C) 35.029 V (D) 35.03 V
16. If velocity, acceleration and force are the fundamental quantities denoted by V, A and F respectively,
then the dimensions of Young’s modulus are :
(A) FA2V–4 (B) F4A2V–1
(C) F2A4V–1 (D) F–1A4V2
17. The dimensions of the quantities in one or more of the following pairs are the same. Identify the
pair/pairs
(a) Torque and work (b) Angular momentum and Plank’s constant
(c) Energy density and Young’s modulus (d) Light year and wavelength
(A) ‘a’ only (B) ‘a’ and ‘b’
(C) ‘a’, ‘b’ and ‘c’ (D) ‘a’, ‘b’, ‘c’ and ‘d’.
18. The frequency ‘f’ of a mass ‘m’ suspended from a spring of force constant k is given by f = Cmxky where
C is a dimensionless constant. The values of x and y are :
1 1 1 1
(A) x  ; y (B) x   ; y  
2 2 2 2
1 1 1 1
(C) x  ; y   (D) x   ; y 
2 2 2 2

95
PHYSICS UNITS & DIMENSIONS

19. Given that y = a sin ωt + bt + ct2cos ωt


The unit of abc is same as that of :
y
(A) y (B)
t
2 3
 y  y
(C)   (D)  
t t
20. A distance of about 50cm is measured using a metre stick having mm division. The percentage error is
(A) 0.2% (B) 0.4%
(C) 0.02% (D) 0.002%
21. The physical quantity not having same dimensions are :
(B) speed and  0 0 
1/2
(A) momentum and plank’s constant
P
(C) speed and (D) surface tension and spring constant

cos  T x
22. A hypothetical experiment conducted to determine Young’s modulus, gave the formula y  .
l3
If T = time period,  = torque and l = length, then the value of x is :
(A) zero (B) 1
(C) 2 (D) 3
a b –c
23. If a physical quantity is represented by x = M L T and if the percentage errors in the measurement of
M, L and T are %, % and % respectively, then the total percentage error in x is :
(A) (a +b+c)% (B) (a –b+c) %
(C) (a –b–c)% (D) none of these
24. The unit of surface tension may be expressed as
(A) Joule meter (B) newton meter
(C) Joule metre2 (D) newton metre2
25. Which of the following is NOT the unit of energy?
(A) Nm (B) J
(C) Wh (D) kW
26. The dimensions of angular frequency are the same as that of the
(A) frequency (B) time period
(C) angle (D) angle/frequency
27. Which of the following is the dimensional formula for the gravitational constant?
(A) M 1L3T 2 (B) M 2 L2T 2
(C) M 0 L0T 0 (D) M 2 L2T 2
28. Which of the following pair does not possess same dimensions?
(A) Impulse and momentum (B) Angular frequency and velocity gradient
(C) Stress and strain (D) Surface tension and surface energy
29. If n is the numerical value of the physical quantity in the system in which its unit is u, then which of the
following relations is correct?
n u
(A)  constant (B)  constant
u n
(C) nu  constant (D) none of the above

96
UNITS & DIMENSIONS PHYSICS

 g / R
1/2
30. Given that g is acceleration due to gravity and R is the radius of the earth. Then possesses the
dimensions of
(A) Orbital speed (B) Angular speed
(C) Escape velocity (D) Time period
31. If L and R denote the inductance and resistance, then the dimensional formula for R/L is same as that for
(A) frequency (B) time period
2
(C) (frequency) (D) (time period)2
1/2
p F 
32. The frequency of vibration of a string is given by     . Here p is the number of segments in
2l  m 
which the string is divided, F is the tension in the string and l is its length. The dimensional formula for m is
(A) M 0 L0T 0 (B) ML1T 0
(C) ML0T 1 (D) M 0 LT 1
33. The least count of instrument is 0.01 cm. Taking all precautions, the most possible error in the
measurement can be
(A) 0.005 cm (B) 0.001 cm
(C) 0.01 cm (D) 0.02 cm
34. The least count of the meter rod is 0.1 cm. What is the permissible error in the length of the rod measured with it?
(A)  0.2 cm (B)  0.1 cm
(C)  0.05 cm (D)  0.05 cm
35. The mass of a body is 20.000 g and its volume is 10.00 cm3. If the measured values are expressed up to
the correct significant figures, the maximum error in the value of density is
(A) 0.001 g cm–3 (B) 0.010 g cm–3
(C) 0.100 g cm–3 (D) none of the above
36. Which of the following is the least precisely recorded observation?
(A) 5 m (B) 500 cm
(C) 5000 mm (D) 5000.0 mm
1
The frequency of oscillation of a stretched string is given by f   F /   . Here l = length and F =
1/2
37.
2l
force. What is the dimensional formula for ?
(A) MLT 1 (B) ML1T 0
(C) M 1L0T 0 (D) M 0 L0T 0
Given that  dx  x  a  where a constant. Using dimensional analysis, the value of n is
38.  a n sin 1 
2ax  x 2
 a 
(A) 1 (B) 0
(C) –1 (D) none of the above
39. The volume V of the liquid crossing through a tube is related to the area of cross section A, velocity v and
time t as V  A v  t  . Which of the following relations is correct?
(A)      (B)     
(C)      (D)     
40. The velocity v of the water waves is proportional to   g where  = wavelength,  = density and g
  

= acceleration due to gravity. Which of the following relation is correct?


(A)      (B)     
(C)      (D)     

97
PHYSICS UNITS & DIMENSIONS

41. Given that I = moment of inertia, pm  magnetic dipole moment and B = magnetic induction, then the
I
dimensional formula for is same as that of
pm B
(A) time (B) length
(C) time2 (D) length2
42. Suppose the kinetic energy of a body oscillating with amplitude A and at a distance x is given by
Bx
K . The dimensions of B are the same as that of
x  a2
2

(A) work/time (B) work  distance


(C) work / distance (D) work  time
43. Given that q = charge, V = potential, C = capacitance, which of the following have the same dimensions?
2 2 2 2
(A) q / V , CV , q C (B) qV , C / V , q C
2 2 2 2
(C) qV , CV , q / C (D) qV , CV , q C
44. The dimensional formula for mechanical equivalent of heat is same as that for
(A) heat (B) work
(C) heat  work (D) work/heat
45. The method of dimensional analysis can be used to derive which of the following relation?
(B) A sin t  kx 
0  t
(A) N e
1 2 1 2
(C) mv  I  (D) none of the above
2 2
46. The dimensional formula for resistance conductance is same as that for
(A) dielectric constant (B) permittivity
(C) permeability (D) potential energy
47. Given that the displacement of an oscillating particle is given by y  A sin  Bx  Ct  D . The
dimensional formula be (ABCD) is
(A) M 0 L1T 0 (B) M 0 L0T 1
(C) M 0 L1T 1 (D) M 0 L0T 0
2
48. Given the L = inductance, I = electric current, then the dimensions of LI are same as that for
(A) power (B) magnetic flux
(C) potential (D) none of the above
49. Given that p = momentum, c = speed of light, then the dimensions of pc are same as that of
(A) power (B) force
(C) angular momentum (D) torque
50. Suppose speed of light (C), force (F) and kinetic energy (K) are taken as the fundamental units, then the
dimensional formula for mass will be
(A) KC 2 (B) KF 2
(C) CK 2 (D) FC 2
51. Suppose the velocity of light (C), acceleration due to gravity (g) and pressure (p) are taken as the
fundamental units. What will be the dimensional formula for mass in this system of units?
2 3 4
(A) p c g (B) p c g
4 3 3 3
(C) p c g (D) p g c

98
UNITS & DIMENSIONS PHYSICS

52. The Richardson equation given by I  AT 2e B / KT . The dimensional formula for AB2 is same as that for
(A) IT 2 (B) kT
2
(C) Ik (D) Ik 2 / T
53. Which of the following is NOT dimensionless?
(A) Relative density (B) Relative velocity
(C) Relative refractive index (D) Relative permittivity
54. What is the dimensional formula for the Boltzmann’s constant?
(A) ML2T 2 1 (B) MLT 2 1
(C) ML2T 1 1 (D) ML1T 2 1
55. In a system of units, the Planck’s constant (h), the gravitational constant (G) and the speed of light (C)
are taken as the fundamental units. What is the dimensional formula of force in this system of units?
(A) h0G 1C 4 (B) h1G 0C 4
(C) h 1G 4C 0 (D) h4G 0C 2
56. kg/ms is the unit of
(A) surface energy (B) surface tension
(C) modulus of elasticity (D) viscosity
57. The unit of force as well as distance are doubled. How many times will the unit of kinetic energy be?
(A) 1/2 (B) 2
(C) 4 (D) 8
58. Which of the following is not expressed in suitable units ?
(A) [Surface energy] = N/m (B) [Stress] = J/m3
2
(C) [Energy density] = N/m (D) [Thurst] = J/m2
59. Temperature can be expressed as a derived unit in terms of
(A) Mass & length (B) Length & time
(C) Mass, length & time (D) none of the above
60. Which of the following is dimensionless?
(A) Moment of momentum (B) Moment of force
(C) Moment of inertia (D) None of the above
61. Which of the following will not have the dimensions of potential energy
(A) Torque  angular displacement (B) Rotational inertia  (angular frequency)2
(C) Position vector  applied force (D) Displacement  momentum
62. What is the dimensional formula for the universal gas constant R ?
(A) ML2T 2 1 mol 1 (B) MLT 2 1 mol 1
(C) ML2T 1 mol (D) MLT 1 mol
63. Energy E, velocity V and time T are taken as the fundamental units. What is the dimensional formula for
energy per unit area ?
(A) EVT (B) EV 1T 1
(C) EV 2T 2 (D) EV 1T 2
64. If area (A), velocity (V) and density (D) are taken as the fundamental units, what is the dimensional
formula for force ?
(A) AV 2 D (B) A2VD
(C) AVD 2 (D) AVD

99
PHYSICS UNITS & DIMENSIONS

65. If we double the amount of quantity in its unit, what happens to the numerical value of the physical
quantity measured with it ?
(A) Halved (B) Doubled
(C) Tripled (D) Quadrupled
66. If force (F), acceleration (A), time (T) are used as fundamental units, the dimensional formula for length will
be
(A) F 0 AT 2 (B) FA0T 2
(C) FA2T 0 (D) FAT
67. Given that K = kinetic energy, V = velocity, T = time. If they are chosen as the fundamental units, then
what is the dimensional formula for surface tension ?
(A) KV 2T 2 (B) KV 2T 2
(C) K 2V 2T 2 (D) K 2V 2T 2
68. A stone is lying in a fluid stream. the force acting on it depends on the density of the fluid, the velocity of
flow and the maximum area of cross–section perpendicular to the direction of flow. The force F and the
velocity v of flow are related as
1
(A) F  (B) F  v
v
1
(C) F  v 2 (D) F 
v2
69. The resistance experienced by a body lying in a stream of fluid depends on velocity of flow, density of
the fluid, length of the body and viscosity of the fluid. If the resistance (F) arise directly as the square of
velocity, then it is related to the coefficient of viscosity as
(A) F   (B) F  
1 0

(C) F   (D) F  
2

70. How are the numerical value (N) and unit (U) of a physical quantity related?
(A) N  U (B) N  U
1 1
(C) N  (D) N 
U U
71. Which of the following is the dimensional formula for capacitance?
(A) M1L2T4 A2 (B) M1L2T4A2
(C) M1L2T4 A2 (D) M1L2T4 A2
72. In view of their units and dimensions, which of the following is different from the other three?
(A) Phase difference between two waves (B) Mechanical equivalent of heat
(C) Loudness of sound (D) Poisson’s ratio
RT  aV / RT
73. Given that p  e . The dimensional formula of a is same as that of
V b
Here P, V, R & T represent Pressure, Volume, Gas constant and temperature respectively.
(A) V (B) p
(C) T (D) R
74. The units of length, velocity and force are doubled. Which of the following is the correct change in the
other units?
(A) Unit of time is doubled (B) Unit of mass is doubled
(C) Unit of momentum is doubled (D) Unit of energy is doubled

100
UNITS & DIMENSIONS PHYSICS

75. A soap bubble oscillates with time period T, which in turn depends on the pressure (p), density    , and

surface tension   . Which of the following correctly represents the expression for T 2 ?

 2  p3
(A) (B)
p3 
p 3 
(C) (D)
 p 3
76. If A = amplitude, t = time, y = displacement, c = speed of wave, T = time period,   2 / T. Then, on
the basis of dimensional analysis, which of the following expression is correct?
At At
(A) y  sin ct (B) y  cos ct / A
T T
A
(C) y  sin ct (D) y  A t cos ct
T
77. In a system of units, the units of length, mass and time are 10 cm, 10 g and 0.1s respectively. The unit of
force will be equivalent to
(A) 0.1 N (B) 1 N
(C) 10 N (D) 100 N
78. What is the number of significant figures in  3.20  4.80 105 ?
(A) 5 (B) 4
(C) 3 (D) 2
79. In which of the following numerical values, all zeros are significant?
(A) 0.2020 (B) 20.2 10
(C) 20.20 (D) none of the above
1 2
80. The Bernoulli’s equation may be written as p   v  h  g  K (a constant). The unit of v is same as that of
2
(A) pressure (B) thrust
(C) angle (D) none of the above
81. Which of the following pairs has the same units?
(A) Wavelength and Rydberg constant (B) Relative velocity & Relative density
(C) Thermal capacity & Boltzmann constant (D) Time period and acceleration gradient
82. The “pascal second” is equivalent to the unit of
(A) thrust (B) surface energy
(C) stress (D) viscosity
83. The product of pressure and volume have the same units as the product of
(A) charge and potential (B) electric field strength and distance
(C) electromotive force and capacitance (D) magnetic moment and magnetic induction
84. Which of the following has ‘meter kelvin” as the unit?
(A) Rydberg constant (B) Wein’s constant
(C) Solar constant (D) Gas constant
85. The unit of latent heat is equivalent to the unit of
(A) (force)2 (B) (acceleration)2
(C) (velocity)2 (D) (density)2

101
PHYSICS UNITS & DIMENSIONS

86. The dimensional formula for acceleration, velocity and length are  2 ,  1 and  . What is the
dimensional formula for the coefficient of friction?
(A)  (B)  1  0  0
(C)  0  1  0 (D)  0  0  1
1 2
87. The Bernouli’s equation is given by p   v  h  g  K where p = pressure,   density,  
2
speed, h = height of liquid column, g = acceleration due to gravity and K is a constant. The dimensional
formula for K is same as that for
(A) velocity gradient (B) pressure gradient
(C) modulus of elasticity (D) thrust
88. A force F is applied on a square plate of side L. If the percentage error in the determination of L is 2%
and that in F is 4%, the permissible error in pressure is
(A) 2% (B) 4%
(C) 6% (D) 8%
89. If x  a  b and a and b are the errors in the measurement of a and b respectively, then the
maximum percentage error in the value of x will be
 a b   a b 
(A)    100 (B)    100
 a b   a b 
 a b   a b 
(C)    100 (D)    100
 a b a b   a b a b 
90. Which of the following don’t have the same dimensional formula as the velocity? Given that 0 
permeability of free space, 0  permittivity of free space, v = frequency,   wavelength, p 
pressure,   density,   angular frequency, k = wave number.
1
(A) (B) 
 0 0
(C) p/ (D) k
91. The damping force for a body moving through a fluid is proportional to velocity. What will be the
dimensional formula for the constant of proportionality?
(A) MLT 1 (B) ML0T 1
(C) ML1T (D) ML1T 1

 
92. The velocity of a spherical ball through a viscous liquid is given by    0 1  ekt , where  0 is the
initial velocity and t represents time. If k depends on radius of ball (r), coefficient of viscosity ( ) and
mass of the ball (m), then
(A) k  mr /  (B) k   m / r
(C) k  r / m (D) k  mr
2
93. A cube has a side 1.2 10 m. Its volume will be recorded as –
(A) 1.728 × m3 (B) 1.72 × m3
(C) 1.7 × m3 (D) .72 × m3

102
UNITS & DIMENSIONS PHYSICS

94. Which of the following physical quantities do not have the same dimensions?
(A) Pressure, Young’s modulus, stress (B) Electromotive force, voltage, potential
(C) Heat, Work, Energy (D) Electric dipole, electric field, flux
95. In the formula, X  3YZ 2 , X and Z have dimensions of capacitance and magnetic induction
respectively. What are dimensions of Y in MKSQ system?
(A) [M–3L–1T3Q4] (B) [M–3L–2T4Q4]
(C) [M–2L–2T4Q4] (D) [M–3L–2T4Q1]

ANSWER KEY

Questions Answers Questions Answers Questions Answers


1 B 33 C 65 A
2 D 34 B 66 A
3 A 35 D 67 B
4 C 36 B 68 C
5 A 37 B 69 B
6 B 38 B 70 C
7 A 39 D 71 B
8 B 40 B 72 D
9 A 41 C 73 B
10 D 42 B 74 C
11 A 43 C 75 A
12 A 44 D 76 B
13 C 45 D 77 A
14 A 46 A 78 C
15 B 47 B 79 C
16 A 48 D 80 D
17 D 49 D 81 C
18 D 50 A 82 D
19 D 51 C 83 A
20 A 52 C 84 B
21 A 53 B 85 C
22 A 54 A 86 D
23 A 55 A 87 C
24 C 56 D 88 D
25 D 57 C 89 C
26 A 58 D 90 D
27 A 59 D 91 B
28 C 60 D 92 C
29 C 61 D 93 C
30 B 62 A 94 D
31 A 63 C 95 B
32 B 64 A

103
PHYSICS UNITS & DIMENSIONS

WINDOW TO JEE MAIN

Questions based on Measurement of length


1. A student measured the length of a rod and wrote it as 3.50 cm. Which instrument did he use to measure
it? [JEE (M) 2014]
(A) A meter scale
(B) A Vernier caliper where the 10 divisions in Vernier scale matches with 9 division in main scale and
main scale has 10 divisions in 1 cm.
(C) A screw gauge having 100 divisions in the circular scale and pitch as 1 mm.
(D) A screw gauge having 50 divisions in the circular scale and pitch as 1 mm.
Questions based on Dimensional formulae & Dimensional equation

B2
2. The dimensions of , where B is magnetic field and  0 is the magnetic permeability of vacuum, is
20
[JEE (M) 2020]
2 1 2
(A) MLT (B) ML T
(C) ML2T 1 (D) ML2T 2
3. The dimension of stopping potential V0 in photoelectric effect in units of Planck’s constant ‘h’, speed of
light ‘c’ and gravitational constant ‘G’ and ampere A is : [JEE (M) 2020]
0 3 2 1 2/3 5/3 1/3 1
(A) h c G A (B) h c G A
(C) h1/3G2/3c1/3 A1 (D) h0c5G 1 A1
hc5
4. A quantity f is given by f  where c is the speed of light, G universal gravitational constant and h
G
is the Planck’s constant. Dimension of f is that of: [JEE (M) 2020]
(A) volume (B) energy
(C) momentum (D) area
5. Let l, r, c and v represent inductance, resistance, capacitance and voltage, respectively. The dimension of
l
in SI units will be: [JEE (M) 2019]
rcv
(A) [LTA] (B) [LA–2]
(C) [A–1] (D) [LT2]
6. Let  0  denote the dimensional formula of the permittivity of vacuum. If M = mass, L = length,
T = time and A = electric current, then: [JEE (M) 2013]
(A)  0   M L T A 
1 3 2
(B)  0   M L T A 
1 3 5 2

(C)  0   M1L2T1A 2  (D)  0   M1L2T1A 


7. The dimension of magnetic field in M, L, T and C (coulomb) is given as [2008]
(A) MLT–1C–1 (B) MT2C–2
–1 –1
(C) MT C (D) MT–2C–1
8. Out of the following pair, which one does not have identical dimensions is [2005]
(A) impulse and momentum
(B) angular momentum and planck’s constant

104
UNITS & DIMENSIONS PHYSICS

(C) work and torque


(D) moment of inertia and moment of a force (towards north-west)
9. Which one of the following represents the correct dimensions of the coefficient of viscosity?
[2004]
(A) ML–1T–1 (B) MLT–1
(C) ML–1T–2 (D) ML–2T–2
10. The physical quantities not having same dimensions are [2003]
(A) torque and work (B) momentum and planck’s constant
(D) speed and  0 0 
1/2
(C) stress and young’s modulus
1
11. Dimension of , where symbols have their usual meaning, are [2003]
 0 0
(A) [L–1T] (B) [L–2T2]
(C) [L2T–2] (D) [LT–1]
12. Identify the pair whose dimensions are equal [2002]
(A) torque and work (B) stress and energy
(C) force and stress (D) force and work
Questions based on Derivation of formulae
13. If speed (V), acceleration (A) and force (F) are considered as fundamental units, the dimension of
Young's modulus will be: [JEE (M) 2019]
–2 2 2 –4 2
(A) V A F (B) V A F
(C) V–4A–2F (D) V–2A2F–2
14. The characteristic distance at which quantum gravitational effects are significant, the Planck length, can
be determined from a suitable combination of the fundamental physical constants G, and c. Which of
the following correctly gives the Planck length? [JEE (M) 2018]
2 3 2
(A) G c (B) G c
1/2
G 
(C)   (D) G1/2 2c
 c 
e2
15. A physical quantity of the dimensions of length that can be formed out of c, G and is [c is velocity
4 0
of light, G is universal constant of gravitation and e is charge]: [JEE (M) 2017]
1/2 1/2
2  e2  1  e2 
(A) c  G  (B) 2  
 4 0  c  G 4 0 
1/2
1 e2 1  e2 
(C) G
c 4 0
(D) 2 G 
c  4 0 
Questions based on Significant figures
16. The density of a material in SI units is 128 kg m–3. In certain units in which the unit of length is 25 cm
and the unit of mass is 50 g, the numerical value of density of the material is: [JEE (M) 2019]
(A) 410 (B) 640
(C) 16 (D) 40

105
PHYSICS UNITS & DIMENSIONS

17. The respective number of significant figures for the numbers 23.023, 0.0003 and 2.1  10–3 are
[2010]
(A) 5, 1, 2 (B) 5, 1, 5
(C) 5, 5, 2 (D) 4, 4, 2
18. A body of mass m = 3.513 kg is moving along the x-axis with a speed of 5.00 ms–1. The magnitude of its
momentum is recorded as [2008]
–1 –1
(A) 17.6 kg ms (B) 17.565 kg ms
–1
(C) 17.56 kg ms (D) 17.57 kg ms–1
Questions based on Accuracy and Precision
19. A simple pendulum is being used to determine the value of gravitational acceleration g at a constant
place. The length of the pendulum is 25.0 cm and a stop watch with 1 s resolution measures the time
taken for 40 oscillations to be 50 s. The accuracy in g is : [JEE (M) 2020]
(A) 4.40 % (B) 3.40 %
(C) 2.40 % (D) 5.40 %
Questions based on Various types of errors
20. The relative error in the determination of the surface area of a sphere is . Then the relative error in the
determination of its volume is: [JEE (M) 2018]
3
(A)  (B) 
2
5 2
(C)  (D) 
2 3
21. A student measures the time period of 100 oscillations of a simple pendulum four times. That data set is
90 s, 91 s, 95 s and 92 s. If the minimum division in the measuring clock is 1 s, then the reported mean
time should be : [JEE (M) 2016]
(A) 92  5.0s (B) 92  1.8s
(C) 92  3s (D) 92  2s

Questions based on Propagation of errors


22. The diameter and height of a cylinder are measured by a meter scale to be 12.6 ± 0.1 cm and 34.2 ± 0.1 cm,
respectively. What will be the value of its volume in appropriate significant figures? [JEE (M) 2019]
(A) 4260 ± 80 cm3 (B) 4300 ± 80 cm3
(C) 4264.4 ± 81.0 cm3 (D) 4264 ± 81 cm3

23. The relative uncertainty in the period of a satellite orbiting around the earth is 10 –2. If the relative
uncertainty in the radius of the orbit is negligible, the relative uncertainty in the mass of the earth is:
[JEE (M) 2018]
(A) 6  10 –2
(B) 10 –2

(C) 2  10 –2
(D) 3  10–2
24. The density of a material in the shape of a cube is determined by measuring three sides of the cube and
its mass. If the relative errors in measuring the mass and length are respectively 1.5% and 1%, the
maximum error in determining the density is: [JEE (M) 2018]
(A) 3.5 % (B) 4.5 %
(C) 6 % (D) 2.5 %

106
UNITS & DIMENSIONS PHYSICS

25. The following observations were taken for determining surface tension T of water by capillary method:
Diameter of capillary, D = 1.25 × 10–2 m [JEE (M) 2017]
–2
rise of water, h = 1.45 × 10 m
rhg
Using g = 9.80 m/s2 and the simplified relation T  103 N/m, the possible error in surface tension
2
is closest to :
(A) 2.4 % (B) 10 %
(C) 0.15 % (D) 1.5 %
L
26. The period of oscillation of a simple pendulum is T  2 . Measured value of L is 20.0 cm known
g
to 1 mm accuracy and time for 100 oscillations of the pendulum is found to be 90s using a wrist watch of
1 s resolution. The accuracy in the determination of g is: [JEE (M) 2015]
(A) 1% (B) 5%
(C) 2% (D) 3%
27. Resistance of a given wire is obtained by measuring the current flowing in it and the voltage difference
applied across it. If the percentage errors in the measurement of the current and the voltage difference are
3% each, then error in the value of resistance of the wire is: [2012]
(A) 6% (B) zero
(C) 1% (D) 3%
Questions based on Rounding off
28. For the four sets of three measured physical quantities as given below. Which of the following options is
correct? [JEE (M) 2020]
(i) A1  24.36, B1  0.0724, C1  256.2
(ii) A2  24.44, B2  16.082, C2  240.2
(iii) A3  25.2, B3  19.2812, C3  236.183
(iv) A4  25, B1  236.191, C4  19.5
(A) A1  B1  C1  A3  B3  C3  A2  B2  C2  A4  B4  C4
(B) A1  B2  C1  A2  B2  C2  A3  B3  C3  A4  B4  C4
(C) A4  B4  C4  A1  B1  C1  A2  B2  C2  A3  B3  C3
(D) A4  B4  C4  A1  B1  C1  A3  B3  C3  A2  B2  C2

Questions based on Errors-Miscellaneous


29. Consider an expanding sphere of instantaneous radius R whose total mass remains constant. The
expansion is such that the instantaneous density  remains uniform throughout the volume. The rate of
 1 d 
fractional change in density   is constant. The velocity v of any point on the surface of the
  dt 
expanding sphere is proportional to [JEE (M) 2017]
(A) R 3 (B) R
2 1
(C) R 3 (D)
R

107
PHYSICS UNITS & DIMENSIONS

Questions based on Experimental skills (Screw Gauge & Vernier Caliper)


30. If the screw on a screw-gauge is given six rotations, it moves by 3mm on the main scale. If there are 50
divisions on the circular scale the least count of the screw gauge is: [JEE (M) 2020]
(A) 0.01 cm (B) 0.02 mm
(C) 0.001 mm (D) 0.001 cm
31. The pitch and the number of divisions, on the circular scale, for a given screw gauge are 0.5 mm and 100
respectively. When the screw gauge is fully tightened without any object, the zero of its circular scale lies
3 divisions below the mean line. The readings of the main scale and the circular scale, for a thin sheet,
are 5.5 mm and 48 respectively, the thickness of this sheet is: [JEE (M) 2019]
(A) 5.755 m (B) 5.725 mm
(C) 5.740 m (D) 5.950 mm
32. The least count of the main scale of a screw gauge is 1 mm. The minimum number of divisions on its
circular scale required to measure 5mm diameter of wire is: [JEE (M) 2019]
(A) 50 (B) 100
(C) 200 (D) 500
33. A screw gauge with a pitch of 0.5 mm and a circular scale with 50 divisions is used to measure the
thickness of a thin sheet of Aluminium. Before starting the measurement, it is found that when the two
jaws of the screw gauge are brought in contact, the 45th division coincides with the main scale line and
that the zero of the main scale is barely visible. What is the thickness of the sheet if the main scale
reading is 0.5mm and the 25th division coincides with the main scale line? [JEE (M) 2016]
(A) 0.80 mm (B) 0.70 mm
(C) 0.50 mm (D) 075 mm
34. A spectrometer gives the following reading when used to measure the angle of a prism.
Main scale reading: 58.5 degree
Vernier scale reading: 09 divisions [2012]
Given that 1 division on main scale corresponds to 0.5 degree. Total divisions on the Vernier scale is 30
and match with 29 divisions of the main scale. The angle of the prism from the above data:
(A) 58.59 degree (B) 58.77 degree
(C) 58.65 degree (D) 59 degree
35. A screw gauge gives the following reading when used to measure the diameter of a wire.
Main scale reading: 0 mm
Circular scale reading: 52 divisions
Given that 1 mm on main scale corresponds to 100 divisions of the circular scale. The diameter of wire
from the above data is: [2011]
(A) 0.052 cm (B) 0.026 cm
(C) 0.005 cm (D) 0.52 cm
36. In an experiment the angles are required to be measured using an instrument, 29 divisions of the main
scale exactly coincide with the 30 divisions of the Vernier scale. If the smallest division of the main scale
is half-a degree (= 0.5o), then the least count of the instrument is: [2009]
(A) half minute (B) one degree
(C) half degree (D) one minute
37. Two full turns of the circular scale of a screw gauge cover a distance of 1 mm on its main scale. The total number of
divisions on the circular scale is 50. Further, it is found that the screw gauge has a zero error of –0.03 mm. While
measuring the diameter of a thin wire, a student notes the main scale reading of 3 mm and the number of circular
scale divisions in line with the main scale as 35. The diameter of the wire is: [2008]
(A) 3.32 mm (B) 3.73 mm
(C) 3.67 mm (D) 3.38 mm

108
UNITS & DIMENSIONS PHYSICS

ANSWER KEY

Questions Answers Questions Answers


1 B 20 A
2 B 21 D
3 D 22 A
4 B 23 C
5 C 24 B
6 B 25 D
7 C 26 D
8 D 27 A
9 A 28 A
10 B 29 B
11 C 30 D
12 A 31 B
13 B 32 C
14 C 33 A
15 D 34 C
16 D 35 A
17 A 36 D
18 A 37 D
19 A

109
PHYSICS UNITS & DIMENSIONS

WINDOW TO NEET

Questions based on Dimensional formulae & Dimensional equation


1. The pair of quantities having same dimensions is [NEET Kar. 2013]
(A) Young’s modulus and energy (B) impulse and surface tension
(C) angular momentum and work (D) work and torque
1
2. The dimension of ( 0 0 ) 2 are [2012 M, 2011]
1/2 1/2 1
(A) [L T ] (B) [L T]
(C) [LT ] 1
(D) [L1/2 T1/2 ]

3. The dimension of
1
 0 E 2 , where 0 is permittivity of free space and E is electric field, is
2
[2010]
2 2 1 2
(A) ML T (B) ML T
2 1
(C) ML T (D) MLT1
4. Which two of the following five physical parameters have the same dimensions? [2008]
(a) Energy density (b) Refractive index
(c) Dielectric constant (d) Young’s modulus
(e) Magnetic field
(A) (b) and (d) (B) (c) and (e)
(C) (a) and (d) (D) (a) and (e)
5. Dimensions of resistance in an electric circuit, in terms of dimension of mass M, of length L, of time T
and of current I, would be [2007]
2 2 2 1 1
(A) ML T (B) ML T I
2 3 2
(C) ML T I (D) ML2T-3I1
b
6. The velocity of v of a particle at time t is given by v  at  , where a, b and c are constant. The
t c
dimensions of a, b and c are respectively [2006]
(A) L2 , T and LT2 (B) LT 2 , LT and L
2
(C) L, LT and T 2 (D) LT ,L and T
7. The ratio of the dimension of Planck’s constant and that of the moment of inertia is the dimension of
[2005]
(A) time (B) frequency
(C) angular momentum (D) velocity
8. The dimensions of universal gravitational constant are [2004]
2 2 1 1 3 2
(A) M L T (B) M L T
2 1
(C) ML T (D) M2 L3T2
9. The unit of permittivity of free space, 0 is [2004]
(A) Coulomb2 / (Newton-metre)2 (B) Coulomb/ Newton-metre
(C) Newton-meter2 / Coulomb (D) Coulomb2 / Newton-metre2

110
UNITS & DIMENSIONS PHYSICS

10. The unit of the Stefan-Boltzmann’s constant is [2002]


2 4 2
(A) W/m K (B) W/m
2
(C) W/m K (D) W/m2 K2
11. The dimensions of Planck’s constant are same as [2001]
(A) energy (B) power
(C) momentum (D) angular momentum
12. Which one of the following groups have quantities that do not have the same dimensions?
[2000]
(A) pressure, stress (B) velocity, speed
(C) force, impulse (D) work, energy
Questions based on Derivation of formulae

e2
13. A physical quantity of the dimensions of length that can be formed out of c, G and is [c is velocity
4 0
of light, G is universal constant of gravitation and e is charge] [2017]
1/2 1/2
 e2 
2 1  e2 
(A) c G  (B) 2  
 4 0  c  G 4 0 
1/2
1 e2 1  e2 
(C) G (D) 2 G 
c 4 0 c  4 0 
14. If energy (E), velocity (V) and time (T) are chosen as the fundamental quantities, the dimensional formula
of surface tension will be: [2015]
(A) [ EV 1T 2 ] (B) [ EV 2T 2 ]
(C) [ E 2V 1T 3 ] (D) [ EV 2T 1 ]
15. If dimensions of critical velocity vc of a liquid flowing through a tube are expressed as [ x  y r x ] , where
 ,  and r are the coefficients of viscosity of liquid, density of liquid and radius of the tube respectively,
then the values of x, y and z are given by : [2015 RS]
(A) –1, –1, 1 (B) –1, –1, –1
(C) 1, 1, 1 (D) 1, –1, –1
16. If force (F), velocity (V) and time (T) are taken as fundamental units, then the dimensions of mass are:
[2014]
(A) [ FVT 1 ] (B) [ FVT 2 ]
(C) [ FV 1T 1 ] (D) [ FV
1
T]
Questions based on Conversion of one system

17. The density of material in CGS system of units is 4g/cm3 . In a system of units in which unit of length is
10 cm and unit of mass is 100 g, the value of density of material will be [2011 M]
(A) 0.4 (B) 40
(C) 400 (D) 0.04

111
PHYSICS UNITS & DIMENSIONS

Questions based on Propagation of errors


18. In an experiment four quantities a, b, c and d are measured with percentage error 1%, 2 %, 3% and 4%
a 3b 2
respectively. Quantity P is calculated as follows P = % error in P is: [2013]
cd
(A) 10% (B) 7%
(C) 4% (D) 14%
19. If the error in the measurement of radius of a sphere is 2%, then the error in the determination of volume
of the sphere will be: [2008]
(A) 4% (B) 6%
(C) 8% (D) 2%
Questions based on Experimental skills (Screw Gauge & Vernier Caliper)
20. A student measured the diameter of a small steel ball using a screw gauge of least count 0.001 cm. The
main scale reading is 5 mm and zero of circular scale division coincides with 25 divisions above the
reference level. If screw gauge has a zero error of –0.004 cm, the correct diameter of the ball is:
[NEET 2018]
(A) 0.521 cm (B) 0.525 cm
(C) 0.529 cm (D) 0.053 cm

ANSWER KEY

Questions Answers Questions Answers


1 D 11 D
2 C 12 C
3 B 13 D
4 C 14 B
5 C 15 D
6 D 16 D
7 B 17 B
8 B 18 D
9 D 19 B
10 A 20 C

112

You might also like